151
IA RTM COMPILATIONS PRELIMS 2022 February 2022 INSIGHTSIAS SIMPLIFYING IAS EXAM PREPARATION www.insightsactivelearn.com | www.insightsonindia.com

RTM-Feb-2022- Compilation - INSIGHTSIAS

Embed Size (px)

Citation preview

IA

RTM COMPILATIONS

PRELIMS 2022

February 2022

INSIGHTSIAS

SIMPLIFYING IAS EXAM PREPARATION

www.insightsactivelearn.com | www.insightsonindia.com

Revision Through MCQs (RTM) Compilation (Feb - 2022)

Telegram: https://t.me/insightsIAStips Youtube: https://www.youtube.com/channel/UCpoccbCX9GEIwaiIe4HLjwA

2

Revision Through MCQs (RTM) Compilation (Feb - 2022)

Telegram: https://t.me/insightsIAStips Youtube: https://www.youtube.com/channel/UCpoccbCX9GEIwaiIe4HLjwA

3

Table of Contents

RTM- REVISION THROUGH MCQS –1st-Feb-2022 .................................................................. 4

RTM- REVISION THROUGH MCQS –2nd-Feb-2022 ................................................................. 9

RTM- REVISION THROUGH MCQS –3rd-Feb-2022 ................................................................ 15

RTM- REVISION THROUGH MCQS –4th-Feb-2022 ................................................................ 21

RTM- REVISION THROUGH MCQS –5th-Feb-2022 ................................................................ 26

RTM- REVISION THROUGH MCQS –7th-Feb-2022 ................................................................ 34

RTM- REVISION THROUGH MCQS –8th-Feb-2022 ................................................................ 41

RTM- REVISION THROUGH MCQS –9h-Feb-2022................................................................. 47

RTM- REVISION THROUGH MCQS –10th-Feb-2022 .............................................................. 53

RTM- REVISION THROUGH MCQS –11th-Feb-2022 .............................................................. 59

RTM- REVISION THROUGH MCQS –12th-Feb-2022 .............................................................. 67

RTM- REVISION THROUGH MCQS –14th-Feb-2022 .............................................................. 72

RTM- REVISION THROUGH MCQS –15th-Feb-2022 .............................................................. 78

RTM- REVISION THROUGH MCQS –16th-Feb-2022 .............................................................. 84

RTM- REVISION THROUGH MCQS –17th-Feb-2022 .............................................................. 92

RTM- REVISION THROUGH MCQS –18th-Feb-2022 .............................................................. 99

RTM- REVISION THROUGH MCQS –21st-Feb-2022 ............................................................ 105

RTM- REVISION THROUGH MCQS –22nd-Feb-2022 ........................................................... 112

RTM- REVISION THROUGH MCQS –23nd-Feb-2022 ........................................................... 117

RTM- REVISION THROUGH MCQS –24th-Feb-2022 ............................................................ 124

RTM- REVISION THROUGH MCQS –25th-Feb-2022 ............................................................ 132

RTM- REVISION THROUGH MCQS –26th-Feb-2022 ............................................................ 138

RTM- REVISION THROUGH MCQS –28th-Feb-2022 ............................................................ 144

Revision Through MCQs (RTM) Compilation (Feb - 2022)

Telegram: https://t.me/insightsIAStips Youtube: https://www.youtube.com/channel/UCpoccbCX9GEIwaiIe4HLjwA

4

RTM- REVISION THROUGH MCQS –1st-Feb-2022

1. Consider the following statements about African Union (AU):

1. It is a continental union consisting of more than 71 countries of the continent of

Africa.

2. Its secretariat is located in Addis Ababa, Ethiopia.

Which of the statements given above is/are correct?

(a) 1 only

(b) 2 only

(c) Both 1 and 2

(d) Neither 1 nor 2

Ans: (b)

Explanation:

• The African Union (AU) is a continental union consisting of 55 countries of

the continent of Africa, with exception of various territories of European

possessions located in Africa.

• The bloc was founded on 26 May 2001 in Addis Ababa, Ethiopia and launched

on 9 July 2002 in South Africa.

• The intention of the AU is to replace the Organisation of African Unity (OAU),

established on 25 May 1963 in Addis Ababa by 32 signatory governments.

• The AU’s secretariat, the African Union Commission, is based in Addis

Ababa.

Refer: https://www.insightsonindia.com/2022/02/01/african-union-suspends-burkina-

faso-over-mutiny/

2. Consider the following pairs:

Regions sometimes mentioned in news: Country

1. Addis Ababa : Mauritius

2. Bamako : Mali

3. Rabat : Burkina Faso

Which of the pairs given above are correctly matched?

(a) 1 and 2 only

(b) 2 only

(c) 2 and 3 only

(d) 3 only

Ans: (b)

Explanation:

Revision Through MCQs (RTM) Compilation (Feb - 2022)

Telegram: https://t.me/insightsIAStips Youtube: https://www.youtube.com/channel/UCpoccbCX9GEIwaiIe4HLjwA

5

• Addis Ababa: Ethiopia

• Bamako: Mali

• Rabat: Morocco

Refer: https://www.insightsonindia.com/2022/02/01/african-union-suspends-burkina-

faso-over-mutiny/

3. Consider the following statements about UNCITRAL:

1. It is a subsidiary body of the U.N. General Assembly (UNGA)

2. It has been adopted by 49 countries, including Singapore.

3. It is responsible for helping to facilitate international trade and investment.

Which of the statements given above is/are correct?

(a) 1 and 2 only

(b) 2 and 3 only

(c) 1 and 3 only

(d) 1, 2 and 3

Ans: (d)

Explanation:

• United Nations Commission on International Trade Law (UNCITRAL) is a

subsidiary body of the U.N. General Assembly (UNGA) responsible for helping to

facilitate international trade and investment.

• It has been adopted by 49 countries, including Singapore, UK, US and South

Africa.

• UNCITRAL allows foreign professionals and creditors direct access to domestic

courts and enables them to participate in and commence domestic insolvency

proceedings.

Refer: https://www.insightsonindia.com/2022/02/01/united-nations-commission-on-

international-trade-law-uncitral/

4. Consider the following statements about MGNREGA:

1. It was introduced in 2005 as a social measure that guarantees “the right to work”.

2. A minimum of 110 job seekers shall apply to sanction a new work under MGNREGA.

3. It mandates Gram Sabhas to recommend the works that are to be undertaken.

Which of the statements given above is/are correct?

(a) 1 and 2 only

(b) 2 and 3 only

(c) 1 and 3 only

(d) 1, 2 and 3

Revision Through MCQs (RTM) Compilation (Feb - 2022)

Telegram: https://t.me/insightsIAStips Youtube: https://www.youtube.com/channel/UCpoccbCX9GEIwaiIe4HLjwA

6

Ans: (c)

Explanation:

• The scheme was introduced in 2005 as a social measure that guarantees “the

right to work”.

• The key tenet of this social measure and labour law is that the local government

will have to legally provide at least 100 days of wage employment in rural

India to enhance their quality of life.

• The act mandates Gram sabhas to recommend the works that are to be

undertaken and at least 50% of the works must be executed by them.

Refer: https://www.insightsonindia.com/2022/02/01/mgnrega/

5. Which of the following statements is/are correct regarding the benefits of Food

Fortification?

1. It is a safe method of improving nutrition among people.

2. It does not require any changes in food habits and patterns of people.

3. It does not alter the characteristics of the food.

Select the correct answer using the code below:

(a) 1 and 2 only

(b) 2 and 3 only

(c) 1 and 3 only

(d) 1, 2 and 3

Ans: (d)

Explanation:

• Since the nutrients are added to staple foods that are widely consumed, this is

an excellent method to improve the health of a large section of the population, all

at once.

• Fortification is a safe method of improving nutrition among people. The

addition of micronutrients to food does not pose a health risk to people.

• It does not require any changes in food habits and patterns of people. It is

a socio-culturally acceptable way to deliver nutrients to people.

• It does not alter the characteristics of the food—the taste, the feel, the look.

• It can be implemented quickly as well as show results in improvement of

health in a relatively short period of time.

• This method is cost-effective especially if advantage is taken of the existing

technology and delivery platforms.

Refer: https://www.insightsonindia.com/2022/02/01/fortified-rice-3/

Revision Through MCQs (RTM) Compilation (Feb - 2022)

Telegram: https://t.me/insightsIAStips Youtube: https://www.youtube.com/channel/UCpoccbCX9GEIwaiIe4HLjwA

7

6. With reference to National Policy on Biofuels (NBP) -2018, consider the following

statements:

1. The Policy envisages an indicative target of blending 20% ethanol in diesel by 2030.

2. The Policy categorises biofuels as “Basic Biofuels” and “Advanced Biofuels”.

Which of the statements given above is/are correct?

(a) 1 only

(b) 2 only

(c) Both 1 and 2

(d) Neither 1 nor 2

Ans: (b)

Explanation:

• S1: The National Policy on Biofuels-2018 approved by the Government envisages

an indicative target of 20% blending of ethanol in petrol and 5% blending of

bio-diesel in diesel by 2030.

• S2: The Policy categorises biofuels as “Basic Biofuels” viz. First Generation (1G)

bioethanol & biodiesel and “Advanced Biofuels” – Second Generation (2G)

ethanol, Municipal Solid Waste (MSW) to drop-in fuels, Third Generation (3G)

biofuels, bio-CNG etc. to enable extension of appropriate financial and fiscal

incentives under each category.

Refer: https://www.insightsonindia.com/2022/02/01/ethanol-as-an-alternate-fuel-2/

7. Among Indian States, which one of following has largest area under forests in 2021?

(a) Madhya Pradesh

(b) Arunachal Pradesh

(c) Chhattisgarh

(d) Odisha

Ans: (a)

Explanation:

• Among Indian States, Madhya Pradesh with 11% of India’s total forest

cover, had the largest area under forests in 2021, followed by Arunachal

Pradesh (9%), Chhattisgarh (8%), Odisha (7%) and Maharashtra (7%).

Refer: facts for prelims: https://www.insightsonindia.com/2022/02/01/mission-2022-

insights-daily-current-affairs-pib-summary-01-february-2022/

8. Consider the following statements about BharatNet:

1. It was originally launched in 2005 as the National Optical Fibre Network (NOFN) and

renamed as Bharat-Net in 2012.

Revision Through MCQs (RTM) Compilation (Feb - 2022)

Telegram: https://t.me/insightsIAStips Youtube: https://www.youtube.com/channel/UCpoccbCX9GEIwaiIe4HLjwA

8

2. It seeks to provide connectivity to 2.5 lakh Gram Panchayats (GPs) through optical

fibre.

3. It is a flagship mission implemented by Bharat Broadband Network Ltd. (BBNL).

Which of the statements given above is/are correct?

(a) 1 and 2 only

(b) 2 and 3 only

(c) 1 and 3 only

(d) 1, 2 and 3

Ans: (b)

Explanation: About BharatNet:

• BharatNet Project was originally launched in 2011 as the National Optical

Fibre Network (NOFN) and renamed as Bharat-Net in 2015.

• It seeks to provide connectivity to 2.5 lakh Gram Panchayats (GPs) through

optical fibre.

• It is a flagship mission implemented by Bharat Broadband Network Ltd.

(BBNL).

• The objective is to facilitate the delivery of e-governance, e-health, e-

education, e-banking, Internet and other services to rural India.

Refer: https://www.insightsonindia.com/2021/10/22/bharatnet-project-3/

9. “Production Gap Report” is released by which of the following?

(a) International Energy Agency

(b) World Bank

(c) International Monetary Fund

(d) United Nations Environment Programme

Ans: (d)

Explanation:

• The United Nations Environment Programme’s (UNEP) latest Production Gap

Report has revealed that 15 of the top fossil fuel producing countries,

including India, are not prepared to meet the requirements of the 2015

Paris Climate Agreement.

• Paris Agreement seeks to keep global warming “well below 2 degrees” above pre-

industrial levels.

Refer: https://www.insightsonindia.com/2021/10/22/indias-fossil-fuel-production-

exceeds-paris-agreement-goals/

Revision Through MCQs (RTM) Compilation (Feb - 2022)

Telegram: https://t.me/insightsIAStips Youtube: https://www.youtube.com/channel/UCpoccbCX9GEIwaiIe4HLjwA

9

10. Bovine Mastitis is caused by:

1. Virus

2. Fungus

3. Bacteria

Select the correct answer using the code below:

(a) 1 only

(b) 2 and 3 only

(c) 3 only

(d) 1, 2 and 3

Ans: (d)

Explanation:

• Bovine Mastitis is a common infectious disease, which affects farm

productivity due to fall in milk quality, thus impacting income-generating

activities.

• It is a potentially fatal mammary gland infection, which is most common in dairy

cattle worldwide.

• Mastitis is caused by microorganisms ranging from virus, mycoplasma,

fungus and bacteria. Physical injury to the mammary region, poor hygiene

and/or trauma, also cause this condition.

Refer: facts for prelims: https://www.insightsonindia.com/2021/10/22/mission-2022-

insights-daily-current-affairs-pib-summary-22-october-2021-2/

RTM- REVISION THROUGH MCQS –2nd-Feb-2022

11. Consider the following statements:

1. The Constitution of India empowers the Supreme Court and High Court to punish

people for their respective contempt.

2. The Contempt of Courts Act of 1971 defines the power of the High Court to punish

contempts of its subordinate courts.

Which of the statements given above is/are correct?

(a) 1 only

(b) 2 only

(c) Both 1 and 2

(d) Neither 1 nor 2

Ans: (c)

Revision Through MCQs (RTM) Compilation (Feb - 2022)

Telegram: https://t.me/insightsIAStips Youtube: https://www.youtube.com/channel/UCpoccbCX9GEIwaiIe4HLjwA

10

Explanation:

• Article 129 and 215 of the Constitution of India empowers the Supreme Court

and High Court respectively to punish people for their respective contempt.

• Section 10 of The Contempt of Courts Act of 1971 defines the power of the High

Court to punish contempts of its subordinate courts.

• The Constitution also includes contempt of court as a reasonable restriction to

the freedom of speech and expression under Article 19, along with elements like

public order and defamation.

Refer: https://www.insightsonindia.com/2022/02/02/contempt-of-court-7/

12. Recently, a new scheme PM-DevINE launched to fund infrastructure and social

development projects in which one of the following regions of India?

(a) Andaman and Nicobar

(b) Jammu and Kashmir

(c) Deccan region

(d) North-East region

Ans: (d)

Explanation: Prime Minister’s Development Initiative for North-East (PM-DevINE):

• New scheme PM-DevINE launched to fund infrastructure and social development

projects in the North-East.

• An initial allocation of Rs. 1,500 crore made to enable livelihood activities for

youth and women under the scheme.

Refer: https://www.insightsonindia.com/2022/02/02/highlights-of-the-union-budget-

2022/

13. Consider the following statements ‘Nal Se Jal’ Yojana:

1. It aims to provide piped drinking water to every rural home by 2024.

2. It is a component of the government’s Jal Jivan Mission.

Which of the statements given above is/are correct?

(a) 1 only

(b) 2 only

(c) Both 1 and 2

(d) Neither 1 nor 2

Ans: (c)

Explanation:

• Launched in 2019.

• Nodal Agency: Ministry of Jal Shakti

Revision Through MCQs (RTM) Compilation (Feb - 2022)

Telegram: https://t.me/insightsIAStips Youtube: https://www.youtube.com/channel/UCpoccbCX9GEIwaiIe4HLjwA

11

• Aim: To provide piped drinking water to every rural home by 2024

• It is a component of the government’s Jal Jivan Mission.

Refer: https://www.insightsonindia.com/2022/02/02/nal-se-jal-yojana/

14. Consider the following statements about Jal Jeevan Mission:

1. It is under the Ministry of Jal Shakti.

2. It was launched in 2021.

3. It envisages supply of 55 litres of water per person per day to every rural household.

Which of the statements given above is/are correct?

(a) 1 and 2 only

(b) 2 and 3 only

(c) 1 and 3 only

(d) 1, 2 and 3

Ans: (c)

Explanation:

• JJM envisages supply of 55 litres of water per person per day to every rural

household through Functional Household Tap Connections (FHTC) by 2024.

• It is under the Ministry of Jal Shakti.

• It was launched in 2019.

Refer: https://www.insightsonindia.com/2022/02/02/nal-se-jal-yojana/

15. Consider the following pairs:

River Source of river/origin

1. Godavari River Trimbak

2. Krishna River Mahabaleshwar

3. Kaveri River Bangrabalige valley

4. Penna Nandi Hills

Which of the above pairs is/are correctly matched?

(a) 1 and 2 only

(b) 1, 2 and 4 only

(c) 3 and 4 only

(d) 1, 2, 3 and 4

Ans: (b)

Explanation:

• Krishna, the fourth largest river in India, rises in western Maharashtra state in

the Western Ghats range near the town of Mahabaleshwar and flows through

Maharashtra, Karnataka, Telangana and Andhra Pradesh.

Revision Through MCQs (RTM) Compilation (Feb - 2022)

Telegram: https://t.me/insightsIAStips Youtube: https://www.youtube.com/channel/UCpoccbCX9GEIwaiIe4HLjwA

12

• Cauvery rises at Talakaveri in the Brahmagiri range in the Western Ghats and

flows through Karnataka and Tamil Nadu.

• Penna originates in Nandi Hills in the Chikballapur district and flows through

Karnataka, Tamil Nadu and Andhra Pradesh.

• Godavari which is the third largest river in India originates in the Western Ghats

near Thriambak Hills in the Nasik district of Maharashrta and flows through

Maharashtra, Telangana, Andhra Pradesh, Chhattisgarh and Orissa.

Refer: https://www.insightsonindia.com/2022/02/02/five-river-linking-projects-

announced-in-union-budget/

16. Consider the following statements:

1. An IFSC caters to customers outside the jurisdiction of the domestic economy.

2. Currently, GIFT-IFSC is the maiden international financial services centre in India.

Which of the statements given above is/are correct?

(a) 1 only

(b) 2 only

(c) Both 1 and 2

(d) Neither 1 nor 2

Ans: (c)

Explanation:

• An IFSC caters to customers outside the jurisdiction of the domestic

economy.

• Such centres deal with flows of finance, financial products and services across

borders.

• Currently, GIFT-IFSC is the maiden international financial services centre

in India.

Refer: https://www.insightsonindia.com/2022/02/02/international-arbitration-centre/

17. Consider the following statements about Singapore International Arbitration Centre

(SIAC):

1. It is a not-for-profit international arbitration organization.

2. It administers arbitrations under its own rules of arbitration only.

Which of the statements given above is/are correct?

(a) 1 only

(b) 2 only

(c) Both 1 and 2

(d) Neither 1 nor 2

Revision Through MCQs (RTM) Compilation (Feb - 2022)

Telegram: https://t.me/insightsIAStips Youtube: https://www.youtube.com/channel/UCpoccbCX9GEIwaiIe4HLjwA

13

Ans: (a)

Explanation:

• It is a not-for-profit international arbitration organisation based in Singapore,

which administers arbitrations under its own rules of arbitration and the United

Nations Commission on International Trade Law (UNCITRAL) Arbitration Rules.

Refer: https://www.insightsonindia.com/2022/02/02/international-arbitration-centre/

18. Consider the following statements about Central Bank Digital Currency (CBDC):

1. It is the legal tender issued by a central bank in a digital form.

2. It is exchangeable one-to-one with the fiat currency.

Which of the statements given above is/are correct?

(a) 1 only

(b) 2 only

(c) Both 1 and 2

(d) Neither 1 nor 2

Ans: (c)

Explanation:

• A Central Bank Digital Currency (CBDC), or national digital currency, is simply

the digital form of a country’s fiat currency. Instead of printing paper currency or

minting coins, the central bank issues electronic tokens. This token value is

backed by the full faith and credit of the government.

• Four major use cases of CBDC in the Indian context:

o ‘Fit-for-purpose’ money used for social benefits and other targeted

payments in a country. For such cases, the central bank can pay

intended beneficiaries pre-programmed CBDC, which could be accepted

only for a specific purpose.

o CBDCs could be used for faster cross-border remittance payments.

International collaboration among the major economies of the world,

including India, could help create the necessary infrastructure and

arrangements for CBDC transfer and conversion.

o Payment instruments could be made available for payment transactions

to be made via CBDC. Furthermore, universal access attributes of a

CBDC could also include an offline payment functionality.

o Instant lending to micro, small, and medium enterprises (MSMEs) in

India can be possible with the help of CBDC.

Refer: https://www.insightsonindia.com/2022/02/02/what-is-the-digital-rupee-

announced-by-sitharaman-in-budget/

Revision Through MCQs (RTM) Compilation (Feb - 2022)

Telegram: https://t.me/insightsIAStips Youtube: https://www.youtube.com/channel/UCpoccbCX9GEIwaiIe4HLjwA

14

19. With reference to the governance of ‘public sector banking in India, consider the

following statements:

1. Capital infusion into public sector banks by the Government of India has steadily

increased in the last decade.

2. To put the public sector banks in order, the merger of associate banks with the

parent State Bank of India has been affected.

Which of the statements given above is/are correct?

(a) 1 only

(b) 2 only

(c) Both 1 and 2

(d) Neither 1 nor 2

Ans: (b)

Explanation:

• S1: Capital infusion has not been steady, therefore statement 1 is wrong.

o https://www.livemint.com/Industry/3qTu3MjDuTCrPIpqKnGcrO/The-

whys-and-hows-of-public-sector-bank-recapitalisation.html

• One of the desired objectives of consolidation of banks is the streamlining of

banking operations and reduce their NPA burden. Thus, Statement 2 is

correct.

Refer: UPSC CSE 2018

20. Consider the following statements:

1. The definition of “Critical Wildlife Habitat is incorporated in the Forest Rights Act,

2006.

2. For the first time in India, Baigas have been given Habitat Rights.

3. Union Ministry of Environment, Forest and Climate Change officially decides and

declares Habitat Rights for Primitive and Vulnerable Tribal Groups in any part of

India.

Which of the statements given above is/are correct?

(a) 1 and 2 only

(b) 1, 2 and 2 only

(c) 3 only

(d) 1, 2 and 3

Ans: (a)

Explanation:

Revision Through MCQs (RTM) Compilation (Feb - 2022)

Telegram: https://t.me/insightsIAStips Youtube: https://www.youtube.com/channel/UCpoccbCX9GEIwaiIe4HLjwA

15

• The phrase ‘critical wildlife habitat’ is defined only in the Scheduled Tribes and

Other Traditional Forest Dwellers (Recognition of Forest Rights) Act, 2006, and

NOT in the Wildlife (Protection) Act, 1972.

• Union Ministry of Tribal Affairs officially decides and declares Habitat Rights for

Primitive and Vulnerable Tribal Groups in any part of India.

• Source: http://pib.nic.in/newsite/PrintRelease.aspx?relid=69806

• Source: http://www.downtoearth.org.in/news/baiga-tribals-become-india-s-

first-community-to-get-habitat-rights52452

Refer: UPSC CSE 2018

RTM- REVISION THROUGH MCQS –3rd-Feb-2022

21. Consider the following statements regarding Motion of Thanks:

1. It is addressed by the Prime Minister of India.

2. It must be passed in both the House.

Which of the statements given above is/are correct?

(a) 1 only

(b) 2 only

(c) Both 1 and 2

(d) Neither 1 nor 2

Ans: (b)

Explanation:

• The first session after each general election and the first session of every fiscal

year is addressed by the president. In this address, the president outlines the

policies and programmes of the government in the preceding year and

ensuing year. This address of the president is called the ‘Motion of Thanks’. At

the end of the discussion, the motion is put to vote.

• This motion must be passed in both the House. Otherwise, it amounts to the

defeat of the government.

Refer: https://www.insightsonindia.com/2022/02/03/motion-of-thanks-to-the-president/

22. The Justice Puttaswamy case is a landmark case of the Supreme Court of India that

mainly deals with

(a) Uniform Civil Code

(b) Reservations in Jobs

(c) Right to privacy

Revision Through MCQs (RTM) Compilation (Feb - 2022)

Telegram: https://t.me/insightsIAStips Youtube: https://www.youtube.com/channel/UCpoccbCX9GEIwaiIe4HLjwA

16

(d) Abolition of Untouchability

Ans: (c)

Explanation:

• The Justice S. Puttaswamy vs. Union of India (September 2018) case, where

the apex court unanimously recognised the fundamental right to privacy of

every individual guaranteed by the Constitution.

Refer: https://www.insightsonindia.com/2022/02/03/amendments-to-criminal-laws/

23. With reference to Lokpal in India, consider the following statements:

1. The term "Lokpal" was coined by Dr. L.M.Singhvi in 1963.

2. It was created in 1964 to address governmental corruption.

3. The source of salary for Lokpal is Consolidated Fund of India.

Which of the statements given above is/are correct?

(a) 1 and 2 only

(b) 2 and 3 only

(c) 1 and 3 only

(d) 1, 2 and 3

Ans: (c)

Explanation:

• The term "Lokpal" was coined by Dr. L.M.Singhvi in 1963.

• The term of office for Lokpal Chairman and Members is 5 years or till attaining

age of 70 years.

• The salary, allowances and other conditions of service of chairperson are

equivalent to Chief Justice of India and members is equivalent to Judge of

Supreme Court. If the person is already getting the pension (for being a former

judge), the equivalent pension amount will be deducted from the salary.

• The source of salary for Lokpal and Members is Consolidated Fund of India.

Refer: https://www.insightsonindia.com/2022/02/03/keralas-proposal-to-limit-

lokayuktas-powers/

Revision Through MCQs (RTM) Compilation (Feb - 2022)

Telegram: https://t.me/insightsIAStips Youtube: https://www.youtube.com/channel/UCpoccbCX9GEIwaiIe4HLjwA

17

24. Consider the following statements about Electoral Bonds:

1. The electoral bonds are interest-free bearer instruments used to donate money

anonymously to political parties.

2. The electoral bonds are available for purchase for 10 days in the beginning of every

quarter.

3. The bonds will be issued in multiples of Rs 500 and Rs 1000.

Which of the statements given above is/are correct?

(a) 1 and 2 only

(b) 2 and 3 only

(c) 1 and 3 only

(d) 1, 2 and 3

Ans: (a)

Explanation:

• Announced in the 2017 Union Budget, electoral bonds are interest-free bearer

instruments used to donate money anonymously to political parties. A bearer

instrument does not carry any information about the buyer or payee and the

holder of the instrument (which is the political party) is presumed to be its

owner.

• S2: The bonds will be issued in multiples of Rs 1,000, Rs 10,000, Rs 100,000

and Rs 1 crore (the range of a bond is between Rs 1,000 to Rs 1 crore).

• S3: The electoral bonds are available for purchase for 10 days in the beginning of

every quarter.

Refer: https://www.insightsonindia.com/2022/02/03/electoral-bonds-2/

25. Consider the following statements about the Indus Water Treaty:

1. It is a water-distribution treaty between India and China.

2. It was signed in Delhi on 1960.

Which of the statements given above is/are correct?

(a) 1 only

(b) 2 only

(c) Both 1 and 2

(d) Neither 1 nor 2

Ans: (d)

Explanation:

• It is a Water-Distribution Treaty, signed in Karachi on 1960, between India

(Pm Jawaharlal Nehru) and Pakistan (President Ayub Khan), brokered by the

World Bank.

Revision Through MCQs (RTM) Compilation (Feb - 2022)

Telegram: https://t.me/insightsIAStips Youtube: https://www.youtube.com/channel/UCpoccbCX9GEIwaiIe4HLjwA

18

Refer: https://www.insightsonindia.com/2022/02/03/permanent-indus-commission/

26. Which of the following is/are right bank tributaries of River Indus:

1. Shyok River

2. Zanskar River

3. Hunza River

4. Jhelum River

5. Kurram River

Select the correct answer using the code below:

(a) 1, 2, 3 and 4 only

(b) 2, 3, 4 and 5 only

(c) 1, 3 and 5 only

(d) All of the above

Ans: (c)

Explanation:

• Tributaries

o Left: Zanskar River, Suru River, Soan River, Jhelum River, Chenab

River, Ravi River, Beas River, Sutlej River, Panjnad River, Ghaggar-Hakra

River, Luni River

o Right: Shyok River, Hunza River, Gilgit River, Swat River, Kunar River,

Kabul River, Kurram River, Gomal River, Zhob River

Refer: https://www.insightsonindia.com/2022/02/03/permanent-indus-commission/

27. The term “Quantum key distribution” is used in the context of:

(a) Digital security infrastructure

(b) Secure communication method

(c) Internet of things

(d) None of the above

Ans: (b)

Explanation:

• Quantum key distribution (QKD) is a secure communication method for

exchanging encryption keys only known between shared parties. The

communication method uses properties found in quantum physics to exchange

cryptographic keys in such a way that is provable and guarantees security.

• QKD works by using photons — the particles which transmit light — to transfer

data.

• QKD allows two distant users, who do not share a long secret key initially, to

produce a common, random string of secret bits, called a secret key.

Revision Through MCQs (RTM) Compilation (Feb - 2022)

Telegram: https://t.me/insightsIAStips Youtube: https://www.youtube.com/channel/UCpoccbCX9GEIwaiIe4HLjwA

19

• Using the one-time pad encryption this key is proven to be secure to encrypt and

decrypt a message, which can then be transmitted over a standard

communication channel.

Refer: https://www.insightsonindia.com/2022/02/03/isro-demonstrates-hack-proof-

quantum-communication/

28. With reference to the Parliament of India, consider the following statements:

1. A private member’s bill is a bill presented by a Member of Parliament who is not

elected but only nominated by the President of India.

2. Recently, a private member’s bill has been passed in the Parliament of India for the

first time in its history.

Which of the statements given above is/are correct?

(a) 1 only

(b) 2 only

(c) Both 1 and 2

(d) Neither 1 nor 2

Ans: (d)

Explanation:

• Statement 1: It is introduced by any member who is not a Minister. A bill

introduced by a Minister is called as a public bill.

• Statement 2: Around 14 private member’s bills have been passed since

independence.

Refer: UPSC CSE 2017

29. From the ecological point of view, which one of the following assumes importance in

being a good link between the Eastern Ghats and the Western Ghats?

(a) Sathyamangalam Tiger Reserve

(b) Nallamala Forest

(c) Nagarhole National Park

(d) Seshachalam Biosphere Reserve

Ans: (a)

Explanation:

• Sathyamangalam forest range is a significant wildlife corridor in the Nilgiri

Biosphere Reserve between the Western Ghats and the rest of the Eastern Ghats

and a genetic link between the four other protected areas which it adjoins,

including the Billigiriranga Swamy Temple Wildlife Sanctuary, Sigur Plateau,

Mudumalai National Park and Bandipur National Park.

Revision Through MCQs (RTM) Compilation (Feb - 2022)

Telegram: https://t.me/insightsIAStips Youtube: https://www.youtube.com/channel/UCpoccbCX9GEIwaiIe4HLjwA

20

Refer: UPSC CSE 2017

30. Consider the following statements in respect of Trade Related Analysis of Fauna and

Flora in Commerce (TRAFFIC):

1. TRAFFIC is a bureau under United Nations Environment Programme (UNEP).

2. The mission of TRAFFIC is to ensure that trade in wild plants and animals is not a

threat to the conservation of nature.

Which of the above statements is/are correct?

(a) 1 only

(b) 2 only

(c) Both 1 and 2

(d) Neither 1 nor 2

Ans: (b)

Explanation:

• Statement 1: It is a NGO, not a bureau under UNEP. So, 1 is wrong.

o TRAFFIC is governed by the TRAFFIC Committee, a steering group

composed of members of TRAFFIC’s partner organizations, WWF and

IUCN.

• Statement 2: It is working globally on Wildlife trade monitoring network.

o It specializes in investigating and analysing wildlife trade trends,

patterns, impacts and drivers to provide the leading knowledge base on

trade in wild animals and plants.

Refer: UPSC CSE 2017

Revision Through MCQs (RTM) Compilation (Feb - 2022)

Telegram: https://t.me/insightsIAStips Youtube: https://www.youtube.com/channel/UCpoccbCX9GEIwaiIe4HLjwA

21

RTM- REVISION THROUGH MCQS –4th-Feb-2022

31. Which of the following Indian States share the longest border with Myanmar?

(a) Arunachal Pradesh

(b) Nagaland

(c) Mizoram

(d) Manipur

Ans: (a)

Explanation:

• Source: India and Myanmar share a 1,643 km unfenced border along Arunachal

Pradesh (520 km), Nagaland (215 km), Manipur (398 km) and Mizoram (510

km) and permit a ‘free movement’ regime upto 16 km beyond the border.

Refer: https://www.insightsonindia.com/2022/02/04/myanmar-past-and-present/

32. Article 16 of the Constitution of India is related to

(a) equal protection of the laws within the territory of India.

(b) Prohibition of discrimination on grounds of religion, race, caste, sex or place of

birth.

(c) Equality of opportunity in matters of public employment.

(d) Abolition of Untouchability.

Ans: (c)

Explanation:

• Article 16 of the Constitution of India, talks about the right of equal opportunity

in the matters of public employment.

• Refer: https://theindianconstitution.com/article-16-equality-in-public-

employment/

Refer: https://www.insightsonindia.com/2022/02/04/haryana-private-sector-quota-law-

stayed/

33. Consider the following statements:

1. The Moon is Earth's only natural satellite.

2. The Moon provides the best linkage to Earth’s early history.

3. The first physical evidence of water has been found on the Moon by ISRO’s

Chandrayaan-2 mission.

Which of the statements given above is/are correct?

(a) 1 and 2 only

Revision Through MCQs (RTM) Compilation (Feb - 2022)

Telegram: https://t.me/insightsIAStips Youtube: https://www.youtube.com/channel/UCpoccbCX9GEIwaiIe4HLjwA

22

(b) 2 and 3 only

(c) 1 and 3 only

(d) 1, 2 and 3

Ans: (a)

Explanation:

• S1: The Moon is Earth's only natural satellite.

• S2: Why are we interested in studying the moon?

o The Moon is the closest cosmic body at which space discovery can be

attempted and documented.

o It is also a promising test bed to demonstrate technologies required for

deep-space missions.

o The Moon provides the best linkage to Earth’s early history.

o It offers an undisturbed historical record of the inner Solar system

environment.

• S3: The Chandrayaan-2 mission, which was lost after it hard- landed on the

dark side of the Moon in 2019, remains active in the form of its

orbiter hovering over the Moon.

o The lander and rover malfunctioned in the final moments and crash-

landed, getting destroyed in the process.

o The primary objective of Chandrayaan 2 was to demonstrate the ability

to soft-land on the lunar surface and operate a robotic rover on the

surface.

Refer: https://www.insightsonindia.com/2022/02/04/chandrayaan-3-2/

34. With reference to India’s Drone Rules 2021, consider the following statements:

1. The aim of the Rules is to create a 'digital sky platform'

2. Nano and model drones (made for research or recreation purposes) are exempt from

type certification.

3. Type Certificate required only when a drone is to be operated in India.

Which of the statements given above is/are correct?

(a) 1 and 2 only

(b) 2 and 3 only

(c) 1 and 3 only

(d) 1, 2 and 3

Ans: (d)

Explanation:

Revision Through MCQs (RTM) Compilation (Feb - 2022)

Telegram: https://t.me/insightsIAStips Youtube: https://www.youtube.com/channel/UCpoccbCX9GEIwaiIe4HLjwA

23

• S1: The aim of the Rules is to create a 'digital sky platform' ("the Platform")

which is a business-friendly single-window online system, with minimum human

interference, where most of the permissions will be self-generated.

• Testing of drones for issuance of Type Certificate to be carried out by Quality

Council of India or authorised testing entities.

• S2: Nano and model drones (made for research or recreation purposes) are

exempt from type certification.

• S3: Type Certificate required only when a drone is to be operated in India.

Importing and manufacturing drones purely for exports are exempt from type

certification and unique identification number.

Refer: https://www.insightsonindia.com/2022/02/04/drone-rules-2021/

35. Consider the following statements about National Adaptation Fund for Climate Change

(NAFCC):

1. It is a Central Sector Scheme which was set up in the year 2015-16.

2. National Biodiversity Authority (NBA) has been designated as National Implementing

Entity (NIE) for implementation of adaptation projects under NAFCC by Govt. of

India.

Which of the statements given above is/are correct?

(a) 1 only

(b) 2 only

(c) Both 1 and 2

(d) Neither 1 nor 2

Ans: (c)

Explanation:

• The National Adaptation Fund for Climate Change (NAFCC) is a Central

Sector Scheme which was set up in the year 2015-16. The overall aim of

NAFCC is to support concrete adaptation activities which mitigate the adverse

effects of climate change. The activities under this scheme are implemented in a

project mode. The projects related to adaptation in sectors such as agriculture,

animal husbandry, water, forestry, tourism etc. are eligible for funding under

NAFCC. National Bank for Agriculture and Rural Development (NABARD) is

the National Implementing Entity (NIE).

Refer: https://www.insightsonindia.com/2022/02/04/national-adaptation-fund-for-

climate-change-nafcc/

Revision Through MCQs (RTM) Compilation (Feb - 2022)

Telegram: https://t.me/insightsIAStips Youtube: https://www.youtube.com/channel/UCpoccbCX9GEIwaiIe4HLjwA

24

36. With reference to United Nations Framework Convention on Climate Change (UNFCCC)

and India, consider the following statements:

1. India ratified the UNFCCC in 1995.

2. The nodal agency for the UNFCCC in India is the NITI Aayog.

Which of the statements given above is/are correct?

(a) 1 only

(b) 2 only

(c) Both 1 and 2

(d) Neither 1 nor 2

Ans: (d)

Explanation:

• India ratified the UNFCCC in 1993.

• The nodal agency for the UNFCCC in India is the Ministry of Environment,

Forests and Climate Change (MoEFCC).

Refer: https://www.insightsonindia.com/2022/02/04/indias-stand-at-cop-26/

37. Consider the following statements about River Cities Alliance (RCA):

1. It is envisaged as a facilitatory platform for initiating river-sensitive planning and

development.

2. It is open to all river cities of India.

Which of the statements given above is/are correct?

(a) 1 only

(b) 2 only

(c) Both 1 and 2

(d) Neither 1 nor 2

Ans: (c)

Explanation:

• The River Cities Alliance (RCA) has been launched with the objective to provide

the member cities with a platform to discuss and exchange information on

aspects that are vital for sustainable management of urban rivers, sharing best

practices and supporting innovation.

• The alliance has been launched initially with 30 cities including Dehradun,

Rishikesh, Haridwar, Srinagar, Varanasi, Kanpur, Prayagraj etc.

• The Alliance is open to all river cities of India. Any river city can join the Alliance

at any time.

Refer: Facts for Prelims: https://www.insightsonindia.com/2022/02/04/mission-2022-

insights-daily-current-affairs-pib-summary-04-february-2022-2/

Revision Through MCQs (RTM) Compilation (Feb - 2022)

Telegram: https://t.me/insightsIAStips Youtube: https://www.youtube.com/channel/UCpoccbCX9GEIwaiIe4HLjwA

25

38. The 'National Mission on Pilgrimage Rejuvenation and Spiritual Augmentation Drive

(PRASAD)' was launched by the

(a) Ministry of Culture

(b) Ministry of Tourism

(c) Ministry of Rural Development

(d) Ministry of Housing and Urban Affairs

Ans: (b)

Explanation:

• The Ministry of Tourism launched the PRASAD scheme that aims at integrated

development of pilgrimage destinations in a prioritised, planned, and sustainable

manner in order to provide a complete religious tourism experience.

Refer: Facts for Prelims: https://www.insightsonindia.com/2022/02/04/mission-2022-

insights-daily-current-affairs-pib-summary-04-february-2022-2/

39. Consider the following statements:

1. A whip is an official of a political party who acts as the party's 'enforcer' inside the

legislative assembly or house of parliament.

2. The concept of the whip was inherited from the British parliamentary system.

3. Under the Tenth Schedule of the Indian Constitution a political party has a

constitutional right to issue a whip to its legislators.

Which of the given above statements is/are correct?

(a) 1 and 2 only

(b) 2 and 3 only

(c) 1 and 3 only

(d) 1, 2 and 3

Ans: (d)

Explanation: What is a whip?

• A whip is an official of a political party who acts as the party’s ‘enforcer’

inside the legislative assembly or house of parliament.

• Parties appoint a senior member from among their House contingents to issue

whips — this member is called a Chief Whip, and he/she is assisted by

additional Whips.

• India inherited the concept of the whip from the British parliamentary

system.

• The anti-defection law (10th Schedule) provides for disqualification of a

legislator if he votes contrary to the party whip. As a result, members are

Revision Through MCQs (RTM) Compilation (Feb - 2022)

Telegram: https://t.me/insightsIAStips Youtube: https://www.youtube.com/channel/UCpoccbCX9GEIwaiIe4HLjwA

26

compelled to obey the party whip, in order to avoid losing their seat in the

House.

Refer: https://www.insightsonindia.com/2021/08/12/what-who-is-a-whip/

40. With reference to types of whips issued by the political parties in India, consider the

following statements:

1. One-line whip is issued to direct the members to be present in the House at the time

of voting.

2. Two-line whip is issued to members directing them to vote as per the party line.

Which of the given above statements is/are correct?

(a) 1 only

(b) 2 only

(c) Both 1 and 2

(d) Neither 1 nor 2

Ans: (d)

Explanation:

• There are three types of whips or instructions issued by the party

o One-line whip is issued to inform members of a party about a vote. It

allows a member to abstain in case they decide not to follow the party

line.

o Two-line whip is issued to direct the members to be present in the

House at the time of voting.

o Three-line whip is issued to members directing them to vote as per the

party line.

Refer: https://www.insightsonindia.com/2021/08/12/what-who-is-a-whip/

RTM- REVISION THROUGH MCQS –5th-Feb-2022

41. Consider the following about Panna Tiger Reserve:

1. It is situated in the Vindhya mountain range in the northern part of Madhya

Pradesh.

2. It is one of the natural habitats of Sambar deer and Sloth bear.

Which of the statements given above is/are correct?

(a) 1 only

(b) 2 only

(c) Both 1 and 2

Revision Through MCQs (RTM) Compilation (Feb - 2022)

Telegram: https://t.me/insightsIAStips Youtube: https://www.youtube.com/channel/UCpoccbCX9GEIwaiIe4HLjwA

27

(d) Neither 1 nor 2

Ans: (c)

Explanation:

• S1: The Panna tiger reserve is situated in the Vindhya mountain range in the

northern part of Madhya Pradesh.

• S2: Among the animals found here are the Bengal tiger, Indian leopard, chital,

chinkara, nilgai, Sambar deer and sloth bear, rusty-spotted cat, Asian palm

civet.

Refer: https://www.insightsonindia.com/2022/02/05/ken-betwa-link-project-2/

42. Consider the following statements about Ken River:

1. It flows through the states of Madhya Pradesh, Rajasthan and Uttar Pradesh.

2. It is a tributary of the Yamuna.

Which of the statements given above is/are correct?

(a) 1 only

(b) 2 only

(c) Both 1 and 2

(d) Neither 1 nor 2

Ans: (b)

Explanation:

• The Ken River is one of the major rivers in the Bundelkhand region of central

India and flows through the states of Madhya Pradesh and Uttar Pradesh. It is

a tributary of the Yamuna.

• Key facts:

o Ken and Betwa rivers originate in MP and are the tributaries of Yamuna.

o Ken meets with Yamuna in Banda district of UP and with Betwa in

Hamirpur district of UP.

o Rajghat, Paricha and Matatila dams are over Betwa river.

o Ken River passes through Panna tiger reserve.

Refer: https://www.insightsonindia.com/2022/02/05/ken-betwa-link-project-2/

43. Consider the following statements about the United Nations Environment Programme

(UNEP):

1. It was founded in 1972 following the landmark UN Conference on the Human

Environment.

2. It aims to help the world meet the 17 Sustainable Development Goals.

3. It is headquartered in Nairobi, Kenya.

Revision Through MCQs (RTM) Compilation (Feb - 2022)

Telegram: https://t.me/insightsIAStips Youtube: https://www.youtube.com/channel/UCpoccbCX9GEIwaiIe4HLjwA

28

Which of the statements given above is/are correct?

(a) 1 and 2 only

(b) 2 and 3 only

(c) 1 and 3 only

(d) 1, 2 and 3

Ans: (d)

Explanation:

• S1: Founded in 1972 following the landmark UN Conference on the Human

Environment.

o UNEP was conceived to monitor the state of the environment, inform

policy making with science and coordinate responses to the world’s

environmental challenges.

• S2: As a member of the United Nations Development Group, UNEP aims to help

the world meet the 17 Sustainable Development Goals.

• S3: Headquartered in Nairobi, Kenya.

Refer: https://www.insightsonindia.com/2022/02/05/unep50/

44. As the leading global environmental authority, UNEP administers, or provides

secretariat to which of the following environmental agreements (MEAs)?

1. Convention on Biological Diversity (CBD)

2. Convention on International Trade in Endangered Species of Wild Fauna and Flora

(CITES)

3. Convention on the Conservation of Migratory Species of Wild Animals (CMS)

4. Minamata Convention on Mercury

5. Vienna Convention for the Protection of the Ozone Layer

Select the correct answer using the code below:

(a) 1, 2 and 3 only

(b) 2, 3, 4 and 5 only

(c) 1, 3 and 5 only

(d) All of the above

Ans: (d)

Explanation: UNEP hosts the secretariats of many critical multilateral

environmental agreements and research bodies. These include the following:

• The Convention on Biological Diversity

• The Convention on International Trade in Endangered Species of Wild Fauna

and Flora

• The Minamata Convention on Mercury

Revision Through MCQs (RTM) Compilation (Feb - 2022)

Telegram: https://t.me/insightsIAStips Youtube: https://www.youtube.com/channel/UCpoccbCX9GEIwaiIe4HLjwA

29

• The Basel, Rotterdam and Stockholm Conventions

• The Vienna Convention for the Protection of Ozone Layer and the Montreal

Protocol

• The Convention on Migratory Species

• The Carpathian Convention

• The Bamako Convention

• The Tehran Convention

Refer: https://www.insightsonindia.com/2022/02/05/unep50/

45. Recently, ‘Xinjiang’ was in news, is an autonomous territory in:

(a) Russia

(b) Myanmar

(c) Vietnam

(d) China

Ans: (d)

Explanation:

• Xinjiang is technically an autonomous region within China — its largest

region, rich in minerals, and sharing borders with many countries, including

India, Russia and Afghanistan.

Refer: https://www.insightsonindia.com/2022/02/05/uighurs-3/

46. Consider the following statements:

1. Pink hydrogen is generated through electrolysis powered by nuclear energy.

2. Grey Hydrogen is produced from natural gas, where the emissions are captured

using carbon capture and storage.

3. Blue Hydrogen is produced from natural gas where the associated emissions are

released to the air.

Which of the given above statements is/are correct?

Revision Through MCQs (RTM) Compilation (Feb - 2022)

Telegram: https://t.me/insightsIAStips Youtube: https://www.youtube.com/channel/UCpoccbCX9GEIwaiIe4HLjwA

30

(a) 1 only

(b) 2 and 3 only

(c) 1, 2 and 3

(d) 2 only

Ans: (a)

Explanation:

• Brown hydrogen is produced using coal where the emissions are released to the

air

• Grey hydrogen is produced from natural gas where the associated emissions are

released to the air

• Blue hydrogen is produced from natural gas, where the emissions are captured

using carbon capture and storage

• Green hydrogen is produced from electrolysis powered by renewable electricity.

• Pink hydrogen is generated through electrolysis powered by nuclear energy

Refer: https://www.insightsonindia.com/2022/02/05/gail-starts-indias-maiden-project-of-

blending-hydrogen-into-natural-gas-system-in-indore-context/

47. ‘Syngas’ is a fuel gas mixture consisting primarily of:

1. Hydrogen

2. Carbon monoxide

3. Carbon dioxide

4. Methane

Select the correct answer using the code below:

(a) 1 and 2 only

(b) 1, 2 and 3 only

(c) 2 and 3 only

(d) All of the above

Ans: (d)

Explanation:

• Syngas is a mixture consisting carbon monoxide (CO), hydrogen (H2), carbon

dioxide (CO2), natural gas (CH4), and water vapour (H2O).

Refer: https://www.insightsonindia.com/2022/02/05/coal-gasification-and-liquefaction-2/

48. Consider the following statements:

1. Coal gasification is one of the more water-intensive forms of energy production.

2. Coal liquefaction is a process in which coal is converted into liquid fuels or

petrochemicals.

Revision Through MCQs (RTM) Compilation (Feb - 2022)

Telegram: https://t.me/insightsIAStips Youtube: https://www.youtube.com/channel/UCpoccbCX9GEIwaiIe4HLjwA

31

3. The liquefied coal emits twice as much CO2 as burning oil.

Which of the given above statements is/are correct?

(a) 1 and 2 only

(b) 2 and 3 only

(c) 1 and 3 only

(d) 1, 2 and 3

Ans: (d)

Explanation:

• S1: What is coal gasification?

o It is the process of producing syngas, a mixture consisting carbon

monoxide (CO), hydrogen (H2), carbon dioxide (CO2), natural gas (CH4),

and water vapour (H2O).

o During gasification, coal is blown with oxygen and steam while also being

heated under high pressure. During the reaction, oxygen and water

molecules oxidize the coal and produce syngas.

o Coal gasification is one of the more water-intensive forms of energy

production.

o There are also concerns about water contamination, land subsidence and

disposing of waste water safely.

• S2 & S3: What is coal liquefaction?

o Also called Coal to Liquid (CTL) technology, it is an alternative route

to produce diesel and gasoline and makes economic sense only in a

world of high crude oil prices.

o The process involves gasification of coal, which in turn will produce

synthetic gas (a mix of CO+H2). The synthetic gas can be liquefied to its

fuel equivalent in presence of cobalt/iron-based catalysts at higher

pressure and temperature.

o However, liquefied coal emits twice as much CO2 as burning oil. It

also emits a large volume of SO2

Refer: https://www.insightsonindia.com/2022/02/05/coal-gasification-and-liquefaction-2/

49. Consider the following statements about Ramsar Convention.

1. It is the United Nations Convention that provides the framework for the conservation

and wise use of wetlands.

2. Almost 90% of UN member states, have acceded to become “Contracting Parties”.

3. It does not consider underground aquifers and human-made sites as wetlands.

4. Montreux Record is maintained as part of the Ramsar List.

Which of the above statements is/are incorrect?

Revision Through MCQs (RTM) Compilation (Feb - 2022)

Telegram: https://t.me/insightsIAStips Youtube: https://www.youtube.com/channel/UCpoccbCX9GEIwaiIe4HLjwA

32

(a) 1 and 3 only

(b) 2 and 4 only

(c) 1, 2 and 3 only

(d) 2, 3 and 4 only

Ans: (a)

Explanation: here the directive word is incorrect!!

• The Convention on Wetlands, called the Ramsar Convention, is the

intergovernmental treaty that provides the framework for the conservation and

wise use of wetlands and their resources.

• The Convention was adopted in the Iranian city of Ramsar in 1971 and came

into force in 1975. Since then, almost 90% of UN member states, from all the

world’s geographic regions, have acceded to become “Contracting Parties”.

• The Convention uses a broad definition of wetlands. It includes all lakes and

rivers, underground aquifers, swamps and marshes, wet grasslands, peatlands,

oases, estuaries, deltas and tidal flats, mangroves and other coastal areas, coral

reefs, and all human-made sites such as fish ponds, rice paddies, reservoirs and

salt pans.

• Montreux Record under the Convention is a register of wetland sites on the List

of Wetlands of International Importance where changes in ecological character

have occurred, are occurring, or are likely to occur as a result of technological

developments, pollution or other human interference.

• It is maintained as part of the Ramsar List.

Refer: https://www.insightsonindia.com/2022/02/05/world-wetlands-day-february-2/

50. Consider the following statements about Chilika lake:

1. It is one of the largest fresh water lakes in Asia.

2. It is famous for the phumdis floating over it.

3. It was the first Indian site to be placed on Montreux Record.

Which of the given above statements is/are correct?

(a) 1 and 2 only

(b) 3 only

(c) 2 and 3 only

(d) 1 and 3 only

Ans: (b)

Explanation: S3: It was the first Indian site to be placed on Montreux Record in 1993. In

2002, Chilika was taken out of the Montreux Record, in light of the improved conditions

of the lake.

Revision Through MCQs (RTM) Compilation (Feb - 2022)

Telegram: https://t.me/insightsIAStips Youtube: https://www.youtube.com/channel/UCpoccbCX9GEIwaiIe4HLjwA

33

• S1: The Chilika lake in Odisha, Asia’s largest brackish water lake, was once part

of the Bay of Bengal.

• S2: Loktak lake is famous for the phumdis (heterogeneous mass of vegetation,

soil and organic matter at various stages of decomposition) floating over it.

Refer: https://www.insightsonindia.com/2022/02/05/world-wetlands-day-february-2/

Revision Through MCQs (RTM) Compilation (Feb - 2022)

Telegram: https://t.me/insightsIAStips Youtube: https://www.youtube.com/channel/UCpoccbCX9GEIwaiIe4HLjwA

34

RTM- REVISION THROUGH MCQS –7th-Feb-2022

51. Which of the following event led to the termination of Non-Cooperation Movement by

Mahatma Gandhi?

(a) Partition of Bengal

(b) Passing of Rowlatt Act

(c) Jallianwala Bagh Massacre

(d) Chauri - Chaura Incident

Ans: (d)

Explanation: What is Chauri Chaura incident?

• The incident (4th February, 1922) occurred at Chauri Chaura in the Gorakhpur

district of the United Province, (modern Uttar Pradesh) in British India.

• During this incident, a large group of protesters, participating in the Non-

cooperation movement, clashed with police, who opened fire.

• The demonstrators attacked and set fire to a police station in retaliation, killing

all of its occupants.

• In response to this, Mahatma Gandhi, who was strictly against violence,

halted the Non-cooperation Movement on the national level on 12 February

1922, as a direct result of this incident.

Refer: https://www.insightsonindia.com/2022/02/07/100-years-for-chauri-chaura-

incident/

52. Which of the following were the causes behind the launch of ‘Non-cooperation

movement’?

1. Khilafat wrong

2. “Punjab wrongs” of 1919

3. Resentment with Rowlatt Act

Select the correct answer code:

(a) 1 and 2 only

(b) 2 and 3 only

(c) 1 and 3 only

(d) 1, 2 and 3

Ans: (d)

Explanation:

• In 1919 Gandhiji gave a call for a satyagraha against the Rowlatt Act that the

British had just passed. The Act curbed fundamental rights such as the freedom

of expression and strengthened police powers.

Revision Through MCQs (RTM) Compilation (Feb - 2022)

Telegram: https://t.me/insightsIAStips Youtube: https://www.youtube.com/channel/UCpoccbCX9GEIwaiIe4HLjwA

35

• In April 1919 there were a number of demonstrations and hartals in the country

and the government used brutal measures to suppress them. The Jallianwala

Bagh atrocities, inflicted by General Dyer in Amritsar on Baisakhi day were a

part of this repression.

• The Khilafat issue was another such cause. In 1920 the British imposed a

harsh treaty on the Turkish Sultan or Khalifa. People were furious about this as

they had been about the Jallianwala massacre. Also, Indian Muslims were keen

that the Khalifa be allowed to retain control over Muslim sacred places in the

erstwhile Ottoman Empire.

• The leaders of the Khilafat agitation, Mohammad Ali and Shaukat Ali, wished to

initiate a full-fledged Non-Cooperation Movement. Gandhiji supported their call

and urged the Congress to campaign against “Punjab wrongs” (Jallianwala

massacre), the Khilafat wrong and demand swaraj

Refer: https://www.insightsonindia.com/2022/02/07/100-years-for-chauri-chaura-

incident/

53. Consider the following statements:

1. The power of Judicial Review comes from the Constitution of India itself.

2. The power of judicial review is evoked to protect and enforce the fundamental rights

guaranteed in Part III of the Constitution.

Which of the statements given above is/are correct?

(a) 1 only

(b) 2 only

(c) Both 1 and 2

(d) Neither 1 nor 2

Ans: (c)

Explanation: What is Judicial Review?

• Judicial review is the power of Judiciary to review any act or order of Legislative

and Executive wings and to pronounce upon the constitutional validity when

challenged by the affected person.

• Judicial review present in India:

o The power of Judicial Review comes from the Constitution of India itself

(Articles 13, 32, 136, 142 and 147 of the Constitution).

o The power of judicial review is evoked to protect and enforce the

fundamental rights guaranteed in Part III of the Constitution.

o Article 13 of the Constitution prohibits the Parliament and the state

legislatures from making laws that “may take away or abridge the

fundamental rights” guaranteed to the citizens of the country.

Revision Through MCQs (RTM) Compilation (Feb - 2022)

Telegram: https://t.me/insightsIAStips Youtube: https://www.youtube.com/channel/UCpoccbCX9GEIwaiIe4HLjwA

36

o The provisions of Article 13 ensure the protection of the fundamental

rights and consider any law “inconsistent with or in derogation of the

fundamental rights” as void.

Refer: https://www.insightsonindia.com/2022/02/07/sc-to-weigh-between-national-

security-judicial-scrutiny/

54. Consider the following statements:

1. Under Article 82 of the Constitution of India, the Parliament enacts a Delimitation

Act after every Census.

2. The Delimitation Commission is appointed by the Prime Minister of India.

Which of the statements given above is/are correct?

(a) 1 only

(b) 2 only

(c) Both 1 and 2

(d) Neither 1 nor 2

Ans: (a)

Explanation:

• What is Delimitation?

o Delimitation literally means the process of fixing limits or boundaries of

territorial constituencies in a state that has a legislative body.

• Who carries out the exercise?

o Delimitation is undertaken by a highly powerful commission. They are

formally known as Delimitation Commission or Boundary

Commission.

o These bodies are so powerful that its orders have the force of law and

they cannot be challenged before any court.

• Composition of the Commission:

o According to the Delimitation Commission Act, 2002, the Delimitation

Commission will have three members: a serving or retired judge of the

Supreme Court as the chairperson, and the Chief Election Commissioner

or Election Commissioner nominated by the CEC and the State Election

Commissioner as ex-officio members.

• Constitutional Provisions:

o Under Article 82, the Parliament enacts a Delimitation Act after every

Census.

o Under Article 170, States also get divided into territorial constituencies

as per Delimitation Act after every Census.

Revision Through MCQs (RTM) Compilation (Feb - 2022)

Telegram: https://t.me/insightsIAStips Youtube: https://www.youtube.com/channel/UCpoccbCX9GEIwaiIe4HLjwA

37

Refer: https://www.insightsonindia.com/2022/02/07/delimitation-panel-2/

55. Consider the following statements about Pradhan Mantri Matru Vandana Yojana:

1. It was launched in 2021 as a flagship maternity benefits programme of the

Government of India.

2. It is a centrally sponsored scheme being executed by the Ministry of Women and

Child Development.

Which of the statements given above is/are correct?

(a) 1 only

(b) 2 only

(c) Both 1 and 2

(d) Neither 1 nor 2

Ans: (b)

Explanation:

• It is a Maternity Benefit Programme that is implemented in all the districts of the

country in accordance with the provision of the National Food Security Act,

2013.

• The direct benefit cash transfer is to help expectant mothers meet enhanced

nutritional requirements as well as to partially compensate them for wage loss

during their pregnancy.

• The scheme was announced on December 31, 2016. It is being implemented in

all districts of the country with effect from 1st January, 2017.

Refer: https://www.insightsonindia.com/2022/02/07/pradhan-mantri-matru-vandana-

yojana-4/

56. Consider the following statements:

1. A green bond is a fixed-income instrument designed specifically to support specific

climate-related or environmental projects.

2. The Climate Action Network is the largest green bond issuer in 2021.

Which of the statements given above is/are correct?

(a) 1 only

(b) 2 only

(c) Both 1 and 2

(d) Neither 1 nor 2

Ans: (a)

Explanation: What Is a Green Bond?

• A green bond is a type of fixed-income instrument that is specifically earmarked

to raise money for climate and environmental projects.

Revision Through MCQs (RTM) Compilation (Feb - 2022)

Telegram: https://t.me/insightsIAStips Youtube: https://www.youtube.com/channel/UCpoccbCX9GEIwaiIe4HLjwA

38

• These bonds are typically asset-linked and backed by the issuing entity’s

balance sheet, so they usually carry the same credit rating as their issuers’ other

debt obligations.

• Green bonds may come with tax incentives to enhance their attractiveness to

investors.

• The World Bank is a major issuer of green bonds. It has issued 164 such

bonds since 2008, worth a combined $14.4 billion. In 2020, the total issuance of

green bonds was worth almost $270 billion, according to the Climate Bond

Initiative.

Refer: https://www.insightsonindia.com/2022/02/07/green-bonds-4/

57. Which of the following recently declared as India’s first OECM (other effective area-

based conservation measures) site?

(a) Vanaparvam Biodiversity Park

(b) Aravalli Biodiversity Park

(c) Panchavati Biological Park

(d) Yamuna Biodiversity Park

Ans: (b)

Explanation:

• On World Wetlands Day, that is, on February 2, the Aravalli Biodiversity Park

(Gurugram) was announced as the first Other Effective Area – based

Conservation Measures site, OECM site.

• The OECM tag is provided by the International Union for Conservation of Nature,

IUCN. According to IUCN, the OECM sites are not protected but are rich in

biodiversity.

Refer: facts for prelims: https://www.insightsonindia.com/2022/02/07/mission-2022-

insights-daily-current-affairs-pib-summary-07-february-2022/

58. With reference to OECM (other effective area-based conservation measures) site,

consider the following statements:

1. The OECM tag is provided by the Conservation International.

2. The OECM areas were defined at the Convention on Biological Diversity that was

held in 2018.

Which of the statements given above is/are correct?

(a) 1 only

(b) 2 only

(c) Both 1 and 2

Revision Through MCQs (RTM) Compilation (Feb - 2022)

Telegram: https://t.me/insightsIAStips Youtube: https://www.youtube.com/channel/UCpoccbCX9GEIwaiIe4HLjwA

39

(d) Neither 1 nor 2

Ans: (b)

Explanation:

• The OECM tag is provided by the International Union for Conservation of

Nature, IUCN. According to IUCN, the OECM sites are not protected but are rich

in biodiversity.

• The OECM areas were defined at the Convention on Biological Diversity that

was held in 2018.

• The OECM tag does not bring any legal, financial or management implications,

but designates the area as a biodiversity hotspot on the international map.

• ‘Other effective area-based conservation measures’ (OECMS) is a conservation

designation for areas that are achieving the effective in-situ conservation of

biodiversity outside of protected areas.

Refer: facts for prelims: https://www.insightsonindia.com/2022/02/07/mission-2022-

insights-daily-current-affairs-pib-summary-07-february-2022/

59. Consider the following statements about the Istanbul Convention:

1. The treaty is the world’s first binding instrument to prevent and tackle violence

against women.

2. The convention was adopted by the Council of Europe Committee of Ministers in

2021.

Which of the statements given above is/are correct?

(a) 1 only

(b) 2 only

(c) Both 1 and 2

(d) Neither 1 nor 2

Ans: (a)

Explanation:

• It is also called as the Council of Europe Convention on preventing and

combating violence against women and domestic violence.

• The treaty is the world’s first binding instrument to prevent and tackle

violence against women.

• It is the most comprehensive legal framework that exists to tackle violence

against women and girls, covering domestic violence, rape, sexual assault,

female genital mutilation (FGM), so-called honour-based violence, and forced

marriage.

Revision Through MCQs (RTM) Compilation (Feb - 2022)

Telegram: https://t.me/insightsIAStips Youtube: https://www.youtube.com/channel/UCpoccbCX9GEIwaiIe4HLjwA

40

• When a government ratifies the Convention, they are legally bound to follow

it.

o The convention was adopted by the Council of Europe Committee of

Ministers on 7 April 2011.

o The Convention sets minimum standards for governments to meet when

tackling violence against women.

Refer: https://www.insightsonindia.com/2021/11/22/istanbul-convention-on-violence-

against-women-2/

60. Consider the following statements about the Istanbul Convention:

1. The treaty is the world’s first binding instrument to prevent and tackle violence

against women.

2. The convention was adopted by the Council of Europe Committee of Ministers in

2021.

Which of the statements given above is/are correct?

(a) 1 only

(b) 2 only

(c) Both 1 and 2

(d) Neither 1 nor 2

Ans: (a)

Explanation:

• It is also called as the Council of Europe Convention on preventing and

combating violence against women and domestic violence.

• The treaty is the world’s first binding instrument to prevent and tackle

violence against women.

• It is the most comprehensive legal framework that exists to tackle violence

against women and girls, covering domestic violence, rape, sexual assault,

female genital mutilation (FGM), so-called honour-based violence, and forced

marriage.

• When a government ratifies the Convention, they are legally bound to follow

it.

o The convention was adopted by the Council of Europe Committee of

Ministers on 7 April 2011.

o The Convention sets minimum standards for governments to meet when

tackling violence against women.

Refer: https://www.insightsonindia.com/2021/11/22/istanbul-convention-on-violence-

against-women-2/

Revision Through MCQs (RTM) Compilation (Feb - 2022)

Telegram: https://t.me/insightsIAStips Youtube: https://www.youtube.com/channel/UCpoccbCX9GEIwaiIe4HLjwA

41

RTM- REVISION THROUGH MCQS –8th-Feb-2022 61. Consider the following statements:

1. Article 25 of the Constitution guarantees freedom of religion to all persons in India.

2. Like all fundamental rights, the state can restrict the right for grounds of public

order, decency, morality, health and other state interests.

Which of the statements given above is/are correct?

(a) 1 only

(b) 2 only

(c) Both 1 and 2

(d) Neither 1 nor 2

Ans: (c)

Explanation:

• Article 25(1) of the Constitution guarantees the freedom of conscience and the

right freely to profess, practise and propagate religion. It is a right that

guarantees negative liberty — which means that the state shall ensure that there

is no interference or obstacle to exercising this freedom.

• Limitations: Like all fundamental rights, the state can restrict the right for

grounds of public order, decency, morality, health and other state interests.

Refer: https://www.insightsonindia.com/2022/02/08/freedom-of-religion-and-attire/

62. The doctrine of “essentiality” was invented by the Supreme Court in which of the

following case?

(a) Shirur Mutt Case

(b) Amna Bint Basheer Case

(c) Fathima Tasneem Case

(d) Kesavananda Bharti Case

Ans: (a)

Explanation:

• Shirur Mutt case in 1954: The doctrine of “essentiality” was invented by the

Supreme Court. The court held that the term “religion” will cover all rituals and

practices “integral” to a religion, and took upon itself the responsibility of

determining the essential and non-essential practices of a religion.

Refer: https://www.insightsonindia.com/2022/02/08/freedom-of-religion-and-attire/

63. Consider the following statements.

1. The Lieutenant Governor of a Union Territory is empowered to promulgate

ordinances only when the assembly is dissolved.

Revision Through MCQs (RTM) Compilation (Feb - 2022)

Telegram: https://t.me/insightsIAStips Youtube: https://www.youtube.com/channel/UCpoccbCX9GEIwaiIe4HLjwA

42

2. The Governor of a state can issue ordinances only with the approval of the

President.

3. The President’s ordinance making power is not a discretionary power.

Which of the above statements is/are incorrect?

(a) 2 only

(b) 3 only

(c) 2 and 3 only

(d) 1, 2 and 3

Ans: (c)

Explanation:

• S1: Such an ordinance has the same force as an act of the assembly. Every such

ordinance must be approved by the assembly within six weeks from its

reassembly.

o He can also withdraw an ordinance at any time.

o But, he cannot promulgate an ordinance when the assembly is

dissolved or suspended. Further, no such ordinance can be

promulgated or withdrawn without the prior permission of the

President.

• S2: The governor of a state can also issue ordinances under Article 213 of the

Constitution of India, when the state legislative assembly is not in session.

• S3: The President can send back the advice given by the Council of Ministers

and ask the Council to reconsider the decision. In doing this, the President acts

on his (or her) own discretion. When the President thinks that the advice has

certain flaws or legal lacunae, or that it is not in the best interests of the

country, the President can ask the Council to reconsider the decision. Although,

the Council can still send back the same advice and the President would then be

bound by that advice, such a request by the President to reconsider the decision,

would naturally carry a lot of weight. So, this is one way in which the president

can act in his own discretion.

Refer: https://www.insightsonindia.com/2022/02/08/kerala-governor-signs-lok-ayukta-

ordinance/

64. Consider the following statements:

1. Article 143 of the Constitution grants the President certain law-making powers to

promulgate ordinances during the recess of Parliament.

2. These ordinances have the same force and effect as an Act of Parliament but are in

the nature of temporary laws.

Which of the statements given above is/are correct?

Revision Through MCQs (RTM) Compilation (Feb - 2022)

Telegram: https://t.me/insightsIAStips Youtube: https://www.youtube.com/channel/UCpoccbCX9GEIwaiIe4HLjwA

43

(a) 1 only

(b) 2 only

(c) Both 1 and 2

(d) Neither 1 nor 2

Ans: (b)

Explanation:

• The ordinance making power is the most important legislative power of the

President and the Governor. It has been vested in them to deal with unforeseen

or urgent situations.

• Article 123 of the Constitution grants the President certain law-making

powers to promulgate ordinances during the recess of Parliament.

• These ordinances have the same force and effect as an Act of Parliament

but are in the nature of temporary laws.

• Likewise, the Governor of a state can issue ordinances under Article 213 of the

Constitution, when the state legislative assembly (or either of the two Houses in

states with bicameral legislatures) is not in session.

• The Constitution permits the central and State governments to make laws when

Parliament (or the State Legislature) is not in session.

Refer: https://www.insightsonindia.com/2022/02/08/kerala-governor-signs-lok-ayukta-

ordinance/

65. Consider the following statements about the Iran Nuclear Deal:

1. It was the result of prolonged negotiations between Iran and Australia.

2. Under the deal, Iran agreed to significantly cut its stores of centrifuges, enriched

uranium and heavy-water.

Which of the statements given above is/are correct?

(a) 1 only

(b) 2 only

(c) Both 1 and 2

(d) Neither 1 nor 2

Ans: (b)

Explanation:

• Also known as the Joint Comprehensive Plan of Action (JCPOA).

• The JCPOA was the result of prolonged negotiations from 2013 and 2015

between Iran and P5+1 (China, France, Germany, Russia, the United

Kingdom, the United States, and the European Union, or the EU).

Revision Through MCQs (RTM) Compilation (Feb - 2022)

Telegram: https://t.me/insightsIAStips Youtube: https://www.youtube.com/channel/UCpoccbCX9GEIwaiIe4HLjwA

44

• Under the deal, Tehran agreed to significantly cut its stores of centrifuges,

enriched uranium and heavy-water, all key components for nuclear weapons.

Refer: https://www.insightsonindia.com/2022/02/08/iran-nuclear-deal-5/

66. Consider the following statements about Operation AAHT:

1. It is a nationwide operation to curb human trafficking.

2. It is launched by the Narcotics Control Bureau.

Which of the statements given above is/are correct?

(a) 1 only

(b) 2 only

(c) Both 1 and 2

(d) Neither 1 nor 2

Ans: (a)

Explanation:

• It is a nationwide operation to curb human trafficking.

• Launched by the Railway Protection Force.

• As part of “Operation AAHT”, special teams will be deployed on all long-distance

trains/routes with focus on rescuing victims, particularly women and children,

from the clutches of traffickers.

Refer: facts for prelims: https://www.insightsonindia.com/2022/02/08/mission-2022-

insights-daily-current-affairs-pib-summary-08-february-2022/

67. Consider the following statements:

1. The reserve is located in Telangana.

2. This sanctuary is catchment for the rivers Godavari and Kadam.

3. The sanctuary is one of the richest teak forests in the state.

The above given statements refers to which of the following protected site?

(a) Sathyamangalam Tiger Reserve

(b) Kawal Tiger Reserve

(c) Bandipur Tiger Reserve

(d) Kanha Tiger Reserve

Ans: (b)

Explanation:

• The Kawal Tiger Reserve is going to host its first ever ‘Bird Walk’ on February 12

and 13.

• Kawal is home to a rich diversity in flora and fauna with more than 300 species

of birds, and over 600 tree species with different forest compositions.

Revision Through MCQs (RTM) Compilation (Feb - 2022)

Telegram: https://t.me/insightsIAStips Youtube: https://www.youtube.com/channel/UCpoccbCX9GEIwaiIe4HLjwA

45

• The reserve is located in Telangana.

• The reserve is the oldest sanctuary in the northern Telangana region of the state.

• This sanctuary is catchment for the rivers Godavari and Kadam.

• The sanctuary is one of the richest teak forests in the state, with dense pristine

areas free of human disturbance.

Refer: Facts for Prelims: https://www.insightsonindia.com/2022/02/08/mission-2022-

insights-daily-current-affairs-pib-summary-08-february-2022/

68. Consider the following statements:

1. The president of India can summon a session of parliament at such a place as

he/she thinks fit.

2. The constitution of India provides for three sessions of the parliament in a year but

it is not mandatory to conduct all the sessions.

3. There is no minimum number of days that the parliament is required to meet in a

year

Which of the above given statements is/are correct?

(a) 1 only

(b) 2 only

(c) 1 and 3 only

(d) 2 and 3 only

Ans: (c)

Explanation:

• S1 and S2: Article 85(1) of the Constitution empowers the President to summon

each House of Parliament to meet at such time and place as he thinks fit, but six

months shall not intervene between its last sitting in one Session and the date

appointed for its first sitting in the next Session.

• S3: This is correct, refer to the explanation above.

o http://legislative.gov.in/sites/default/files/coi-4March2016.pdf

Refer: https://www.insightsonindia.com/2021/11/09/sessions-of-parliament/

69. Consider the following statements:

1. Cabinet Committee on Parliament Affairs (CCPA) looks after the progress of

government business in the Parliament.

2. It is headed by Union Home Minister.

Which of the statements given above is/are correct?

(a) 1 only

(b) 2 only

Revision Through MCQs (RTM) Compilation (Feb - 2022)

Telegram: https://t.me/insightsIAStips Youtube: https://www.youtube.com/channel/UCpoccbCX9GEIwaiIe4HLjwA

46

(c) Both 1 and 2

(d) Neither 1 nor 2

Ans: (a)

Explanation:

• Cabinet Committee on Parliamentary Affairs (CCPA): It looks into the

matters related to the progress of government business in the Parliament of

India. It chaired by the Defense Minister of India.

o All committees of the cabinet except that of Parliamentary Affairs and

Accommodation are chaired by the Prime Minister. Cabinet Committee on

Accommodation is headed by Home Minister Amit Shah and Cabinet

committee on Parliamentary Affairs is headed by Rajnath Singh.

• Link: https://www.thehindubusinessline.com/news/national/modi-

reconstitutes-cabinet-panels/article35305951.ece

Refer: https://www.insightsonindia.com/2021/11/09/sessions-of-parliament/

70. Consider the following statements:

1. The president of India can summon a session of parliament at such a place as

he/she thinks fit.

2. The constitution of India provides for three sessions of the parliament in a year but

it is not mandatory to conduct all the sessions.

3. There is no minimum number of days that the parliament is required to meet in a

year

Which of the above given statements is/are correct?

(a) 1 only

(b) 2 only

(c) 1 and 3 only

(d) 2 and 3 only

Ans: (c)

Explanation:

• S1 and S2: Article 85(1) of the Constitution empowers the President to summon

each House of Parliament to meet at such time and place as he thinks fit, but six

months shall not intervene between its last sitting in one Session and the date

appointed for its first sitting in the next Session.

• S3: This is correct, refer to the explanation above.

o http://legislative.gov.in/sites/default/files/coi-4March2016.pdf

Refer: https://www.insightsonindia.com/2021/11/09/sessions-of-parliament/

Revision Through MCQs (RTM) Compilation (Feb - 2022)

Telegram: https://t.me/insightsIAStips Youtube: https://www.youtube.com/channel/UCpoccbCX9GEIwaiIe4HLjwA

47

RTM- REVISION THROUGH MCQS –9h-Feb-2022 71. He was extensively involved in the plans for the Ghadar conspiracy, and went

underground after it was exposed in February 1915. He was a close associate of Rash

Behari Bose. After Bose escaped to Japan, he was considered the most senior leader of

India’s revolutionary movement. He was

(a) Chandrashekhar Azad

(b) Madan Lal Dhingra

(c) Bhagat Singh

(d) Sachindra Nath Sanyal

Ans: (d)

Explanation:

• He was the founder of the Hindustan Republican Association. HRA was

created to carry out armed resistance against the British Empire in India.

• Sanyal founded a branch of the Anushilan Samiti in Patna in 1913.

• In 1912 Delhi Conspiracy Trial, Sanyal with Rashbehari Bose attacked the

then Viceroy Hardinge.

• He was extensively involved in the plans for the Ghadar conspiracy, and went

underground after it was exposed in February 1915. He was a close associate of

Rash Behari Bose.

• After Bose escaped to Japan, Sanyal was considered the most senior leader of

India’s revolutionary movement.

• He was a mentor for revolutionaries like Chandrashekhar Azad and Bhagat

Singh.

• Sanyal and Mahatma Gandhi engaged in a famous debate published in Young

India between 1920 and 1924. Sanyal argued against Gandhi’s gradualist

approach.

• Sanyal was sentenced to life for his involvement in the Kakori conspiracy and

was imprisoned at Cellular Jail in the Andaman and Nicobar Islands, where he

wrote his book titled Bandi Jeevan (A Life of Captivity, 1922).

Refer: https://www.insightsonindia.com/2022/02/09/sachindra-nath-sanyal-1893-1942/

72. Who among the following leaders was/were associated with the Ghadar Party?

1. Lala Hardayal

2. Kartar Singh Saraba

3. Surya Sen

4. Bhai Parmanand

Select the correct answer using the code below:

(a) 1 and 2 only

Revision Through MCQs (RTM) Compilation (Feb - 2022)

Telegram: https://t.me/insightsIAStips Youtube: https://www.youtube.com/channel/UCpoccbCX9GEIwaiIe4HLjwA

48

(b) 1, 2 and 4 only

(c) 3 and 4 only

(d) 1, 2, 3 and 4

Ans: (b)

Explanation:

• The Ghadar Party was a revolutionary group organized around a weekly

newspaper ‘The Ghadar’ with its headquarters at San Francisco and branches

along the US coast and in the Far East.

• The Ghadar programme was to organize assassinations of officials, publish

revolutionary and anti-imperialist literature, and work among Indian troops

stationed abroad, procure arms and bring about a simultaneous revolt in all

British colonies.

• The moving spirits behind the Ghadar Party were Lala Hardayal, Bhagwan

Singh Gyanee, Kartar Singh Saraba, Bark Bhai Parmanand.

• Surya Sen was a Bengali Indian revolutionary who was influential in the

Indian independence movement against British rule in India and is best known

for leading the 1930 Chittagong armoury raid.

Refer: https://www.insightsonindia.com/2022/02/09/sachindra-nath-sanyal-1893-1942/

73. Consider the following statements about PM-CARES Fund :

1. It was set up as a public charitable trust.

2. It can avail donations from the foreign contribution.

3. Prime Minister is the ex-officio Chairman of the PM CARES Fund.

Which of the statements given above is/are correct?

(a) 1 and 2 only

(b) 2 and 3 only

(c) 1 and 3 only

(d) 1, 2 and 3

Ans: (d)

Explanation:

• PM-CARES was set up as a public charitable trust with the trust deed registered

on March 27, 2020.

• It can avail donations from the foreign contribution and donations to fund can

also avail 100% tax exemption.

• PM-CARES is different from the Prime Minister’s National Relief Fund (PMNRF).

• Who administers the fund?

Revision Through MCQs (RTM) Compilation (Feb - 2022)

Telegram: https://t.me/insightsIAStips Youtube: https://www.youtube.com/channel/UCpoccbCX9GEIwaiIe4HLjwA

49

o Prime Minister is the ex-officio Chairman of the PM CARES Fund and

Minister of Defence, Minister of Home Affairs and Minister of Finance,

Government of India are ex-officio Trustees of the Fund.

Refer: https://www.insightsonindia.com/2022/02/09/pm-cares-3/

74. The quadrilateral security dialogue includes

1. Japan

2. United States

3. Australia

Select the correct answer using the code below:

(a) 1 and 3 only

(b) 2 and 3 only

(c) 1 and 2 only

(d) 1, 2 and 3

Ans: (d)

Explanation:

• The quadrilateral security dialogue includes Japan, India, United States and

Australia.

• All four nations find a common ground of being the democratic nations and

common interests of unhindered maritime trade and security.

Refer: https://www.insightsonindia.com/2022/02/09/what-is-quad-2/

75. Consider the following statements:

1. The TRIPS agreement was negotiated in 1995 at the World Bank.

2. It guarantees minimum standards of IP protection.

Which of the statements given above is/are correct?

(a) 1 only

(b) 2 only

(c) Both 1 and 2

(d) Neither 1 nor 2

Ans: (b)

Explanation:

• The TRIPS agreement was negotiated in 1995 at the WTO, it requires all its

signatory countries to enact domestic law.

• It guarantees minimum standards of IP protection. Such legal consistency

enables innovators to monetise their intellectual property in multiple countries.

Revision Through MCQs (RTM) Compilation (Feb - 2022)

Telegram: https://t.me/insightsIAStips Youtube: https://www.youtube.com/channel/UCpoccbCX9GEIwaiIe4HLjwA

50

• In 2001, the WTO signed the Doha Declaration, which clarified that in a public

health emergency, governments could compel companies to license their patents

to manufacturers, even if they did not think the offered price was acceptable.

• This provision, commonly referred to as “compulsory licensing”, was already

built into the TRIPS Agreement and the Doha declaration only clarified its usage.

Refer: https://www.insightsonindia.com/2022/02/09/indias-patent-waiver-plan-at-wto-

and-issues-associated/

76. Consider the following statements about Parvatmala Scheme:

1. It was announced in the Union Budget for 2022-23 to improve connectivity in hilly

areas.

2. The scheme is being presently started in regions like Uttarakhand and Himachal

Pradesh.

Which of the statements given above is/are correct?

(a) 1 only

(b) 2 only

(c) Both 1 and 2

(d) Neither 1 nor 2

Ans: (c)

Explanation:

• The National Ropeways Development Programme – “Parvatmala” was

announced recently by the Union Finance Minister in the Union Budget for

2022-23 to improve connectivity in hilly areas.

• About the scheme:

o This will be a preferred ecologically sustainable alternative in place of

conventional roads in difficult hilly areas.

o The idea is to improve connectivity and convenience for commuters,

besides promoting tourism.

o This may also cover congested urban areas, where conventional mass

transit systems are not feasible.

• Implementation:

o The scheme is being presently started in regions like Uttarakhand,

Himachal Pradesh, Manipur, Jammu & Kashmir and the other North

Eastern states.

Refer: https://www.insightsonindia.com/2022/02/09/parvatmala-scheme/

Revision Through MCQs (RTM) Compilation (Feb - 2022)

Telegram: https://t.me/insightsIAStips Youtube: https://www.youtube.com/channel/UCpoccbCX9GEIwaiIe4HLjwA

51

77. Consider the following statements about the Revamped Distribution Sector Scheme for

better operations & financial sustainability of all DISCOMs:

1. It is a reforms-based and results-linked scheme.

2. It seeks to improve the operational efficiencies and financial sustainability of all

DISCOMs/Power Departments including Private Sector DISCOMs.

Which of the statements given above is/are correct?

(a) 1 only

(b) 2 only

(c) Both 1 and 2

(d) Neither 1 nor 2

Ans: (a)

Explanation:

• S1: It is a reforms-based and results-linked scheme.

• S2: It seeks to improve the operational efficiencies and financial

sustainability of all DISCOMs/Power Departments excluding Private Sector

DISCOMs.

Refer: https://www.insightsonindia.com/2022/02/09/powerthon-2022/

78. Consider the following statements about Wayanad Wildlife Sanctuary:

1. It is part of the Nilgiri Biosphere Reserve.

2. It is the only sanctuary of Kerala where sightings of four-horned antelopes have

been reported.

Which of the statements given above is/are correct?

(a) 1 only

(b) 2 only

(c) Both 1 and 2

(d) Neither 1 nor 2

Ans: (c)

Explanation:

• The sanctuary is a component of the Nilgiri Biosphere Reserve (5,520 sq km)

and is a vital component of the Elephant Reserve No. 7 of South India.

• It is the only sanctuary of Kerala where sightings of four-horned

antelopes have been reported.

• Presence of Egyptian vulture, Himalayan griffon, and Cinereous vultures are

also reported in the sanctuary and the two species of vultures, red-headed

and white-backed vultures, once common in Kerala, are now restricted to the

Wayanad plateau.

Revision Through MCQs (RTM) Compilation (Feb - 2022)

Telegram: https://t.me/insightsIAStips Youtube: https://www.youtube.com/channel/UCpoccbCX9GEIwaiIe4HLjwA

52

• The Nagarhole-Bandipur-Mudumalai-Wayanad forest complex is also one of

the most important tiger habitats in the country.

• The forests of the wildlife division form the major catchments for the tributaries

of the Kabini river system.

Refer: Facts for Prelims: https://www.insightsonindia.com/2022/02/09/mission-2022-

insights-daily-current-affairs-pib-summary-09-february-2022/

79. With reference to Sowa Rigpa, consider the following statements:

1. It is a traditional system of medicine practised in the Himalayan belt of India.

2. It originated in Bhutan.

Which of the statements given above is/are correct?

(a) 1 only

(b) 2 only

(c) Both 1 and 2

(d) Neither 1 nor 2

Ans: (a)

Explanation:

• It is a traditional system of medicine practised in the Himalayan belt of

India.

• It originated in Tibet and popularly practiced in countries namely, India,

Nepal, Bhutan, Mongolia, and Russia.

• The majority of theory and practice of Sowa-Rigpa is similar to “Ayurveda”.

• Yuthog Yonten Gonpo from Tibet is believed to be the father of Sowa Rigpa.

Refer: Facts for Prelims: https://www.insightsonindia.com/2022/02/09/mission-2022-

insights-daily-current-affairs-pib-summary-09-february-2022/

80. The Kabini River is one of the major tributaries of the

(a) River Cauvery

(b) River Krishna

(c) River Godavari

(d) River Pampa

Ans: (a)

Explanation:

• The Kabini River is one of the major tributaries of the river Cauvery in

southern India.

• It originates near Kavilumpara in Kozhikode district of Kerala state by the

confluence of the Panamaram River and the Mananthavady River.

Revision Through MCQs (RTM) Compilation (Feb - 2022)

Telegram: https://t.me/insightsIAStips Youtube: https://www.youtube.com/channel/UCpoccbCX9GEIwaiIe4HLjwA

53

Refer: Facts for Prelims: https://www.insightsonindia.com/2022/02/09/mission-2022-

insights-daily-current-affairs-pib-summary-09-february-2022/

RTM- REVISION THROUGH MCQS –10th-Feb-2022 81. Consider the following statements:

1. First All India Whips Conference was organized in 1952.

2. The All India Whips Conference is organized by the Vice-President of India.

3. India inherited the concept of the whip from the British parliamentary system.

Which of the given above statements is/are correct?

(a) 1 and 3 only

(b) 3 only

(c) 2 and 3 only

(d) 1 and 2 only

Ans: (a)

Explanation:

• Stat1 and 2: First All India Whips Conference was organized at Indore in

1952, in the very first year of general elections to the First Lok Sabha.

• Organizing the All India Whips Conference is one of the functions assigned to

the Ministry of Parliamentary affairs, under Government of India (Allocation of

Business) Rules, 1961 made under article 77(3) of the Constitution.

• Stat3: India inherited the concept of the whip from the British parliamentary

system.

Refer: https://www.insightsonindia.com/2022/02/10/how-are-mps-questions-allowed-

disallowed-2/

82. Consider the following statements:

1. The term ‘Office of whip’ is well-defined in the Constitution of India.

2. The Two-line whip is issued to members directing them to vote as per the party line.

Which of the given above statements is/are correct?

(a) 1 only

(b) 2 only

(c) Both 1 and 2

(d) Neither 1 nor 2

Ans: (d)

Explanation:

Revision Through MCQs (RTM) Compilation (Feb - 2022)

Telegram: https://t.me/insightsIAStips Youtube: https://www.youtube.com/channel/UCpoccbCX9GEIwaiIe4HLjwA

54

• Stat1: The ‘Office of whip’ is mentioned neither in the Constitution of India

nor in the Rules of the House nor in a Parliamentary Statute. It is based on

the conventions of the parliamentary government.

• Stat2: There are three types of whips or instructions issued by the party

o One-line whip: Issued to inform members of a party about a vote. It

allows a member to abstain in case they decide not to follow the party

line.

o Two-line whip: Issued to direct the members to be present in the House

at the time of voting.

o Three-line whip: Issued to members directing them to vote as per

the party line.

Refer: https://www.insightsonindia.com/2022/02/10/how-are-mps-questions-allowed-

disallowed-2/

83. Consider the following statements:

1. Foreigners’ Tribunals are quasi-judicial bodies established as per the administrative

tribunals act, 1985.

2. As per the new rules, only the State administration could move the Tribunal against

a suspect.

Which of the statements given above is/are correct?

(a) 1 only

(b) 2 only

(c) Both 1 and 2

(d) Neither 1 nor 2

Ans: (d)

Explanation:

• S1: Foreigners’ Tribunals are quasi-judicial bodies established as per the

Foreigners’ Tribunal Order, 1964 and the Foreigners’ Act, 1946.

o Composition: Advocates not below the age of 35 years of age with at

least 7 years of practice (or) Retired Judicial Officers from the Assam

Judicial Service (or) Retired IAS of ACS Officers (not below the rank of

Secretary/Addl. Secretary) having experience in quasi-judicial works.

• S2: The amended order (Foreigners (Tribunal) Order, 2019) also

empowers individuals to approach the Tribunals.

o Earlier, only the State administration could move the Tribunal against a

suspect.

Revision Through MCQs (RTM) Compilation (Feb - 2022)

Telegram: https://t.me/insightsIAStips Youtube: https://www.youtube.com/channel/UCpoccbCX9GEIwaiIe4HLjwA

55

Refer: https://www.insightsonindia.com/2022/02/10/what-are-detention-centres-for-

foreigners-2/

84. Consider the following statements about Law Commission of India:

1. It is neither a constitutional body nor a statutory body.

2. The recommendations of the commission are not binding on the government.

Which of the statements given above is/are correct?

(a) 1 only

(b) 2 only

(c) Both 1 and 2

(d) Neither 1 nor 2

Ans: (c)

Explanation:

• S1: The Law Commission of India is an executive body whose chief function is

legal reforms in the country.

• S2: The Commission submits reports to the government on various matters of a

legal nature. The reports are not binding on the government, which can

either reject or accept them.

Refer: https://www.insightsonindia.com/2022/02/10/death-penalty/

85. Consider the following statements:

1. United Nations Peacekeeping is a joint effort between the Department of Peace

Operations and the Department of Operational Support.

2. Every peacekeeping mission is authorized by the United Nations General Assembly.

Which of the statements given above is/are correct?

(a) 1 only

(b) 2 only

(c) Both 1 and 2

(d) Neither 1 nor 2

Ans: (a)

Explanation:

• United Nations Peacekeeping is a joint effort between the Department of

Peace Operations and the Department of Operational Support.

• Every peacekeeping mission is authorized by the Security Council.

Refer: https://www.insightsonindia.com/2022/02/10/un-peacekeepers-4/

86. Consider the following statements about International Court of Justice (ICJ):

1. It was established in 1945 by the United Nations charter.

Revision Through MCQs (RTM) Compilation (Feb - 2022)

Telegram: https://t.me/insightsIAStips Youtube: https://www.youtube.com/channel/UCpoccbCX9GEIwaiIe4HLjwA

56

2. It is situated at the Peace Palace in The Hague.

3. Its official languages are English and French.

Which of the given above statements is/are correct?

(a) 1 and 2 only

(b) 2 and 3 only

(c) 2 and 3 only

(d) 1, 2 and 3

Ans: (d)

Explanation:

• S1: ICJ was established in 1945 by the United Nations charter and started

working in April 1946.

• S2: It is the principal judicial organ of the United Nations, situated at the Peace

Palace in The Hague (Netherlands).

• S3: ICJ is assisted by a Registry, its administrative organ. Its official languages

are English and French.

Refer: https://www.insightsonindia.com/2022/02/10/international-court-of-justice-icj/

87. Consider the following statements:

1. Diego Garcia is an overseas territory of the United Kingdom.

2. It is located in the central Pacific Ocean.

Which of the statements given above is/are correct?

(a) 1 only

(b) 2 only

(c) Both 1 and 2

(d) Neither 1 nor 2

Ans: (a)

Explanation:

• It is an island of the British Indian Ocean Territory, an overseas territory of

the United Kingdom.

• It is the largest of 60 small islands comprising the Chagos Archipelago.

• The Portuguese were the first Europeans to find it and it was then settled by the

French in the 1790s and transferred to British rule after the Napoleonic Wars.

• In 1965, Britain separated the Chagos Islands from Mauritius and set up a joint

military base with the United States on Diego Garcia.

• Britain insists the islands belong to London and has renewed a lease agreement

with the United States to use Diego Garcia until 2036.

Revision Through MCQs (RTM) Compilation (Feb - 2022)

Telegram: https://t.me/insightsIAStips Youtube: https://www.youtube.com/channel/UCpoccbCX9GEIwaiIe4HLjwA

57

Refer: Facts for Prelims: https://www.insightsonindia.com/2022/02/10/mission-2022-

insights-daily-current-affairs-pib-summary-10-february-2022/

88. Consider the following statements about the Bharat Gaurav Scheme:

1. Under this Scheme, theme-based tourist circuit trains can be run either by private

or State-owned operators.

2. The service providers will be allowed to design/furnish the interior of the coaches

based on the theme and put advertising inside and outside of the train.

Which of the statement given above is/are correct?

(a) 1 only

(b) 2 only

(c) Both 1 and 2

(d) Neither 1 nor 2

Ans: (c)

Explanation:

• Under this Scheme, theme-based tourist circuit trains, on the lines of the

Ramayana Express, can be run either by private or State-owned operators.

• Key features of the scheme:

o Service providers, who can be an individual, company, society, trust,

joint venture or consortium will be free to decide themes/circuits.

o They will offer an all-inclusive package to tourists including rail travel,

hotel accommodation and sightseeing arrangement, visit to

historical/heritage sites, tour guides etc.

o They have full flexibility to decide the package cost.

o The service providers will also be able to design/furnish the interior

of the coaches based on the theme and put branding or advertising

inside and outside of the train.

Refer: https://www.insightsonindia.com/2021/11/29/bharat-gaurav-scheme/

89. Gulf Of Aden forms a natural sea link between the

(a) Mediterranean and Red Sea

(b) Red Sea and the Arabian Sea

(c) Mediterranean and Black Seas

(d) Persian Gulf and Caspian Sea

Ans: (b)

Explanation:

Revision Through MCQs (RTM) Compilation (Feb - 2022)

Telegram: https://t.me/insightsIAStips Youtube: https://www.youtube.com/channel/UCpoccbCX9GEIwaiIe4HLjwA

58

• Gulf of Aden, deepwater basin that forms a natural sea link between the Red

Sea and the Arabian Sea. Named for the seaport of Aden, in southern Yemen,

the gulf is situated between the coasts of Arabia and the Horn of Africa.

Refer: facts for prelims: https://www.insightsonindia.com/2021/11/29/mission-2022-

insights-daily-current-affairs-pib-summary-29-november-2021/

90. Consider the following statements about Poshan Gyan:

1. It is a national digital repository on health and nutrition.

2. It is an initiative of National Health Authority (NHA).

Which of the statements given above is/are correct?

(a) 1 only

(b) 2 only

(c) Both 1 and 2

(d) Neither 1 nor 2

Ans: (a)

Explanation:

• It is a national digital repository on health and nutrition.

• Launched by NITI Aayog, in partnership with Bill and Melinda Gates

Foundation and Centre for Social and Behaviour Change, Ashoka University.

• The Poshan Gyan repository is conceptualized as a resource, enabling search of

communication materials on 14 thematic areas of health and nutrition across

diverse languages, media types, target audiences and sources.

Refer: facts for prelims: https://www.insightsonindia.com/2021/11/29/mission-2022-

insights-daily-current-affairs-pib-summary-29-november-2021/

Revision Through MCQs (RTM) Compilation (Feb - 2022)

Telegram: https://t.me/insightsIAStips Youtube: https://www.youtube.com/channel/UCpoccbCX9GEIwaiIe4HLjwA

59

RTM- REVISION THROUGH MCQS –11th-Feb-2022 91. With reference to Parliamentary Privilege’s, consider the following statements:

1. According to the Constitution of India, the privileges and immunities of MP’s are to

be defined by Committee of Privileges.

2. The Speaker of Lok Sabha or Rajya Sabha (RS) Chairperson is the first level of

scrutiny of a privilege motion.

Which of the given above statements is/are correct?

(a) 1 only

(b) 2 only

(c) Both 1 and 2

(d) Neither 1 nor 2

Ans: (b)

Explanation:

• S1: According to the Constitution, the powers, privileges and immunities of

Parliament and MP’s are to be defined by Parliament. No law has so far been

enacted in this respect. In the absence of any such law, it continues to be

governed by British Parliamentary conventions.

• S2: The Speaker/RS chairperson is the first level of scrutiny of a privilege

motion.

• The Speaker/Chair can decide on the privilege motion himself or herself or refer

it to the privileges committee of Parliament.

• If the Speaker/Chair gives consent under relevant rules, the member concerned

is given an opportunity to make a short statement.

Refer: https://www.insightsonindia.com/2022/02/11/privilege-motion-2/

92. Consider the following statements

1. Originally, the Constitution of India expressedly mentioned five privileges.

2. The parliamentary privileges do not extend to the president who is also an integral

part of the Parliament.

3. Till now, the Parliament of India has not made any special law to exhaustively codify

all the privileges.

Which of the given above statements is/are correct?

(a) 1 and 3 only

(b) 2 only

(c) 2 and 3 only

(d) 1, 2 and 3

Ans: (c)

Revision Through MCQs (RTM) Compilation (Feb - 2022)

Telegram: https://t.me/insightsIAStips Youtube: https://www.youtube.com/channel/UCpoccbCX9GEIwaiIe4HLjwA

60

Explanation:

• S2: The parliamentary privileges do not extend to the President who is also

an integral part of the Parliament. Article 361 of the Constitution provides for

privileges for the President.

• S1 and S3: Originally, the Constitution (Article 105) expressedly mentioned

two privileges, that is, freedom of speech in Parliament and right of publication

of its proceedings. With regard to other privileges, it provided that they were to

be the same as those of the British House of Commons, its committees and its

members on the date of its commencement (ie, 26 January, 1950), until defined

by Parliament.

• The 44th Amendment Act of 1978 provided that the other privileges of each

House of Parliament, its committees and its members are to be those which they

had on the date of its commencement (ie, 20 June, 1979), until defined by

Parliament. This means that the position with regard to other privileges remains

same. In other words, the amendment has made only verbal changes by

dropping a direct reference to the British House of Commons, without making

any change in the implication of the provision.

• It should be noted here that the Parliament, till now, has not made any

special law to exhaustively codify all the privileges. They are based on five

sources, namely, 1. Constitutional provisions, 2. Various laws made by

Parliament, 3. Rules of both the Houses, 4. Parliamentary conventions, and 5.

Judicial interpretations.

Refer: https://www.insightsonindia.com/2022/02/11/privilege-motion-2/

93. Consider the following statements:

1. A notice is moved in the form of a motion by any member of either House against

those being held guilty of breach of privilege.

2. The leader of the house is the first level of scrutiny of a privilege motion.

Which of the above statements is/are correct?

(a) 1 only

(b) 2 only

(c) Both 1 and 2

(d) Neither 1 nor 2

Ans: (a)

Explanation:

• A notice is moved in the form of a motion by any member of either House against

those being held guilty of breach of privilege. Each House also claims the right to

Revision Through MCQs (RTM) Compilation (Feb - 2022)

Telegram: https://t.me/insightsIAStips Youtube: https://www.youtube.com/channel/UCpoccbCX9GEIwaiIe4HLjwA

61

punish as contempt actions which, while not breach of any specific privilege, are

offences against its authority and dignity.

• The Speaker/RS chairperson is the first level of scrutiny of a privilege

motion. The Speaker/Chair can decide on the privilege motion himself or herself

or refer it to the privileges committee of Parliament.

Refer: https://www.insightsonindia.com/2022/02/11/privilege-motion-2/

94. Consider the following statements about the Association of Southeast Asian Nations:

1. It is a political and economic union of 10 member states in Southeast Asia.

2. It was created on 8 August 1967.

3. ASEAN Secretariat is based in Bangkok, Thailand.

Which of the statements given above is/are correct?

(a) 1 and 2 only

(b) 2 and 3 only

(c) 1 and 3 only

(d) 1, 2 and 3

Ans: (a)

Explanation:

• The Association of Southeast Asian Nations is a regional organization which

was established to promote political and social stability amid rising tensions

among the Asia-Pacific’s post-colonial states.

• The motto of ASEAN is “One Vision, One Identity, One Community”.

• ASEAN Secretariat – Indonesia, Jakarta.

• Genesis:

o Established in 1967 with the signing of the ASEAN Declaration

(Bangkok Declaration) by its founding fathers.

• Founding Fathers of ASEAN are: Indonesia, Malaysia, Philippines, Singapore

and Thailand.

• Ten Members: Brunei, Cambodia, Indonesia, Laos, Malaysia, Myanmar, the

Philippines, Singapore, Thailand, and Vietnam.

Refer: https://www.insightsonindia.com/2022/02/11/asean-2/

95. What is ‘Tokamak’, sometimes seen in the news?

(a) An Israeli radar system

(b) An American anti-missile system

(c) A defence collaboration between Japan and South Korea

(d) A device used in nuclear-fusion research for magnetic confinement of plasma

Revision Through MCQs (RTM) Compilation (Feb - 2022)

Telegram: https://t.me/insightsIAStips Youtube: https://www.youtube.com/channel/UCpoccbCX9GEIwaiIe4HLjwA

62

Ans: (d)

Explanation:

• A tokamak is a device which uses a powerful magnetic field to confine hot

plasma in the shape of a torus. The tokamak is one of several types of magnetic

confinement devices being developed to produce controlled thermonuclear fusion

power.

Refer: https://www.insightsonindia.com/2022/02/11/nuclear-fusion-technology/

96. Consider the following about International Thermonuclear Reactor (ITER).

1. ITER will produce more energy than the energy required to operate it.

2. China and India are some of the countries collaborating in the ITER project.

3. ITER will be built largely through in-kind contributions from the participant

countries.

4. ITER-India is the first of its kind experimental fusion reactor technology cell that is

hosting scientists from across the globe.

Select the correct answer using the codes below.

(a) 1 and 3 only

(b) 1 and 4 only

(c) 1, 2 and 3 only

(d) 2 and 4 only

Ans: (c)

Explanation: International Thermonuclear Reactor (ITER):

• ITER is a step towards future production of electricity from fusion energy.

Nuclear Fusion is the process in which the Sun and the stars produce the

energy by fusing light nuclei of hydrogen.

• Stat1: ITER will produce at least ten times more energy than the energy required

to operate it. In future demo or commercial reactors based on fusion, this energy

can be converted to electricity.

• Stat2: An unprecedented international scientific and technological collaboration

representing more than half the worlds human population is presently involved

towards construction of ITER. The ITER partners are presently the People’s

Republic of China, the European Union, India, Japan, the Republic of Korea, the

Russian Federation and the United States of America.

• Stat3 and 4: ITER will be built mostly through in-kind contributions from the

participant countries (Parties) in the form of components manufactured by the

Parties and delivered/installed at ITER.

Revision Through MCQs (RTM) Compilation (Feb - 2022)

Telegram: https://t.me/insightsIAStips Youtube: https://www.youtube.com/channel/UCpoccbCX9GEIwaiIe4HLjwA

63

• ITER-India is the Indian Domestic Agency (DA), formed with the

responsibility to provide to ITER the Indian contribution.

Refer: https://www.insightsonindia.com/2022/02/11/nuclear-fusion-technology/

97. With reference to Radio Frequency Identification (RFID), consider the following

statements:

1. RFID uses electromagnetic fields to automatically identify and track tags attached to

objects.

2. An RFID system consists of a tiny radio transponder, a radio receiver and

transmitter.

3. Unlike a barcode, the tag does not need to be within the line of sight of the reader.

Which of the statements given above is/are correct?

(a) 1 and 2 only

(b) 2 and 3 only

(c) 1 and 3 only

(d) 1, 2 and 3

Ans: (d)

Explanation:

• Radio-frequency identification (RFID) uses electromagnetic fields to

automatically identify and track tags attached to objects. An RFID system

consists of a tiny radio transponder, a radio receiver and transmitter. When

triggered by an electromagnetic interrogation pulse from a nearby RFID reader

device, the tag transmits digital data, usually an identifying inventory number,

back to the reader. This number can be used to track inventory goods.

• Passive tags are powered by energy from the RFID reader's interrogating radio

waves. Active tags are powered by a battery and thus can be read at a greater

range from the RFID reader, up to hundreds of meters.

• Unlike a barcode, the tag does not need to be within the line of sight of the

reader, so it may be embedded in the tracked object. RFID is one method of

automatic identification and data capture (AIDC).

Refer: Facts for Prelims: https://www.insightsonindia.com/2022/02/11/mission-2022-

insights-daily-current-affairs-pib-summary-11-february-2022/

98. Consider the following statements:

1. The Infrastructure for Resilient Island States (IRIS) initiative has been launched by

India for developing the infrastructure of small island nations vulnerable to climate

change.

Revision Through MCQs (RTM) Compilation (Feb - 2022)

Telegram: https://t.me/insightsIAStips Youtube: https://www.youtube.com/channel/UCpoccbCX9GEIwaiIe4HLjwA

64

2. The IRIS initiative is a part of the Small Island Developing States (SIDS) group.

Which of the statement given above is/are correct?

(a) 1 only

(b) 2 only

(c) Both 1 and 2

(d) Neither 1 nor 2

Ans: (a)

Explanation: Infrastructure for Resilient Island States

• This initiative has been launched by India for developing the infrastructure of

small island nations vulnerable to climate change.

• The new initiative is the result of cooperation between India, the U.K. and

Australia and included the participation of leaders of small island nations such

as Fiji, Jamaica and Mauritius.

• Implementation: The IRIS initiative is a part of the Coalition for Disaster

Resilient Infrastructure (CDRI) that would focus on building capacity, having

pilot projects, especially in small island developing states.

Refer: https://www.insightsonindia.com/2021/11/03/infrastructure-for-resilient-island-

states/

99. Consider the following statements about the Coalition for Disaster Resilient

Infrastructure (CDRI):

1. It is a platform where knowledge is generated and exchanged on different aspects of

disaster and climate resilience of infrastructure.

2. It is the only international collaboration set up by India in the climate change

sphere.

Which of the following statements given above is/are correct?

(a) 1 only

(b) 2 only

(c) Both 1 and 2

(d) Neither 1 nor 2

Ans: (a)

Explanation: About CDRI:

• Launched by Modi in September 2019 at the UN Secretary-General’s Climate

Action Summit in New York, US.

• It is a platform where knowledge is generated and exchanged on different

aspects of disaster and climate resilience of infrastructure.

Revision Through MCQs (RTM) Compilation (Feb - 2022)

Telegram: https://t.me/insightsIAStips Youtube: https://www.youtube.com/channel/UCpoccbCX9GEIwaiIe4HLjwA

65

• It will create a mechanism to assist countries to upgrade their capacities and

practices, with regard to infrastructure development in accordance with their

risk context and economic needs.

• So far, 25 other countries, including Germany, Italy, Japan, Australia, and the

United States have joined this coalition.

• CDRI is the second international collaboration set up by India in the climate

change sphere, the other being the International Solar Alliance that has now

evolved to the status of a “treaty-based” intergovernmental organisation.

Refer: https://www.insightsonindia.com/2021/11/03/infrastructure-for-resilient-island-

states/

100. Consider the following statements:

1. The United Nations Convention against Corruption (UNCAC) has a ‘Protocol against

the Smuggling of Migrants by Land, Sea and Air’.

2. The UNCAC is the ever-first legally binding global anti-corruption instrument.

3. A highlight of the United Nations Convention against Transnational Organized Crime

(UNTOC) is the inclusion of a specific chapter aimed at returning assets to their

rightful owners from whom they had been taken illicitly.

4. The United Nations Office on Drugs and Crime (UNODC) is mandated by its member

States to assist in the implementation of both UNCAC and UNTOC.

Which of the statements given above are correct?

(a) 1 and 3 only

(b) 2, 3 and 4 only

(c) 2 and 4 only

(d) 1, 2, 3 and 4

Ans: (c)

Explanation:

• S1 is a protocol under UNTOC, and S3 is wrong as UNCAC has that provision.

• The United Nations Convention against Corruption is the only legally binding

international anti-corruption multilateral treaty

• The United Nations Office on Drugs and Crime (UNODC) is mandated by its

Member States to assist in the implementation of both Conventions, which along

with the UN Drug Conventions of 1961, 1971 and 1988 underpin all the

operational work of UNODC.

• See https://www.unodc.org/southasia/en/frontpage/2011/may/indian-govt-

ratifies-two-un-conventions.html

Refer: UPSC CSE 2018

Revision Through MCQs (RTM) Compilation (Feb - 2022)

Telegram: https://t.me/insightsIAStips Youtube: https://www.youtube.com/channel/UCpoccbCX9GEIwaiIe4HLjwA

66

Revision Through MCQs (RTM) Compilation (Feb - 2022)

Telegram: https://t.me/insightsIAStips Youtube: https://www.youtube.com/channel/UCpoccbCX9GEIwaiIe4HLjwA

67

RTM- REVISION THROUGH MCQS –12th-Feb-2022

101. Consider the following statements about anti-defection law:

1. It was added to the Constitution by the 62nd Amendment Act.

2. It includes situations in which an independent MLA, too, joins a party after the

election.

Which of the statements given above is/are correct?

(a) 1 only

(b) 2 only

(c) Both 1 and 2

(d) Neither 1 nor 2

Ans: (b)

Explanation:

• It specifies the circumstances under which changing of political parties by

legislator’s invites action under the law.

• It was added to the Constitution by the 52nd Amendment Act.

• It includes situations in which an independent MLA, too, joins a party after the

election.

Refer: https://www.insightsonindia.com/2022/02/12/anti-defection-law-4/

102. Consider the following statements:

1. The Tenth Schedule was inserted by the 52nd Amendment (1985) to the

Constitution of India.

2. Any question regarding disqualification arising out of defection is to be decided by

the Election Commission of India.

Which of the given above statements is/are correct?

(a) 1 only

(b) 2 only

(c) Both 1 and 2

(d) Neither 1 nor 2

Ans: (a)

Explanation:

• S2: Any question regarding disqualification arising out of defection is to be

decided by the presiding officer of the House.

• S1: The anti-defection law was passed in 1985 through the 52nd

Amendment to the Constitution. The law that was contained in the 10th

Revision Through MCQs (RTM) Compilation (Feb - 2022)

Telegram: https://t.me/insightsIAStips Youtube: https://www.youtube.com/channel/UCpoccbCX9GEIwaiIe4HLjwA

68

Schedule of the Constitution came into effect on March 1, 1985. It was

formulated to bring in stability in the Indian political system.

Refer: https://www.insightsonindia.com/2022/02/12/anti-defection-law-4/

103. Consider the following statements:

1. Under the anti-defection law, the power to decide the disqualification of an MP or

MLA rests with the presiding officer of the legislature.

2. The law allows a party to merge with or into another party provided that at least

one-thirds of its legislators are in favour of the merger.

Which of the statements given above is/are correct?

(a) 1 only

(b) 2 only

(c) Both 1 and 2

(d) Neither 1 nor 2

Ans: (a)

Explanation:

• S1: Under the anti-defection law, the power to decide the disqualification of an

MP or MLA rests with the presiding officer of the legislature.

• The law does not specify a time frame in which such a decision has to be made.

• the Supreme Court observed that anti-defection cases should be decided by

Speakers in three months’ time.

• S2: The law allows a party to merge with or into another party provided that at

least two-thirds of its legislators are in favour of the merger.

• On being elected as the presiding officer of the House, if a member, voluntarily

gives up the membership of his party or rejoins it after he ceases to hold that

office, he won’t be disqualified.

Refer: https://www.insightsonindia.com/2022/02/12/anti-defection-law-4/

104. The Tenth Schedule was introduced in the Constitution of India during the prime

ministership of

(a) Jawaharlal Nehru

(b) Lal Bahadur Shastri

(c) Indira Gandhi

(d) Rajiv Gandhi

Ans: (d)

Explanation:

Revision Through MCQs (RTM) Compilation (Feb - 2022)

Telegram: https://t.me/insightsIAStips Youtube: https://www.youtube.com/channel/UCpoccbCX9GEIwaiIe4HLjwA

69

• Rajiv Gandhi was convinced that peace and political stability are a sine qua non

for progress and prosperity. Therefore, to check the malaise of political horse-

trading (popularly known as Aaya Ram Gaya Ram) and to curb corruption and

political opportunism, he got the 52nd Constitution Amendment Act, 1985

(commonly known as Anti-Defection Act) passed soon after taking over as prime

minister.

• It provided for disqualification of an elected member of legislature on the

grounds of defection to another political party. It was amended subsequently

through the 91st Constitution Amendment Act, 2003.

Refer: https://www.insightsonindia.com/2022/02/12/anti-defection-law-4/

105. Consider the following statements:

1. A Uniform Civil Code is one that would provide for one law for the entire country,

applicable to all religious communities in their personal matters.

2. Article 44 is one of the Directive Principles of State Policy is not enforceable by any

court but the principles laid down therein are fundamental in governance.

3. The duty of the state is greater in other directive principles than in Article 44 of the

Constitution of India.

Which of the statements given above are correct?

(a) 1 only

(b) 2 only

(c) 1 and 2 only

(d) 1, 2 and 3

Ans: (d)

Explanation:

• S1: A Uniform Civil Code is one that would provide for one law for the entire

country, applicable to all religious communities in their personal matters such

as marriage, divorce, inheritance, adoption etc.

• S2: Article 44 is one of the Directive Principles of State Policy. These, as

defined in Article 37, are not justiciable (not enforceable by any court) but

the principles laid down therein are fundamental in governance.

• S3: Article 43 mentions “state shall endeavour by suitable legislation”, while the

phrase “by suitable legislation” is absent in Article 44. All this implies that the

duty of the state is greater in other directive principles than in Article 44.

Refer: https://www.insightsonindia.com/2022/02/12/uniform-civil-code-5/

Revision Through MCQs (RTM) Compilation (Feb - 2022)

Telegram: https://t.me/insightsIAStips Youtube: https://www.youtube.com/channel/UCpoccbCX9GEIwaiIe4HLjwA

70

106. Consider the following statements about Gulf Cooperation Council (GCC):

1. It is an economic and political union comprising of all the Arab countries of the

Persian Gulf except Iraq.

2. The council’s main headquarter is in the city of Riyadh in Saudi Arabia.

Which of the statements given above is/are correct?

(a) 1 only

(b) 2 only

(c) Both 1 and 2

(d) Neither 1 nor 2

Ans: (c)

Explanation:

• It is an economic and political union comprising of all the Arab countries of the

Persian Gulf except Iraq.

• The council’s main headquarter is in the city of Riyadh in Saudi Arabia.

Refer: https://www.insightsonindia.com/2022/02/12/comprehensive-economic-

cooperation-agreement-ceca-2/

107. A SWIFT Code, sometime seen in the news, is related to

(a) International Banking Transaction

(b) Payment Gateway System

(c) Quick Toll Plaza Electronic Payment

(d) None of the above

Revision Through MCQs (RTM) Compilation (Feb - 2022)

Telegram: https://t.me/insightsIAStips Youtube: https://www.youtube.com/channel/UCpoccbCX9GEIwaiIe4HLjwA

71

Ans: (a)

Explanation:

• The SWIFT – Society for Worldwide Interbank Financial Telecommunication

– is a secure financial message carrier — it transports messages from one

bank to its intended bank recipient. Its core role is to provide a secure

transmission channel between banks.

Refer: https://www.insightsonindia.com/2022/02/12/society-for-worldwide-interbank-

financial-telecommunication-swift-3/

108. Consider the following statements about International Labour Organisation (ILO):

1. It was established by the Treaty of Versailles in 1919.

2. It got the Nobel Peace Prize in 1969.

3. It is the only tripartite U.N. agency.

Which of the statements given above is/are correct?

(a) 1 and 2 only

(b) 2 and 3 only

(c) 1 and 3 only

(d) 1, 2 and 3

Ans: (d)

Explanation:

• Established as an agency for the League of Nations following World War I.

• Established by the Treaty of Versailles in 1919.

• It became the first specialised agency of the United Nations (UN) in the year

1946.

• It got the Nobel Peace Prize in 1969.

• It is the only tripartite U.N. agency. It brings together governments, employers

and workers.

• Headquarters: Geneva, Switzerland.

Refer: facts for prelims: https://www.insightsonindia.com/2022/02/12/mission-2022-

insights-daily-current-affairs-pib-summary-12-february-2022-2/

109. World Employment and Social Outlook report is released by which of the following?

(a) International Monetary Fund

(b) World Economic Forum

(c) International Labour Organization

(d) World Trade Organization

Ans: (c)

Revision Through MCQs (RTM) Compilation (Feb - 2022)

Telegram: https://t.me/insightsIAStips Youtube: https://www.youtube.com/channel/UCpoccbCX9GEIwaiIe4HLjwA

72

Explanation: Key reports by ILO:

• World Employment and Social Outlook.

• Global Wage Report

Refer: facts for prelims: https://www.insightsonindia.com/2022/02/12/mission-2022-

insights-daily-current-affairs-pib-summary-12-february-2022-2/

110. Milan 2022 naval exercise is hosted by which one of the following city?

(a) Ernakulam

(b) Visakhapatnam

(c) Andaman

(d) Port Blair

Ans: (b)

Explanation:

• MILAN, a Multilateral Naval Exercise hosted by India, made a modest

beginning in the Andaman and Nicobar Islands in 1995 with participation of four

littoral navies.

• This biennial congregation of friendly navies, over the last two and a half

decades, has progressively grown in magnitude with the previous edition in 2018

being attended by 17 countries.

• MILAN 2022 has been scheduled for the first time at Visakhapatnam, the City of

Destiny. MILAN 2022 is the eleventh edition of the event and would be held

under the aegis of Eastern Naval Command.

• This is the first time the exercise has been shifted from the Andaman to

Vizag, as the scale of the exercise has been enhanced.

Refer: facts for prelims: https://www.insightsonindia.com/2022/02/12/mission-2022-

insights-daily-current-affairs-pib-summary-12-february-2022-2/

RTM- REVISION THROUGH MCQS –14th-Feb-2022

111. Consider the following statements:

1. Shri Madhvacharya was born in Sriperumbudur, Tamil Nadu.

2. Shri Madhvacharya propounded the philosophy of Dwaita.

3. Purandara Dasa and Madhvacharya are contemporaries.

Which of the statements given above is/are correct?

(a) 1 and 2 only

(b) 2 only

(c) 1 and 3 only

(d) 1, 2 and 3

Revision Through MCQs (RTM) Compilation (Feb - 2022)

Telegram: https://t.me/insightsIAStips Youtube: https://www.youtube.com/channel/UCpoccbCX9GEIwaiIe4HLjwA

73

Ans: (b)

Explanation:

• S1: Shri Madhvacharya was born near Udupi (Karnataka). He was born in

1238, on the auspicious day of Vijayadashami, and he was named Vasudeva.

• S2: He propounded the philosophy of Dwaita or Dualism.

• S3: Purandara Dasa (born Srinivasa Nayaka; c. 1484 – c. 1565) was a Haridasa

philosopher and a follower of Madhwacharya's Dwaitha philosophy -saint

from present-day Karnataka, India.

Refer: https://www.insightsonindia.com/2022/02/14/sri-madhvacharya/

112. Consider the following statements:

1. The basic tenet of Dvaita philosophy is the refutation of the Mayavada of Sri

Shankara.

2. The Dvaita Vedanta School believes that God and the individual souls exist as

independent realities.

Which of the statements given above is/are correct?

(a) 1 only

(b) 2 only

(c) Both 1 and 2

(d) Neither 1 nor 2

Ans: (c)

Explanation: About Dvaita philosophy:

• The basic tenet of Dvaita philosophy is the refutation of the Mayavada of Sri

Shankara. Dvaitha emphasizes that the world is real and not just an illusion.

• The soul is bound to this world through ignorance.

• The way for the soul to release itself from this bondage is to seek the grace of Sri

Hari.

• To reach Sri Hari, one has to practice Bhakthi, there is no other way.

• To practice Bhakthi, one needs to meditate.

• To meditate, one needs to clear the mind and attain detachment by studying the

sacred texts.

Refer: https://www.insightsonindia.com/2022/02/14/sri-madhvacharya/

113. Starlink is a satellite internet constellation operated by

(a) SpaceX

(b) NASA

(c) Blue Origin

Revision Through MCQs (RTM) Compilation (Feb - 2022)

Telegram: https://t.me/insightsIAStips Youtube: https://www.youtube.com/channel/UCpoccbCX9GEIwaiIe4HLjwA

74

(d) Virgin Galactic

Ans: (a)

Explanation:

• Starlink is a satellite internet constellation operated by SpaceX providing

satellite Internet access coverage to most of the Earth.

• Recently, Elon Musk’s Starlink has lost 40 satellites that were caught in a

geomagnetic storm a day after they were launched.

Refer: https://www.insightsonindia.com/2022/02/14/geomagnetic-storm/

114. Consider the following statements:

1. Article 29 and 30 of the Constitution of India (COI) contain provisions securing

rights of minorities and minority-run institutions.

2. Article 30 of the COI recognizes linguistic and religious minorities but not those

based on race, ethnicity.

Which of the statements given above is/are correct?

(a) 1 only

(b) 2 only

(c) Both 1 and 2

(d) Neither 1 nor 2

Ans: (c)

Explanation:

• Article 29 and 30 contain provisions securing rights of minorities and minority-

run institutions.

• Constitutional Provisions regarding Minority Educational Institutions:

o Article 30(1) recognizes linguistic and religious minorities but not those

based on race, ethnicity.

o It recognizes the right of religious and linguistic minorities to establish

and administer educational institutions, in effect recognizing the role

educational institutions play in preserving distinct culture.

o A majority community can also establish and administer educational

institutions but they will not enjoy special rights under Article 30(1)(a).

Refer: https://www.insightsonindia.com/2022/02/14/supreme-court-refuses-plea-for-

including-madrasas-vedic-schools-under-rte/

115. What is “S-400 Triumf missile system”, sometimes seen in the news?

(a) A Russian radar system

(b) India’s indigenous anti-missile programme

Revision Through MCQs (RTM) Compilation (Feb - 2022)

Telegram: https://t.me/insightsIAStips Youtube: https://www.youtube.com/channel/UCpoccbCX9GEIwaiIe4HLjwA

75

(c) An American anti-missile system

(d) A Russian designed long-range surface-to-air missile system

Ans: (d)

Explanation:

• The S-400 Triumf is a mobile, surface-to-air missile system (SAM) designed by

Russia.

• It is the most dangerous operationally deployed modern long-range SAM (MLR

SAM) in the world, considered much ahead of the US-developed Terminal High

Altitude Area Defense system (THAAD).

Refer: https://www.insightsonindia.com/2022/02/14/caatsa-2/

116. Consider the following statements about World Food Programme (WFP):

1. It is the food-assistance branch of the United Nations.

2. It was awarded the Nobel Peace Prize in 2019.

3. It is headquartered in Rome.

Which of the statements given above is/are correct?

(a) 1 and 2 only

(b) 2 and 3 only

(c) 1 and 3 only

(d) 1, 2 and 3

Ans: (c)

Explanation:

• The World Food Programme is the food-assistance branch of the United Nations.

It is the world's largest humanitarian organization focused on hunger and food

security, and the largest provider of school meals.

• Founded in 1961, it is headquartered in Rome and has offices in 80 countries.

• The World Food Programme was awarded the Nobel Peace Prize in 2020 for its

efforts to provide food assistance in areas of conflict, and to prevent the use of

food as a weapon of war and conflict.

Refer: https://www.insightsonindia.com/2022/02/14/world-food-programme-3/

117. Fly ash composition primarily consists which of the following?

1. Silicon dioxide

2. Aluminium oxide

3. Calcium oxide

4. Carbon dioxide

Select the correct answer using the code below:

Revision Through MCQs (RTM) Compilation (Feb - 2022)

Telegram: https://t.me/insightsIAStips Youtube: https://www.youtube.com/channel/UCpoccbCX9GEIwaiIe4HLjwA

76

(a) 1, 2 and 3 only

(b) 2, 3 and 4 only

(c) 1 and 4 only

(d) 1, 2, 3 and 4

Ans: (a)

Explanation: What is Fly Ash?

• Popularly known as Flue ash or pulverised fuel ash, it is a coal combustion

product.

• Composition:

• Composed of the particulates that are driven out of coal-fired boilers

together with the flue gases.

• Depending upon the source and composition of the coal being burned, the

components of fly ash vary considerably, but all fly ash includes substantial

amounts of silicon dioxide (SiO2), aluminium oxide (Al2O3) and calcium

oxide (CaO), the main mineral compounds in coal-bearing rock strata.

• Minor constituents include: arsenic, beryllium, boron, cadmium, chromium,

hexavalent chromium, cobalt, lead, manganese, mercury, molybdenum,

selenium, strontium, thallium, and vanadium, along with very small

concentrations of dioxins and PAH compounds. It also has unburnt carbon.

Refer: https://www.insightsonindia.com/2022/02/14/216144/

118. Consider the following statements about Lassa fever:

1. The Lassa virus is named after a town in Uganda.

2. The fever is spread by rats.

Which of the statements given above is/are correct?

(a) 1 only

(b) 2 only

(c) Both 1 and 2

Revision Through MCQs (RTM) Compilation (Feb - 2022)

Telegram: https://t.me/insightsIAStips Youtube: https://www.youtube.com/channel/UCpoccbCX9GEIwaiIe4HLjwA

77

(d) Neither 1 nor 2

Ans: (b)

Explanation:

• The Lassa virus is named after a town in Nigeria where the first cases were

discovered in 1969.

• The disease is primarily found in countries in West Africa including Sierra

Leone, Liberia, Guinea, and Nigeria where it is endemic.

• Spread:

o The fever is spread by rats.

o Person-to-person transmission is also possible.

Refer: https://www.insightsonindia.com/2022/02/15/what-is-lassa-fever/

119. Consider the following statements about Koala:

1. It is a small mammal native to the eastern Himalayas and southwestern China.

2. The species has been listed as Endangered on the IUCN Red List since 2015.

Which of the statements given above is/are correct?

(a) 1 only

(b) 2 only

(c) Both 1 and 2

(d) Neither 1 nor 2

Ans: (d)

Explanation:

• S1: Koala is an arboreal herbivorous marsupial native to Australia.

• S2: Koalas are listed as a vulnerable species by the International Union for

Conservation of Nature.

o In February of 2022, the koala was officially listed as endangered in

the Australian Capital Territory, New South Wales and Queensland.

Refer: facts for prelims: https://www.insightsonindia.com/2022/02/14/mission-2022-

insights-daily-current-affairs-pib-summary-14-february-2022/

120. Atal Innovation Mission is set up under the

(a) Department of Science and Technology

(b) Ministry of Employment

(c) NITI Aayog

(d) Ministry of Skill Development and Entrepreneurship

Ans: (c)

Explanation:

Revision Through MCQs (RTM) Compilation (Feb - 2022)

Telegram: https://t.me/insightsIAStips Youtube: https://www.youtube.com/channel/UCpoccbCX9GEIwaiIe4HLjwA

78

• The Atal Innovation Mission (AIM) is a flagship initiative set up by the NITI

Aayog to promote innovation and entrepreneurship across the length and

breadth of the country.

• Atal Innovation Mission (AIM) in collaboration with UNDP has launched the

Community Innovator Fellowship (CIF) marking the “International Day of Women

& Girls in Science”.

Refer: facts for prelims: https://www.insightsonindia.com/2022/02/14/mission-2022-

insights-daily-current-affairs-pib-summary-14-february-2022/

RTM- REVISION THROUGH MCQS –15th-Feb-2022

121. Which Article of the Constitution of India safeguards one’s right to marry the person

of one’s choice?

(a) Article 19

(b) Article 21

(c) Article 25

(d) Article 29

Ans: (b)

Explanation:

• The right to marry a person of one’s choice is integral to Article 21 (right to life

and liberty) of the Constitution”.

Refer: https://www.insightsonindia.com/2022/02/15/why-is-haryana-bringing-an-anti-

conversion-law/

122. Consider the following statements about ISRO’s Earth Observation Satellite -04:

1. It is a radar imaging satellite capable of providing high-quality images under all

weather conditions.

2. The satellite has a mission life of 100 years.

Which of the statements given above is/are correct?

(a) 1 only

(b) 2 only

(c) Both 1 and 2

(d) Neither 1 nor 2

Ans: (a)

Explanation:

Revision Through MCQs (RTM) Compilation (Feb - 2022)

Telegram: https://t.me/insightsIAStips Youtube: https://www.youtube.com/channel/UCpoccbCX9GEIwaiIe4HLjwA

79

• EOS-04 is a radar imaging satellite capable of providing high-quality images

under all weather conditions.

• It can be used to capture images for agriculture, forestry, flood mapping, soil

moisture and hydrology.

• The satellite has a mission life of 10 years.

Refer: https://www.insightsonindia.com/2022/02/15/eos-04-launch-isros-first-of-2022/

123. With reference to difference between PSLV and GSLV, consider the following

statements:

1. GSLV was developed to launch low-Earth Orbit satellites into polar and sun

synchronous orbits.

2. PSLV was developed to launch the heavier INSAT class of geosynchronous satellites

into orbit.

Which of the statements given above is/are not correct?

(a) 1 only

(b) 2 only

(c) Both 1 and 2

(d) Neither 1 nor 2

Ans: (c)

Explanation: here the directive word is not correct!!

• India has two operational launchers- Polar Satellite Launch Vehicle (PSLV) and

Geosynchronous Satellite Launch Vehicle (GSLV).

• PSLV was developed to launch low-Earth Orbit satellites into polar and sun

synchronous orbits. It has since proved its versatility by launching

geosynchronous, lunar and interplanetary spacecraft successfully.

• On the other hand, GSLV was developed to launch the heavier INSAT class

of geosynchronous satellites into orbit. In its third and final stage, GSLV uses

the indigenously developed cryogenic upper stage.

Refer: https://www.insightsonindia.com/2022/02/15/eos-04-launch-isros-first-of-2022/

124. The outermost part of the Sun's atmosphere is called the

(a) Photosphere

(b) Chromosphere

(c) Corona

(d) None of the above

Ans: (c)

Explanation:

Revision Through MCQs (RTM) Compilation (Feb - 2022)

Telegram: https://t.me/insightsIAStips Youtube: https://www.youtube.com/channel/UCpoccbCX9GEIwaiIe4HLjwA

80

• Corona - The corona is the outermost layer of the Sun, starting at about 1300

miles (2100 km) above the solar surface (the photosphere). The temperature in

the corona is 500,000 K (900,000 degrees F, 500,000 degrees C) or more, up to a

few million K. The corona cannot be seen with the naked eye except during a

total solar eclipse, or with the use of a coronagraph. The corona does not have

an upper limit. Credit: National Solar Observatory

Refer: https://www.insightsonindia.com/2022/02/15/helioswarm-and-muse/

125. The word ModifiedElephant is sometimes mentioned in media in reference to:

(a) Hacking group

(b) Human-wildlife conflict

(c) Genetically modified Elephant

(d) None of the above

Ans: (a)

Explanation:

• It was recently found by an American Agency that ModifiedElephant, a hacking

group, had allegedly planted incriminating evidence on the personal devices of

Indian journalists, human rights activists, human rights defenders, academics

and lawyers.

Revision Through MCQs (RTM) Compilation (Feb - 2022)

Telegram: https://t.me/insightsIAStips Youtube: https://www.youtube.com/channel/UCpoccbCX9GEIwaiIe4HLjwA

81

• What is ModifiedElephant? What’s the issue?

o ModifiedElephant operators have been infecting their targets using

spearphishing emails with malicious file attachments.

o Spearphishing refers to the practice of sending emails to targets that look

like they are coming from a trusted source to either reveal important

information or install different kinds of malware on their computer

systems.

Refer: https://www.insightsonindia.com/2022/02/15/modifiedelephant-a-hacking-group/

126. Consider the following statements about Global Environment Facility:

1. It was established during the Rio Earth Summit of 1992.

2. It is based in New York, United States.

3. It is jointly managed by the United Nations Environment Programme (UNEP) and

International Union for Conservation of Nature (IUCN).

Which of the statements given above is/are correct?

(a) 1 only

(b) 2 and 3 only

(c) 1 and 3 only

(d) 1, 2 and 3

Ans: (a)

Explanation:

• GEF was established during the Rio Earth Summit of 1992.

• It is based in Washington, D.C., United States.

• The GEF is jointly managed by the United Nations Development Programme

(UNDP), the World Bank, and the United Nations Environment Programme

(UNEP).

• 183 nations are united under GEF in partnership with civil society organizations

(CSOs), international institutions, private sector, etc. to address the

environmental issues across the globe.

• It provides funds to the developing countries and transition economies for

projects related to climate change, biodiversity, the ozone layer, etc.

Refer: Facts for Prelims: https://www.insightsonindia.com/2022/02/15/mission-2022-

insights-daily-current-affairs-pib-summary-15-february-2022/

127. Global Environment Facility serves as a financial mechanism for which of the

following conventions?

1. United Nations Convention to Combat Desertification (UNCCD)

Revision Through MCQs (RTM) Compilation (Feb - 2022)

Telegram: https://t.me/insightsIAStips Youtube: https://www.youtube.com/channel/UCpoccbCX9GEIwaiIe4HLjwA

82

2. Stockholm Convention on Persistent Organic Pollutants (POPs)

3. Minamata Convention on Mercury

Select the corrcet answer using the code below:

(a) 1 and 2 only

(b) 2 and 3 only

(c) 1 and 3 only

(d) 1, 2 and 3

Ans: (d)

Explanation: GEF also serves as a financial mechanism for the following

conventions:

• United Nations Convention on Biological Diversity (UNCBD)

• United Nations Framework Convention on Climate Change (UNFCCC)

• United Nations Convention to Combat Desertification (UNCCD)

• Stockholm Convention on Persistent Organic Pollutants (POPs)

• Minamata Convention on Mercury

Refer: Facts for Prelims: https://www.insightsonindia.com/2022/02/15/mission-2022-

insights-daily-current-affairs-pib-summary-15-february-2022/

128. Consider the following statements about One Ocean Summit:

1. It was recently organised by Indian Ocean Commission.

2. It aims to raise the collective level of ambition of the international community on

marine issues.

Which of the statements given above is/are correct?

(a) 1 only

(b) 2 only

(c) Both 1 and 2

(d) Neither 1 nor 2

Ans: (b)

Explanation:

• One Ocean Summit was recently organised by France in Brest, France in

cooperation with the United Nations and the World Bank.

• India also participated in the event.

• The goal of the One Ocean Summit is to raise the collective level of ambition of

the international community on marine issues.

• United Nations has designated the decade between 2021 and 2030 as the

‘Decade of Ocean Science for Sustainable Development’, in a bid to restore

declining marine life and raise awareness.

Revision Through MCQs (RTM) Compilation (Feb - 2022)

Telegram: https://t.me/insightsIAStips Youtube: https://www.youtube.com/channel/UCpoccbCX9GEIwaiIe4HLjwA

83

Refer: Facts for Prelims: https://www.insightsonindia.com/2022/02/15/mission-2022-

insights-daily-current-affairs-pib-summary-15-february-2022/

129. Consider the following statements about the Panthera uncia:

1. It is a common herb of the Himalayas.

2. It was declared ‘endangered’ by the International Union for Conservation of Nature

(IUCN).

Which of the statements given above is/are correct?

(a) 1 only

(b) 2 only

(c) Both 1 and 2

(d) Neither 1 nor 2

Ans: (d)

Explanation: Snow leopard:

• Scientific name: Panthera uncia.

• Habitat: Snow leopards live in the mountains of Central Asia.

• Numbers: There are only between 3,920 and 6,390 snow leopards left in the

wild.

• Range extends through twelve countries: Afghanistan, Bhutan, China, India,

Kazakhstan, Kyrgyzstan, Mongolia, Nepal, Pakistan, Russia, Tajikistan, and

Uzbekistan.

Refer: Refer: https://www.insightsonindia.com/2021/10/25/international-snow-leopard-

day-october-23/

130. With reference to chemical fertilizers in India, consider the following statements:

1. At present, the retail price of chemical fertilizers is market-driven and not

administered by the Government.

2. Ammonia, which is an input of urea, is produced from natural gas.

3. Sulphur, which is a raw material for phosphoric acid fertilizer, is a by-product of all

oil refineries.

Which of the statements given above is/are correct?

(a) 1 only

(b) 2 and 3 only

(c) 2 only

(d) 1,2 and 3

Ans: (b)

Explanation:

Revision Through MCQs (RTM) Compilation (Feb - 2022)

Telegram: https://t.me/insightsIAStips Youtube: https://www.youtube.com/channel/UCpoccbCX9GEIwaiIe4HLjwA

84

• S1: The Union Government subsidies the urea manufacturing units for the cost

of transportation to facilitate the availability of urea at the same maximum retail

price all over the country. Also, the Government is subsidising fertilizers for their

availability at reasonable prices for agricultural purposes. See

https://fert.nic.in/faq

• S2: Common knowledge.

• S3: Sulphur is a by-product of processing natural gas and refining high-sulphur

crude oils. Even if they use low sulphur crude oil, there is still sulphur emission,

so S3 is correct.

• The primary sources of sulfuric acid emissions are combustion of coal, and the

industries that manufacture or use it in production, including metal smelters,

phosphate fertilizer producers, oil refineries, the chemical industry, battery

manufacturers, manufacturers of fabricated metal products, manufacturers of

electronic components, and manufacturers of measuring and controlling devices.

See https://www.sciencedirect.com/topics/chemistry/phosphate-fertilizer and

• https://ibm.gov.in/writereaddata/files/12152016105127IMYB2015_Sulphur_1

5122016_Adv.pdf

Refer: UPSC CSE 2020

RTM- REVISION THROUGH MCQS –16th-Feb-2022 131. Consider the following statements about Organization of Islamic Cooperation:

1. It is an international organization founded in 1956.

2. It is the second largest inter-governmental organization after the United Nations.

3. Its permanent Secretariat is located in Jeddah, Saudi Arabia.

Which of the statements given above is/are correct?

(a) 1 and 2 only

(b) 2 and 3 only

(c) 1 and 3 only

(d) 1, 2 and 3

Ans: (b)

Explanation:

• It is an international organization founded in 1969, consisting of 57 member

states.

• It is the second largest inter-governmental organization after the United

Nations.

Revision Through MCQs (RTM) Compilation (Feb - 2022)

Telegram: https://t.me/insightsIAStips Youtube: https://www.youtube.com/channel/UCpoccbCX9GEIwaiIe4HLjwA

85

• The organisation states that it is “the collective voice of the Muslim

world” and works to “safeguard and protect the interests of the Muslim world in

the spirit of promoting international peace and harmony “.

• The OIC has permanent delegations to the United Nations and the European

Union.

• Permanent Secretariat is in Jeddah, Saudi Arabia.

Refer: https://www.insightsonindia.com/2022/02/16/organization-of-islamic-cooperation-

3/

132. Ukraine shares border with which of the following countries?

1. Poland

2. Slovakia

3. Hungary

4. Romania

5. Moldova

Select the correct answer using the code below:

(a) 1, 2 and 3 only

(b) 2, 3 and 4 only

(c) 1, 4 and 5 only

(d) 1, 2, 3, 4 and 5

Ans: (d)

Explanation:

• Ukraine borders with seven countries: Poland, Slovakia, Hungary, Romania,

Moldova, Russia, and Belarus.

Refer: https://www.insightsonindia.com/2022/02/16/russia-ukraine-crisis/

Revision Through MCQs (RTM) Compilation (Feb - 2022)

Telegram: https://t.me/insightsIAStips Youtube: https://www.youtube.com/channel/UCpoccbCX9GEIwaiIe4HLjwA

86

133. Consider the following statements about SWAMITVA scheme:

1. The scheme is piloted by the Ministry of Rural Development.

2. The scheme is for surveying the land parcels in rural inhabited area using Drone

technology.

3. The scheme is proposed as a Central Sector scheme.

Which of the statements given above is/are correct?

(a) 1 and 2 only

(b) 2 and 3 only

(c) 1 and 3 only

(d) 1, 2 and 3

Ans: (b)

Explanation: Overview and key features of the SVAMITVA (Survey of Villages and

Mapping with Improvised Technology in Village Areas) Yojana’:

• Launched on Panchayati Raj Diwas (April 24th, 2020). Last year, it was

extended to all states. Earlier, it was launched only for 9 states.

• The scheme seeks to map residential land ownership in the rural

sector using modern technology like the use of drones.

• The scheme aims to revolutionize property record maintenance in India.

• The scheme is piloted by the Panchayati Raj ministry.

• Under the scheme, residential land in villages will be measured using drones to

create a non-disputable record.

• Property card for every property in the village will be prepared by states

using accurate measurements delivered by drone-mapping. These cards will be

given to property owners and will be recognised by the land revenue records

department.

• The scheme is proposed as a Central Sector scheme.

Refer: https://www.insightsonindia.com/2022/02/16/swamitva-scheme-2/

134. Consider the following statements about Medaram Jatara:

1. It is the second-largest fair of India.

2. It is celebrated in the state of Telangana.

3. It is celebrated once in two years in the month of “Vaisakha”.

Which of the statements given above is/are correct?

(a) 1 and 2 only

(b) 2 and 3 only

(c) 1 and 3 only

(d) 1, 2 and 3

Revision Through MCQs (RTM) Compilation (Feb - 2022)

Telegram: https://t.me/insightsIAStips Youtube: https://www.youtube.com/channel/UCpoccbCX9GEIwaiIe4HLjwA

87

Ans: (a)

Explanation:

• Medaram Jatara is the second-largest fair of India, after the Kumbh Mela,

celebrated by the second-largest Tribal Community of Telangana- the Koya tribe

for four days.

• Considering the footfall of the festival and its auspicious significance, the

Jathara was declared a State Festival in 1996.

• The Jatara is also known as Sammakka Saralamma Jatara.

• It is a tribal festival honoring the fight of a mother and

daughter, Sammakka and Saralamma, with the reigning rulers against an

unjust law.

• Medaram is a remote place in the Eturnagaram Wildlife Sanctuary, a part

of Dandakaranya, the largest surviving forest belt in the region.

• It is celebrated once in two years in the month of “Magha” on the full moon

day.

Refer: facts for prelims: https://www.insightsonindia.com/2022/02/16/mission-2022-

insights-daily-current-affairs-pib-summary-16-february-2022/

135. Recently, the Supreme Court of India has held that the government aid to an

institution is:

(a) a fundamental right

(b) not a fundamental right

Revision Through MCQs (RTM) Compilation (Feb - 2022)

Telegram: https://t.me/insightsIAStips Youtube: https://www.youtube.com/channel/UCpoccbCX9GEIwaiIe4HLjwA

88

(c) extraordinary right

(d) none of the above

Ans: (b)

Explanation:

• The Supreme Court has held that the government aid to an institution is a

matter of policy and it is not a fundamental right.

• Important observations made by the Court:

o Grant of aid brings with it conditions which the institution receiving it is

bound to comply with. If an institution does not want to accept the

conditions, it can decline the grant but cannot say that the grant must be

on its own terms.

o A decision to grant aid is by way of policy. While doing so, the

government is not only concerned with the interest of the institutions but

the ability to undertake such an exercise.

o As regards aided institutions, there cannot be any difference between a

minority and non-minority one. Article 30 of the Constitution of India is

subject to its own restrictions being reasonable.

Refer: https://www.insightsonindia.com/2021/09/28/right-to-get-aid-from-govt-not-

fundamental-right/

136. Formosa Strait separates:

(a) Spain from Morocco

(b) Australia from New Zealand

(c) China from Taiwan

(d) Indonesia from Singapore

Ans: (c)

Explanation:

• The Taiwan Strait is a 110-mile-wide channel that separates mainland China

from the island of Taiwan.

• It is also known as the Formosa Strait or the Tai-hai (the Tai Sea).

• The Taiwan Strait makes up part of the South China Sea, and its northern

portion is linked to the East China Sea.

• The strait borders the south eastern part of China and runs along the eastern

part of China’s Fujian Province.

Revision Through MCQs (RTM) Compilation (Feb - 2022)

Telegram: https://t.me/insightsIAStips Youtube: https://www.youtube.com/channel/UCpoccbCX9GEIwaiIe4HLjwA

89

Refer: facts for prelims: https://www.insightsonindia.com/2021/09/28/mission-2022-

insights-daily-current-affairs-pib-summary-28-september-2021-2/

137. What is the correct sequence of occurrence of the following cities in Central Asia as

one proceeds from North to South?

1. Bishkek

2. Kabul

3. Ashgabat

4. Astana

Select the correct answer using the code below:

(a) 4-1-2-3

(b) 4-1-3-2

(c) 1-4-2-3

(d) 1-4-3-2

Ans: (b)

Explanation:

Revision Through MCQs (RTM) Compilation (Feb - 2022)

Telegram: https://t.me/insightsIAStips Youtube: https://www.youtube.com/channel/UCpoccbCX9GEIwaiIe4HLjwA

90

Refer: world map

138. Consider the following pairs:

Tributary river Main River

1. Dhauliganga Yamuna River

2. Banas Chambal River

3. Indravati Godavari River

Which of the above pairs is/are correctly matched?

(a) 1 and 2 only

(b) 2 and 3 only

(c) 3 only

(d) 1, 2 and 3

Ans: (b)

Explanation:

• S1: The Dhauliganga is a turbulent Himalayan river which rises in the border regions of

India and China and flows south into the Garhwal region of Uttarakhand, India. It joins

the Alaknanda, the major source stream of the Ganges river.

• S2: The Banas is a river which lies entirely within the state of Rajasthan in western

India. It is a tributary of the Chambal River, itself a tributary of the Yamuna, which

in turn merges into the Ganga.

• S3: Indravati River is a tributary of the Godavari River, in central India.

Revision Through MCQs (RTM) Compilation (Feb - 2022)

Telegram: https://t.me/insightsIAStips Youtube: https://www.youtube.com/channel/UCpoccbCX9GEIwaiIe4HLjwA

91

Refer: Major/minor River of India.

139. The Global Risk Report is published by which of the following?

(a) Financial Action Task Force

(b) United Nations Security Council

(c) International Criminal Police Organization

(d) World Economic Forum

Ans: (d)

Explanation:

• The 16th edition of the Global Risks Report, published by the World

Economic Forum with support from Marsh McLennan, highlights the disruptive

implications of major risks, including the COVID-19 pandemic that may reshape

our world in 2021 and over the next decade.

• The report draws on the survey results from nearly 700 experts and decision-

makers globally who were asked about their concerns for the next decade, how

global risks interact and where opportunities exist to collectively act to mitigate

these threats.

Refer: https://www.insightsonindia.com/2021/02/09/insights-into-editorial-towards-

sustainable-growth/

140. Which of the following has/have been accorded ‘Geographical Indication’ status?

1. Kashmir Saffron

2. Pawndum

3. Gulbarga Tur Dal

4. Khola Chilli

5. Kaji Nemu

Select the correct answer using the code below:

(a) 1, 2, 4 and 5 only

(b) 2, 3, 4 and 5 only

(c) 4 and 5 only

(d) All of the above

Ans: (d)

Explanation: List of Geographical Indications [GI Tags] in India

• Kaji Nemu : Assam

• Kashmir Saffron: Jammu & Kashmir

• Pawndum: Mizoram

• Gulbarga Tur Dal: Karnataka

• Khola: Chilli Goa

Revision Through MCQs (RTM) Compilation (Feb - 2022)

Telegram: https://t.me/insightsIAStips Youtube: https://www.youtube.com/channel/UCpoccbCX9GEIwaiIe4HLjwA

92

Refer: https://en.wikipedia.org/wiki/List_of_geographical_indications_in_India

RTM- REVISION THROUGH MCQS –17th-Feb-2022

141. Consider the following statements:

1. Every Census in independent India has published data on Scheduled Castes and

Scheduled Tribes, but not on other castes.

2. Socio Economic and Caste Census 2011 was the first caste-based census since

1931 Census of India.

Which of the statements given above is/are correct?

(a) 1 only

(b) 2 only

(c) Both 1 and 2

(d) Neither 1 nor 2

Ans: (c)

Explanation:

• S1: Every Census in independent India from 1951 to 2011 has published data

on Scheduled Castes and Scheduled Tribes, but not on other castes. Before that,

every Census until 1931 had data on caste.

• S2: SECC 2011 was the first caste-based census since 1931 Census of India,

and it was launched on 29 June 2011 from the Sankhola village of Hazemara

block in West Tripura district.

Refer: https://www.insightsonindia.com/2022/02/17/caste-census-3/

142. Consider the following statements about North Atlantic Treaty Organization:

1. It is an intergovernmental military alliance.

2. It was established by Washington treaty.

3. Its headquarters is located in Geneva, Switzerland.

Which of the statements given above is/are correct?

(a) 1 and 2 only

(b) 2 and 3 only

(c) 1 and 3 only

(d) 1, 2 and 3

Ans: (a)

Explanation:

• It is an intergovernmental military alliance.

Revision Through MCQs (RTM) Compilation (Feb - 2022)

Telegram: https://t.me/insightsIAStips Youtube: https://www.youtube.com/channel/UCpoccbCX9GEIwaiIe4HLjwA

93

• Established by Washington treaty.

• Treaty that was signed on 4 April 1949.

• Headquarters — Brussels, Belgium.

• Headquarters of Allied Command Operations — Mons, Belgium.

Refer: https://www.insightsonindia.com/2022/02/17/north-atlantic-treaty-organization-

3/

143. Which one of the following nation is not the part of AUKUS Pact?

(a) Australia

(b) Argentina

(c) United States

(d) United Kingdom

Ans: (b)

Explanation:

• The UK, US and Australia, in 2021, announced a historic security pact in the

Asia-Pacific, in what’s seen as an effort to counter China. It is called the

AUKUS pact and AUKUS alliance.

• Under the AUKUS alliance, the three nations have agreed to enhance the

development of joint capabilities and technology sharing, foster deeper

integration of security and defence-related science, technology, industrial bases

and supply chains.

• Under the first major initiative of AUKUS, Australia would build a fleet of

nuclear-powered submarines with the help of the US and the UK, a

capability aimed at promoting stability in the Indo-Pacific region.

Refer: https://www.insightsonindia.com/2022/02/17/aukus-alliance-2/

144. Consider the following statements about the World Sustainable Development Summit:

1. It is the annual flagship event of the Tata Institute of Fundamental Research (TIFR).

2. It provides a platform for global leaders and practitioners to discuss over climatic

issues of universal importance.

Which of the statements given above is/are correct?

(a) 1 only

(b) 2 only

(c) Both 1 and 2

(d) Neither 1 nor 2

Ans: (b)

Explanation: About the World Sustainable Development Summit:

Revision Through MCQs (RTM) Compilation (Feb - 2022)

Telegram: https://t.me/insightsIAStips Youtube: https://www.youtube.com/channel/UCpoccbCX9GEIwaiIe4HLjwA

94

• It is the annual flagship event of The Energy and Resources Institute (TERI).

• It is the sole Summit on global issues taking place in the developing world.

• It provides a platform for global leaders and practitioners to discuss and

deliberate over climatic issues of universal importance.

• It strives to provide long-term solutions for the benefit of the global community

by assembling the world’s most enlightened leaders and thinkers on a single

platform.

Refer: https://www.insightsonindia.com/2022/02/17/world-sustainable-development-

summit-4/

145. What is ‘green hydrogen’, sometimes mentioned in the news?

(a) Hydrogen generated through electrolysis powered by nuclear energy.

(b) Hydrogen generated through electrolysis with solar power.

(c) Hydrogen created from fossil sources, where the carbon emissions are captured and

stored.

(d) Hydrogen produced by electrolysis using renewable energy.

Ans: (d)

Explanation:

• Hydrogen when produced by electrolysis using renewable energy is known

as Green Hydrogen which has no carbon footprint.

• Yellow hydrogen is a relatively new phrase for hydrogen made through

electrolysis using solar power.

• Pink hydrogen is generated through electrolysis powered by nuclear energy.

• Grey Hydrogen is hydrogen produced using fossil fuels such as natural gas.

• Blue hydrogen is created from fossil sources, where the carbon emissions are

captured and stored.

Refer: https://www.insightsonindia.com/2022/02/17/india-and-the-green-hydrogen-

potential/

146. Consider the following states:

1. Arunachal Pradesh

2. Mizoram

3. Manipur

4. Meghalaya

With reference to the States mentioned above, in terms of percentage of forest cover to

the total geographical area, which one of the following is the correct descending order?

Revision Through MCQs (RTM) Compilation (Feb - 2022)

Telegram: https://t.me/insightsIAStips Youtube: https://www.youtube.com/channel/UCpoccbCX9GEIwaiIe4HLjwA

95

(a) 1-2-4-3

(b) 2-1-4-3

(c) 1-2-3-4

(d) 2-1-3-4

Ans: (b)

Explanation: descending 📉

• In terms of forest cover as percentage of total geographical area, the top five

States are Mizoram, Arunachal Pradesh, Meghalaya, Manipur and Nagaland.

• The states that have shown the highest increase in forest cover are Telangana

(3.07%), Andhra Pradesh (2.22%) and Odisha (1.04%).

• Five states in the Northeast – Arunachal Pradesh, Manipur, Meghalaya, Mizoram

and Nagaland have all shown loss in forest cover.

• Madhya Pradesh has the largest forest cover in the country followed by

Arunachal Pradesh, Chhattisgarh, Odisha and Maharashtra.

Refer: https://fsi.nic.in/isfr-2021/chapter-2.pdf

147. As per India State of Forest Report-2021, which one of the following Tiger Reserve has

the highest forest cover?

(a) Kawal Tiger Reserve

(b) Buxa Tiger Reserve

(c) Pakke Tiger Reserve

(d) Anamalai Tiger Reserve

Ans: (c)

Explanation:

• In the present ISFR 2021, FSI has included a new chapter related to the

assessment of forest cover in the Tiger Reserves, Corridors and Lion

conservation area of India.

• Pakke Tiger Reserve in Arunachal Pradesh has the highest forest cover, at

nearly 97%.

Refer: https://fsi.nic.in/forest-report-2021-details

148. Which of the following pairs (National Parks/Tiger reserve: State) is not correctly

matched?

(a) Kawal Tiger Reserve: Madhya Pradesh

(b) Buxa Tiger Reserve: West Bengal

(c) Anamalai Tiger Reserve: Tamil Nadu

(d) Indravati Tiger Reserve: Chhattisgarh

Revision Through MCQs (RTM) Compilation (Feb - 2022)

Telegram: https://t.me/insightsIAStips Youtube: https://www.youtube.com/channel/UCpoccbCX9GEIwaiIe4HLjwA

96

Ans: (a)

Explanation:

• Buxa (West Bengal), Anamalai (Tamil Nadu) and Indravati reserves

(Chhattisgarh) have shown an increase in forest cover.

• The highest losses have been found in Kawal (Telangana), Bhadra (Karnataka)

and the Sundarbans reserves (West Bengal).

Refer: https://fsi.nic.in/forest-report-2021-details

149. The sanctuary takes its name from the River. Popularly known as Muthodi Wildlife

Sanctuary, after the village on its periphery, it was declared a Project Tiger reserve. But

apart from the tiger, it is a great place to sight and observe other mammals, reptiles,

and more than 250 species of birds, many of which are endemic to the Western Ghats.

The above paragraph refers to which of the following protected site?

(a) Bhandipur National Park and Tiger Reserve

(b) Nagarhole National Park

(c) Bhadra Wildlife Sanctuary and Tiger Reserve

(d) Eravikulam National Park

Ans: (c)

Explanation: Bhadra Wildlife Sanctuary and Tiger Reserve:

• The sanctuary takes its name from the Bhadra River, its lifeline. Popularly

known as Muthodi Wildlife Sanctuary, after the village on its periphery, it was

declared a Project Tiger reserve.

• Refer: https://en.wikipedia.org/wiki/Bhadra_Wildlife_Sanctuary

Refer: https://fsi.nic.in/forest-report-2021-details

150. With reference to India’s Gir National Park, consider the following statements:

1. It is the only known habitat of the Asiatic Lion.

2. It is part of the Khathiar-Gir dry deciduous forests ecosystem.

3. It was also the location of the Indian Hangul Conservation Project.

Which of the statements given above is/are correct?

(a) 1 and 2 only

(b) 2 and 3 only

(c) 1 and 3 only

(d) 1, 2 and 3

Ans: (a)

Explanation:

Revision Through MCQs (RTM) Compilation (Feb - 2022)

Telegram: https://t.me/insightsIAStips Youtube: https://www.youtube.com/channel/UCpoccbCX9GEIwaiIe4HLjwA

97

• S1: The Gir National Park and Wildlife Sanctuary (also called Sasan Gir) is a

wildlife sanctuary and forest located at Talala Gir, Gujarat. It is the only known

habitat of the Asiatic Lion.

• S2: It is part of the Khathiar-Gir dry deciduous forests ecoregion

• S3: Gir has been used by the Gujarat State Forest Department which formed the

Indian Crocodile Conservation Project in 1977 and released close to 1000

marsh crocodiles into Lake Kamaleshwar and other small bodies of water in and

around Gir.

o The Kashmir stag also called hangul, is a subspecies of Central Asian

red deer endemic to Kashmir, India. It is found in dense riverine forests

in the high valleys and mountains of the Kashmir Valley and northern

Chamba district in Himachal Pradesh.

Refer: https://fsi.nic.in/forest-report-2021-details

Revision Through MCQs (RTM) Compilation (Feb - 2022)

Telegram: https://t.me/insightsIAStips Youtube: https://www.youtube.com/channel/UCpoccbCX9GEIwaiIe4HLjwA

98

Revision Through MCQs (RTM) Compilation (Feb - 2022)

Telegram: https://t.me/insightsIAStips Youtube: https://www.youtube.com/channel/UCpoccbCX9GEIwaiIe4HLjwA

99

RTM- REVISION THROUGH MCQS –18th-Feb-2022

151. Consider the following statements regarding National Human Rights Commission.

1. It was established by a Government resolution, for the protection and promotion of

human rights.

2. It has its own investigating staff for investigation into complaints of human rights

violations.

3. It has no power to punish the violators of human rights.

Which of the above statements is/are correct?

(a) 1 and 2 only

(b) 1 and 3 only

(c) 2 and 3 only

(d) 1, 2 and 3

Ans: (c)

Explanation:

• NHRC, established in 1993, is an independent statutory body as per the

provisions of the Protection of Human Rights Act of 1993.

• The Commission has its own investigating staff headed by a Director

General of Police for investigation into complaints of human rights violations.

• While inquiring into complaints under the Act, the Commission shall have all the

powers of a civil court trying a suit under the Code of Civil Procedure, 1908.

• It has no power to punish the violators of human rights. It has no power to

award any monetary relief.

Refer: https://www.insightsonindia.com/2022/02/18/draft-law-on-refugee-rights/

152. Consider the following statements:

1. United Nations Refugee Convention 1951 defines who is a refugee, and sets out the

rights of individuals who are granted asylum and the responsibilities of nations that

grant asylum.

2. India is not a party to the 1951 Refugee Convention.

Which of the statements given above is/are correct?

(a) 1 only

(b) 2 only

(c) Both 1 and 2

(d) Neither 1 nor 2

Ans: (c)

Revision Through MCQs (RTM) Compilation (Feb - 2022)

Telegram: https://t.me/insightsIAStips Youtube: https://www.youtube.com/channel/UCpoccbCX9GEIwaiIe4HLjwA

100

Explanation: About the Refugee Convention 1951:

• It is a United Nations multilateral treaty that defines who is a refugee, and sets

out the rights of individuals who are granted asylum and the responsibilities of

nations that grant asylum.

• The Convention also sets out which people do not qualify as refugees, such as

war criminals. The Convention also provides for some visa-free travel for holders

of travel documents issued under the convention.

• The 1967 Protocol included refugees from all countries as opposed to the 1951

Convention that only included refugees from Europe.

• India is not a party to the 1951 Refugee Convention and its 1967 Protocol,

the key legal documents pertaining to refugee protection.

Refer: https://www.insightsonindia.com/2022/02/18/draft-law-on-refugee-rights/

153. Which one of the following article in the Indian Constitution explicitly prohibits and

criminalises human trafficking and forced labour?

(a) Article 16

(b) Article 17

(c) Article 21

(d) Article 23

Ans: (d)

Explanation:

• Article 23 of the Indian Constitution explicitly prohibits and criminalises

human trafficking and forced labour.

• What are the constitutional & legislative provisions related to Trafficking

in India?

o Trafficking in Human Beings or Persons is prohibited under the

Constitution of India under Article 23 (1).

o The Immoral Traffic (Prevention) Act, 1956 (ITPA) is the premier

legislation for prevention of trafficking for commercial sexual

exploitation.

o Criminal Law (amendment) Act 2013 has come into force wherein

Section 370 of the Indian Penal Code has been substituted with Section

370 and 370A IPC which provide for comprehensive measures to counter

the menace of human trafficking.

Refer: https://www.insightsonindia.com/2022/02/18/human-trafficking/

154. Which one of the following nation is the full member of G20?

(a) Singapore

Revision Through MCQs (RTM) Compilation (Feb - 2022)

Telegram: https://t.me/insightsIAStips Youtube: https://www.youtube.com/channel/UCpoccbCX9GEIwaiIe4HLjwA

101

(b) Mexico

(c) Chile

(d) Malaysia

Ans: (b)

Explanation:

• Full membership of the G20:

• Argentina, Australia, Brazil, Canada, China, France, Germany, India, Indonesia,

Italy, Japan, Mexico, Russia, Saudi Arabia, South Africa, South Korea, Turkey,

the United Kingdom, the United States and the European Union.

Refer: https://www.insightsonindia.com/2022/02/18/g20-summit-2/

155. Consider the following statements about G20 or Group of Twenty:

1. It is an intergovernmental forum comprising 19 countries and the European Union

(EU).

2. It was founded in 2008 in response to several world economic crises.

3. At its 2009 summit, the G20 declared itself the primary venue for international

economic and financial cooperation.

Which of the statements given above is/are correct?

(a) 1 and 2 only

(b) 2 and 3 only

(c) 1 and 3 only

(d) 1, 2 and 3

Ans: (c)

Explanation:

• S1: The G20 or Group of Twenty is an intergovernmental forum comprising 19

countries and the European Union (EU).

• S2: The G20 was founded in 1999 in response to several world economic crises.

• S3: At its 2009 summit, the G20 declared itself the primary venue for

international economic and financial cooperation.

Refer: https://www.insightsonindia.com/2022/02/18/g20-summit-2/

156. Consider the following statements regarding the India-based Neutrino Observatory

(INO):

1. It is jointly steered by the Ministry of Electronics and IT (MeitY) and Department of

Science and Technology (DST).

2. The project includes the construction of an underground laboratory and associated

surface facilities at Pottipuram in Bodi West hills of Tamil Nadu.

Revision Through MCQs (RTM) Compilation (Feb - 2022)

Telegram: https://t.me/insightsIAStips Youtube: https://www.youtube.com/channel/UCpoccbCX9GEIwaiIe4HLjwA

102

Which of the statements given above is/are correct?

(a) 1 only

(b) 2 only

(c) Both 1 and 2

(d) Neither 1 nor 2

Ans: (b)

Explanation:

• The India-based Neutrino Observatory (INO) Project is a multi-institutional

effort aimed at building a world-class underground laboratory with a rock

cover of approx.1200 m for non-accelerator based high energy and nuclear

physics research in India. The initial goal of INO is to study neutrinos.

• It is a mega-science project jointly funded by the Department of Atomic

Energy (DAE) and the Department of Science and Technology (DST).

• The project proposal includes:

o Construction of an underground laboratory and associated surface

facilities at Pottipuram in Bodi West hills of Theni District of Tamil

Nadu.

o Construction of an Iron Calorimeter (ICAL) detector for studying

neutrinos.

o Setting up of National Centre for High Energy Physics at Madurai, for

the operation and maintenance of the underground laboratory, human

resource development and detector R&D along with its applications.

Refer: https://www.insightsonindia.com/2022/02/18/neutrino-project-3/

157. Consider the following statements:

1. Mali is a landlocked country in West Africa.

2. It is located southwest of Algeria.

Which of the statements given above is/are correct?

(a) 1 only

(b) 2 only

(c) Both 1 and 2

(d) Neither 1 nor 2

Ans: (c)

Explanation:

• Mali is a landlocked country in West Africa, located southwest of Algeria.

Revision Through MCQs (RTM) Compilation (Feb - 2022)

Telegram: https://t.me/insightsIAStips Youtube: https://www.youtube.com/channel/UCpoccbCX9GEIwaiIe4HLjwA

103

• Mali borders Algeria to the north-northeast, Niger to the east, Burkina Faso to

the south-east, Ivory Coast to the south, Guinea to the south-west, and Senegal

to the west and Mauritania to the north-west.

Refer: facts for prelims: https://www.insightsonindia.com/2022/02/18/mission-2022-

insights-daily-current-affairs-pib-summary-18-february-2022/

158. India’s first water taxi service was recently inaugurated in

(a) Kerala

(b) Gujarat

(c) Maharashtra

(d) Tamil Nadu

Ans: (c)

Explanation:

• India’s first water taxi service was recently inaugurated in Maharashtra.

• It will connect the Navi Mumbai area to mainland Mumbai.

• The ₹8.37–crore project will presently run on three routes and the State and the

Centre have each shared 50% of the expenditure.

• The three routes include Belapur to Ferry Wharf – the domestic cruise terminal,

Belapur to Elephanta Caves and Belapur to JNPT.

Refer: facts for prelims: https://www.insightsonindia.com/2022/02/18/mission-2022-

insights-daily-current-affairs-pib-summary-18-february-2022/

Revision Through MCQs (RTM) Compilation (Feb - 2022)

Telegram: https://t.me/insightsIAStips Youtube: https://www.youtube.com/channel/UCpoccbCX9GEIwaiIe4HLjwA

104

159. Consider the following states:

1. Arunachal Pradesh

2. Mizoram

3. Manipur

4. Meghalaya

With reference to the States mentioned above, in terms of percentage of forest cover to

the total geographical area, which one of the following is the correct descending order?

(a) 1-2-4-3

(b) 2-1-4-3

(c) 1-2-3-4

(d) 2-1-3-4

Ans: (b)

Explanation: descending 📉

• In terms of forest cover as percentage of total geographical area, the top five

States are Mizoram, Arunachal Pradesh, Meghalaya, Manipur and Nagaland.

• The states that have shown the highest increase in forest cover are Telangana

(3.07%), Andhra Pradesh (2.22%) and Odisha (1.04%).

• Five states in the Northeast – Arunachal Pradesh, Manipur, Meghalaya, Mizoram

and Nagaland have all shown loss in forest cover.

• Madhya Pradesh has the largest forest cover in the country followed by

Arunachal Pradesh, Chhattisgarh, Odisha and Maharashtra.

Refer: https://fsi.nic.in/isfr-2021/chapter-2.pdf

160. With reference to India’s Nagarjunsagar-Srisailam Tiger Reserve, consider the

following statements:

1. The River Godavari forms northern most boundary of the Tiger Reserve.

2. The reserve primarily comprises of southern tropical dry mixed deciduous forest.

3. The native population of the reserve includes the tribes of Chenchus.

Which of the given above statements is/are correct?

(a) 1 and 3 only

(b) 2 only

(c) 2 and 3 only

(d) 1, 2 and 3

Ans: (c)

Explanation:

• S1: River Krishna forms northern most boundary of the Tiger Reserve.

Revision Through MCQs (RTM) Compilation (Feb - 2022)

Telegram: https://t.me/insightsIAStips Youtube: https://www.youtube.com/channel/UCpoccbCX9GEIwaiIe4HLjwA

105

• S2: The reserve primarily comprises of southern tropical dry mixed deciduous

forest

• S3: The native population of the reserve includes the tribes of

Chenchus. https://www.thehindu.com/news/national/andhra-

pradesh/hidden-chenchus-and-crouching-tigers-of-

nallamala/article28763206.ece

Refer: https://en.wikipedia.org/wiki/Nagarjunsagar-Srisailam_Tiger_Reserve

RTM- REVISION THROUGH MCQS –21st-Feb-2022

161. Consider the following statements regarding Guru Ravidas

1. His devotional songs were included in the Guru Granth Sahib.

2. He taught removal of social divisions of caste and gender, and promoted unity in the

pursuit of personal spiritual freedom.

Which of the statements given above is/are correct?

(a) 1 only

(b) 2 only

(c) Both 1 and 2

(d) Neither 1 nor 2

Ans: (c)

Explanation:

• He was a poet and saint of the Bhakti movement.

• Born in Varanasi to the ‘untouchable’ leather tanners caste.

• He is believed to be a disciple of the bhakti saint-poet Ramanandaand a

contemporary of the bhakti saint-poet Kabir.

• One of his famous disciples was the saint, Mirabai.

• Ravidas’ devotional songs were included in the Sikh scriptures, Guru Granth

Sahib.

• The Panch Vani text of the Dadupanthi tradition within Hinduism also includes

numerous poems of Ravidas.

• Ravidas taught removal of social divisions of caste and gender, and promoted

unity in the pursuit of personal spiritual freedoms.

• Among Ravidas’s moral and intellectual achievements were the conception of

“Begampura”, a city that knows no sorrow; and a society where caste and class

have ceased to matter.

Revision Through MCQs (RTM) Compilation (Feb - 2022)

Telegram: https://t.me/insightsIAStips Youtube: https://www.youtube.com/channel/UCpoccbCX9GEIwaiIe4HLjwA

106

Refer: https://www.insightsonindia.com/2022/02/21/guru-ravidas-5/

162. Consider the following statements about Pradhan Mantri Fasal Bima Yojana:

1. It was launched by Ministry of Agriculture & Farmers welfare, Govt. India.

2. To address the demand of farmers, the scheme has been made voluntary for all

farmers from Kharif 2020.

Which of the statements given above is/are correct?

(a) 1 only

(b) 2 only

(c) Both 1 and 2

(d) Neither 1 nor 2

Ans: (c)

Explanation:

• Pradhan Mantri Fasal Bima Yojana (PMFBY) scheme was launched in India by

Ministry of Agriculture & Farmers welfare, New Delhi from Kharif 2016 season

onwards.

• All farmers growing notified crops in a notified area during the season who have

insurable interest in the crop are eligible.

• To address the demand of farmers, the scheme has been made voluntary for

all farmers from Kharif 2020.

o Earlier to Kharif 2020, the enrollment under the scheme was compulsory

for following categories of farmers:

o Farmers in the notified area who possess a Crop Loan account/KCC

account (called as Loanee Farmers) to whom credit limit is

sanctioned/renewed for the notified crop during the crop season. and

o Such other farmers whom the Government may decide to include from

time to time.

• Voluntary coverage : Voluntary coverage may be obtained by all farmers not

covered above, including Crop KCC/Crop Loan Account holders whose credit

limit is not renewed.

Refer: https://www.insightsonindia.com/2022/02/21/pradhan-mantri-fasal-bima-yojana-

2/

163. The concept of ‘Extended Producer Responsibility (EPR)’ is often heard in the context

of

(a) Renewable Energy generation and transmission

(b) Plastic Waste Management

(c) Genetic Modification of plant species

Revision Through MCQs (RTM) Compilation (Feb - 2022)

Telegram: https://t.me/insightsIAStips Youtube: https://www.youtube.com/channel/UCpoccbCX9GEIwaiIe4HLjwA

107

(d) Intellectual Property Rights

Ans: (b)

Explanation:

• Extended producer responsibility (EPR) under the Plastic Waste

Management Rules of 2016, which sought to pin the responsibility of plastic

waste on the producers. It is a tool to make producer companies become

conscious of the environmental consequences of their production systems and

products, and push them to start cleaning up their act.

Refer: https://www.insightsonindia.com/2022/02/21/extended-producers-responsibility-

on-plastic-packaging/

164. The United Arab Emirates shares border with:

1. Saudi Arabia

2. Bahrain

3. Oman

Select the correct answer using the code below:

(a) 1 and 2 only

(b) 2 and 3 only

(c) 1 and 3 only

(d) 1, 2 and 3

Ans: (c)

Explanation:

Refer: https://www.insightsonindia.com/2022/02/21/india-uae-sign-comprehensive-

trade-agreement/

Revision Through MCQs (RTM) Compilation (Feb - 2022)

Telegram: https://t.me/insightsIAStips Youtube: https://www.youtube.com/channel/UCpoccbCX9GEIwaiIe4HLjwA

108

165. The Frontiers Report 2022 is released by which of the following?

(a) United Nations Environment Programme

(b) Intergovernmental Panel on Climate Change

(c) International Union for Conservation of Nature

(d) Reporters Without Borders

Ans: (a)

Explanation:

• The United Nations Environment Programme (UNEP) recently released its

latest annual Frontiers report.

• This is the fourth edition of the Frontiers Report, which was first published

in 2016 with an alert to the growing risk of zoonotic diseases, four years before

the outbreak of the COVID-19 pandemic.

• The Report identifies and offers solutions to three environmental issues that

merit attention and action from governments and the public at large.

Refer: https://www.insightsonindia.com/2022/02/21/environmental-conservation-un-

unep/

166. Consider the following statements:

1. The Inter-Operable Criminal Justice System (ICJS) is a national platform which

invloves integration of the main IT system used for delivery of Criminal Justice.

2. Department of Justice will be responsible for the implementation of the project in

association with the Centre for Development of Advanced Computing (C-DAC).

Which of the statements given above is/are correct?

(a) 1 only

(b) 2 only

(c) Both 1 and 2

(d) Neither 1 nor 2

Ans: (a)

Explanation:

• ICJS is a national platform which invloves integration of the main IT system

used for delivery of Criminal Justice in the country.

• National Crime Records Bureau (NCRB) will be responsible for the

implementation of the project in association with the National Informatics

Centre (NIC).

Refer: https://www.insightsonindia.com/2022/02/21/inter-operable-criminal-justice-

system-project/

Revision Through MCQs (RTM) Compilation (Feb - 2022)

Telegram: https://t.me/insightsIAStips Youtube: https://www.youtube.com/channel/UCpoccbCX9GEIwaiIe4HLjwA

109

167. Consider the following statements:

1. The Falkland Islands are a self-governing British Overseas Territory.

2. The islands are located in the South Pacific Ocean.

Which of the statements given above is/are correct?

(a) 1 only

(b) 2 only

(c) Both 1 and 2

(d) Neither 1 nor 2

Ans: (a)

Explanation:

• Falkland islands are an overseas territory of the United Kingdom located to

the southwest Atlantic Ocean at the southernmost point of South America.

• Positioned both in the southern and western hemispheres of the Earth.

• They are also called Malvinas Islands.

Refer: facts for prelims: https://www.insightsonindia.com/2022/02/21/mission-2022-

insights-daily-current-affairs-pib-summary-21-february-2022-2/

168. Why is a plant called “Withania somnifera” often mentioned in news?

(a) It tends to reduce the biodiversity in the area in which it grows.

(b) Its extract is widely used in traditional Indian medicine.

(c) Its extract is used in the pesticides.

(d) None of the above

Ans: (b)

Explanation:

Revision Through MCQs (RTM) Compilation (Feb - 2022)

Telegram: https://t.me/insightsIAStips Youtube: https://www.youtube.com/channel/UCpoccbCX9GEIwaiIe4HLjwA

110

• ‘Ashwagandha’ (Withania somnifera) is commonly known as ‘Indian winter

cherry’.

o It is a traditional Indian herb that boosts energy, reduces stress and

makes the immune system stronger.

o It is classified as an adaptogen, which means that it can help the body to

manage stress.

• Ashwagandha is widely grown in dry parts of subtropical regions. Rajasthan,

Punjab, Haryana, Uttar Pradesh, Gujarat, Maharashtra and Madhya Pradesh are

major ashwagandha growing states in India.

• Being a hardy and drought-tolerant crop, ashwagandha requires a relatively

dry season throughout its growing period. Areas with 60-75 cm rainfall are

suitable for its cultivation.

• Temperature between 20°C to 35°C is most suitable.

• It grows well in sandy loam or light red soils having pH 7.5-8.0. Black or heavy

soils having good drainage are also suitable for ashwagandha cultivation.

Refer: https://en.wikipedia.org/wiki/Withania_somnifera

169. With reference to India’s Khangchendzonga National Park, consider the following

statements:

1. It is a biodiversity hotspot in the Eastern Himalayas.

2. It is the fourth largest national park in India.

3. It is located in Sikkim, India.

Which of the given above statements is/are correct?

(a) 1 and 2 only

(b) 3 only

(c) 2 and 3 only

(d) 1 and 3 only

Ans: (d)

Explanation:

• S3: Khangchendzonga National Park also Kanchenjunga Biosphere Reserve

is a national park and a Biosphere reserve located in Sikkim, India.

o It was inscribed to the UNESCO World Heritage Sites list in July 2016,

becoming the first "Mixed Heritage" site of India.

• S2: Namdapha National Park is the fourth largest national park in India.

• S1: Eastern Himalayas form a part of the Himalayan global biodiversity

hotspot. This region is exceptionally rich in diversity and endemism. It

Revision Through MCQs (RTM) Compilation (Feb - 2022)

Telegram: https://t.me/insightsIAStips Youtube: https://www.youtube.com/channel/UCpoccbCX9GEIwaiIe4HLjwA

111

comprises of parts of Nepal, Bhutan, Sikkim, Arunanchal Pradesh and

extends up to Burma.

Refer: https://whc.unesco.org/en/list/1513/

170. Which among the following is often recognized as “Genetic Express Highway” linking

two biodiversity hot spots of the country, viz., Eastern Himalaya and Western Ghats?

(a) Nilgiri Biosphere Reserve

(b) Agasthyamala Biosphere Reserve

(c) Pachmarhi Biosphere Reserve

(d) Achanakmar-Amarkantak Biosphere Reserve

Ans: (c)

Explanation:

• Pachmarhi BR area is often recognized as “Genetic Express Highway” linking

two biological hot spots of the country viz. Eastern Himalayas (not Eastern

Ghats) and Western Ghats, also as confluence of northern and southern type of

vegetation.

Refer: www.unesco.org/new/en/natural-sciences/environment/ecological-

sciences/biosphere-reserves/asia-and-the-pacific/india/pachmarhi

Revision Through MCQs (RTM) Compilation (Feb - 2022)

Telegram: https://t.me/insightsIAStips Youtube: https://www.youtube.com/channel/UCpoccbCX9GEIwaiIe4HLjwA

112

RTM- REVISION THROUGH MCQS –22nd-Feb-2022

171. Which article of the Constitution of India prohibits traffic in human beings and

beggars and other similar forms of forced labor?

(a) Article 15

(b) Article 21

(c) Article 23

(d) Article 32

Ans: (c)

Explanation:

• Article 23(1) of the Constitution prohibits traffic in human beings and beggars

and other similar forms of forced labor. Article 23(2) declares that any

contravention of this provision shall be an offense punishable in accordance with

the law.

Refer: https://www.insightsonindia.com/2022/02/22/who-was-gangubai-kathiawadi/

172. Which article of the Constitution of India guarantees the freedom of conscience, the

freedom to profess, practice and propagate religion to all citizens?

(a) Article 15

(b) Article 21

(c) Article 25

(d) Article 27

Ans: (c)

Explanation:

• Article 25 guarantees the freedom of conscience, the freedom to profess,

practice and propagate religion to all citizens.

Refer: https://www.insightsonindia.com/2022/02/22/public-order-a-constitutional-

provision-for-curbing-freedoms/

173. Consider the following statements:

1. The Representation of the People Act, 1951 empowers the Election Commission of

India to recognise political parties and allot symbols.

2. The Election Commission of India is the only authority to decide issues on a dispute

or a merger of political parties in India.

Which of the statements given above is/are correct?

(a) 1 only

Revision Through MCQs (RTM) Compilation (Feb - 2022)

Telegram: https://t.me/insightsIAStips Youtube: https://www.youtube.com/channel/UCpoccbCX9GEIwaiIe4HLjwA

113

(b) 2 only

(c) Both 1 and 2

(d) Neither 1 nor 2

Ans: (b)

Explanation:

• The Election Symbols (Reservation and Allotment) Order, 1968 empowers the EC

to recognise political parties and allot symbols.

• Under Paragraph 15 of the Order, it can decide disputes among rival groups or

sections of a recognised political party staking claim to its name and symbol.

• The EC is also the only authority to decide issues on a dispute or a merger. The

Supreme Court upheld its validity in Sadiq Ali and another vs. ECI in 1971.

Refer: https://www.insightsonindia.com/2022/02/22/election-symbols/

174. Consider the following statements about Financial Action Task Force (FATF):

1. It is an inter-governmental body established in 1989 on the initiative of the G20.

2. Initially it was established to examine and develop measures to combat money

laundering.

3. Its secretariat is housed at the OECD headquarters in Paris.

Which of the statements given above is/are correct?

(a) 1 and 2 only

(b) 2 and 3 only

(c) 1 and 3 only

(d) 1, 2 and 3

Ans: (b)

Explanation: About FATF:

• The Financial Action Task Force (FATF) is an inter-governmental

body established in 1989 on the initiative of the G7.

• It is a “policy-making body” which works to generate the necessary political will

to bring about national legislative and regulatory reforms in various areas.

• The FATF Secretariat is housed at the OECD headquarters in Paris.

• Roles and functions:

o Initially it was established to examine and develop measures to

combat money laundering.

o In October 2001, the FATF expanded its mandate to incorporate efforts to

combat terrorist financing, in addition to money laundering.

o In April 2012, it added efforts to counter the financing of proliferation of

weapons of mass destruction.

Revision Through MCQs (RTM) Compilation (Feb - 2022)

Telegram: https://t.me/insightsIAStips Youtube: https://www.youtube.com/channel/UCpoccbCX9GEIwaiIe4HLjwA

114

Refer: https://www.insightsonindia.com/2022/02/22/fatf/

175. Consider the following statements about National Payments Corporation of India

(NPCI):

1. It is an initiative of Reserve Bank of India (RBI) and Indian Banks Association (IBA)

under the provisions of the Banking Regulation Act, 1949.

2. It has developed the National Electronic Toll Collection (NETC) program to meet the

electronic tolling requirements of the Indian market.

Which of the given above statements is/are correct?

(a) 1 only

(b) 2 only

(c) Both 1 and 2

(d) Neither 1 nor 2

Ans: (b)

Explanation:

• S1: National Payments Corporation of India (NPCI), an umbrella organisation

for operating retail payments and settlement systems in India, is an initiative of

Reserve Bank of India (RBI) and Indian Banks’ Association (IBA) under the

provisions of the Payment and Settlement Systems Act, 2007, for creating a

robust Payment & Settlement Infrastructure in India.

• S2: National Payments Corporation of India (NPCI) has developed the National

Electronic Toll Collection (NETC) program to meet the electronic tolling

requirements of the Indian market.

Refer: https://www.insightsonindia.com/2022/02/22/nepal-will-be-the-first-foreign-

country-to-adopt-indias-unified-payments-interface-upi-system/

176. Consider the following statements about Unified Payments Interface (UPI):

1. It is an instant real-time payment system developed by National Payments

Corporation of India facilitating inter-bank transactions.

2. It caters to the “Peer to Peer” collect request which can be scheduled and paid as per

requirement and convenience.

Which of the given above statements is/are correct?

(a) 1 only

(b) 2 only

(c) Both 1 and 2

(d) Neither 1 nor 2

Ans: (c)

Explanation:

Revision Through MCQs (RTM) Compilation (Feb - 2022)

Telegram: https://t.me/insightsIAStips Youtube: https://www.youtube.com/channel/UCpoccbCX9GEIwaiIe4HLjwA

115

• Unified Payments Interface (UPI) is an instant real-time payment system

developed by National Payments Corporation of India facilitating inter-bank

transactions. The interface is regulated by the Reserve Bank of India and

works by instantly transferring funds between two bank accounts on a mobile

platform.

• It also caters to the “Peer to Peer” collect request which can be scheduled and

paid as per requirement and convenience.

Refer: https://www.insightsonindia.com/2022/02/22/nepal-will-be-the-first-foreign-

country-to-adopt-indias-unified-payments-interface-upi-system/

177. With reference to Indian star tortoise, consider the following statements:

1. It is naturally found in India only.

2. It has been listed as “Vulnerable” by IUCN.

Which of the given above statements is/are correct?

(a) 1 only

(b) 2 only

(c) Both 1 and 2

(d) Neither 1 nor 2

Ans: (b)

Explanation: Indian star tortoise:

• Found across the Indian sub-continent, more specifically, in the Central and

Southern parts of India, in West Pakistan and in Sri Lanka.

• Protected under Schedule IV of Wild Life Protection Act 1972.

• Convention on International Trade inSpecies (CITES): Appendix I

• IUCN Status: Vulnerable.

Refer: https://science.thewire.in/environment/indian-star-tortoises-are-slowly-and-

steadily-losing-the-race-for-survival/

Revision Through MCQs (RTM) Compilation (Feb - 2022)

Telegram: https://t.me/insightsIAStips Youtube: https://www.youtube.com/channel/UCpoccbCX9GEIwaiIe4HLjwA

116

178. Which of the following statements is/are correct about Brow antlered deer?

1. It is found in Keibul Lamjao National Park.

2. It inhabits alpine forests and scrubland.

3. It is classified as Endangered on IUCN Red List.

Which of the above statements is/are correct?

(a) 1 and 2 only

(b) 1 and 3 only

(c) 2 and 3 only

(d) 1, 2 and 3

Ans: (b)

Explanation: Sangai, Brow antlered deer, Dancing Deer:

• Status: State animal of Manipur, Schedule-1 of Wildlife (Protection) Act, 1972,

Endangered on IUCN Red List.

• The brow-antlered deer is found in Keibul Lamjao National Park in Manipur.

It is largely seen over the floating biomass, locally called “phumdi” in the South

Eastern part of Loktak Lake inside the park. Phumdi is the most important and

unique part of Sangai’s habitat.

• Sangai faces threat from steadily degenerating habitat of phumdi as a result of

continuous inundation and flooding caused due to artificial reservoir. Water

quality of the reservoir is degrading due to pollution and stoppage of nutrient

supply. There is also invasion of non-native plants like Paragrass.

• Sangai also faces threats of diseases from the livestock, inbreeding depression

and poaching.

Refer: https://scroll.in/article/991380/in-manipur-the-endangered-sangai-deers-habitat-

is-facing-threat-from-climate-change

179. Consider the following statements:

1. Belarus is the only landlocked country in Eastern Europe.

2. It is bordered by Russia, Ukraine, Poland and Lithuania only.

Which of the given above statements is/are correct?

(a) 1 only

(b) 2 only

(c) Both 1 and 2

(d) Neither 1 nor 2

Ans: (d)

Explanation:

• It is a landlocked country in Eastern Europe.

Revision Through MCQs (RTM) Compilation (Feb - 2022)

Telegram: https://t.me/insightsIAStips Youtube: https://www.youtube.com/channel/UCpoccbCX9GEIwaiIe4HLjwA

117

• It is bordered by Russia to the east and northeast, Ukraine to the south, Poland

to the west, and Lithuania and Latvia to the northwest.

• Also Republic of Moldova is a landlocked country in Eastern Europe.

Refer: World Map

180. The phrase ‘Critical Wildlife Habitat’ (CWH) is defined under the:

(a) Wildlife (Protection) Act, 1972

(b) Environment Protection Act of 1986

(c) Biological Diversity Act, 2002

(d) Forest Rights Act, 2006

Ans: (d)

Explanation:

• The phrase ‘critical wildlife habitat’ is defined only in the Scheduled Tribes and

Other Traditional Forest Dwellers (Recognition of Forest Rights) Act, 2006,

and NOT in the Wildlife (Protection) Act, 1972.

o Critical wildlife habitats (CWHs) are areas inside wildlife sanctuaries,

national parks and tiger reserves – known as 'protected areas' – where

people's activities like cattle grazing or collecting leaves compete with the

needs of wildlife.

Refer: https://www.downtoearth.org.in/news/wildlife-biodiversity/report-flags-violations-

in-creation-of-india-s-first-critical-wildlife-habitats-73249

RTM- REVISION THROUGH MCQS –23nd-Feb-2022

181. Consider the following statements:

1. Krishna river originates in the Western Ghats.

2. It flows through the state of Andhra Pradesh before entering Telangana state.

Revision Through MCQs (RTM) Compilation (Feb - 2022)

Telegram: https://t.me/insightsIAStips Youtube: https://www.youtube.com/channel/UCpoccbCX9GEIwaiIe4HLjwA

118

Which of the statements given above is/are correct?

(a) 1 only

(b) 2 only

(c) Both 1 and 2

(d) Neither 1 nor 2

Ans: (a)

Explanation:

• The Krishna river originates in the Western Ghats near Mahabaleshwar at an

elevation of about 1,300 metres (4,300 ft), in the state of Maharashtra in central

India.

• ’

Refer: https://www.insightsonindia.com/2022/02/23/krishna-river-water-dispute-3/

182. Consider the following statements about Academic Bank of Credit (ABC):

1. It is envisaged as a digital bank that holds the credit earned by a student in any

course.

2. It was proposed under the National Education Policy (NEP) 2020.

Which of the statements given above is/are correct?

(a) 1 only

(b) 2 only

(c) Both 1 and 2

(d) Neither 1 nor 2

Ans: (c)

Revision Through MCQs (RTM) Compilation (Feb - 2022)

Telegram: https://t.me/insightsIAStips Youtube: https://www.youtube.com/channel/UCpoccbCX9GEIwaiIe4HLjwA

119

Explanation:

• Academic Bank of Credit (ABC), proposed under the National Education

Policy (NEP) 2020, was unveiled in July 2021.

• Set-up by the University Grants Commission (UGC).

• Under the ABC, students will be given multiple entry and exit options.

• This enables students to leave a degree or course and get a corresponding

certification and rejoin studies after a certain time and be able to start from

where they had left.

• It will also provide students with the flexibility to move between institutes while

pursuing one degree or leave a course.

Refer: https://www.insightsonindia.com/2022/02/23/academic-bank-of-credit-3/

183. Consider the following statements about the Indus Water Treaty:

1. It is a water-distribution treaty between India and Pakistan.

2. It was brokered by the UN-Water.

Which of the statements given above is/are correct?

(a) 1 only

(b) 2 only

(c) Both 1 and 2

(d) Neither 1 nor 2

Ans: (a)

Explanation:

• It is a Water-Distribution Treaty, signed in Karachi on 1960, between India (Pm

Jawaharlal Nehru) and Pakistan (President Ayub Khan), brokered by the World

Bank.

• Under the provisions of the Indus Waters Treaty, signed between India and

Pakistan in 1960, all the waters of the eastern rivers — the Sutlej, Beas, and

Ravi — amounting to around 33 MAF (million acre-feet) annually is allocated to

India for unrestricted use.

• The waters of western rivers — Indus, Jhelum, and Chenab — amounting to

around 135 MAF annually are largely for Pakistan.

Refer: https://www.insightsonindia.com/2022/02/23/permanent-indus-commission-2/

184. With reference to Indus River, consider the following statements:

1. It originates near the Mansarovar Lake in the Tibetan plateau.

2. It is a transboundary river of Asia.

3. Its left-bank tributary in Ladakh is the Zanskar River.

Revision Through MCQs (RTM) Compilation (Feb - 2022)

Telegram: https://t.me/insightsIAStips Youtube: https://www.youtube.com/channel/UCpoccbCX9GEIwaiIe4HLjwA

120

Which of the statements given above is/are correct?

(a) 1 and 2 only

(b) 2 and 3 only

(c) 1 and 3 only

(d) 1, 2 and 3

Ans: (d)

Explanation:

• The Indus River originates near the Mansarovar Lake in the Tibetan plateau, on

the northern slopes of the Kailash Mountain Range.

• S2: Its left-bank tributary in Ladakh is the Zanskar River, and its left-bank

tributary in the plains is the Panjnad River which itself has five major

tributaries, namely the Chenab, Jhelum, Ravi, Beas, and Sutlej rivers. Its

principal right-bank tributaries are the Shyok, Gilgit, Kabul, Kurram, and Gomal

rivers.

Refer: https://www.insightsonindia.com/2022/02/23/permanent-indus-commission-2/

185. How is the phenomena of gravitational lensing helpful for astronomers?

1. It helps them to investigate how new stars are born.

2. It is useful to estimate the amount and distribution of dark matter in the universe.

Select the correct answer using the code below:

(a) 1 only

(b) 2 only

(c) Both 1 and 2

(d) Neither 1 nor 2

Ans: (c)

Revision Through MCQs (RTM) Compilation (Feb - 2022)

Telegram: https://t.me/insightsIAStips Youtube: https://www.youtube.com/channel/UCpoccbCX9GEIwaiIe4HLjwA

121

Explanation:

• The researchers use the non-observation of the lensing signatures to assess

what fraction of the dark matter could be made of black holes. Gravitational

lensing is useful to cosmologists because it is directly sensitive to the amount

and distribution of dark matter.

• What is gravitational lensing? How does it work?

o Gravitational lensing is an effect of Einstein’s theory of general relativity –

simply put, mass bends light.

o The gravitational field of a massive object will extend far into space, and

cause light rays passing close to that object (and thus through its

gravitational field) to be bent and refocused somewhere else.

o The more massive the object, the stronger its gravitational field and

hence the greater the bending of light rays – just like using denser

materials to make optical lenses results in a greater amount of refraction.

Refer: https://www.insightsonindia.com/2022/02/23/what-is-dark-energy-and-dark-

matter/

186. Consider the following statements about ‘Lucy’ Mission:

1. It is NASA’s first mission to explore the Jupiter Trojan asteroids.

2. It is a solar-powered mission.

Which of the statements given above is/are correct?

(a) 1 only

(b) 2 only

(c) Both 1 and 2

(d) Neither 1 nor 2

Ans: (c)

Explanation:

• This is NASA’s first mission to explore the Jupiter Trojan asteroids.

• It is a solar-powered mission.

• It is estimated to be over 12 years long, during which the spacecraft will visit

eight asteroids covering a distance of about 6.3 billion km to deepen the

understanding of the “young solar system”.

Refer: https://www.insightsonindia.com/2022/02/23/nasas-lucy-mission-2/

187. In India, the Coastal Regulation Zone rules (CRZ rules) were notified under which of

the following?

(a) Wild Life (Protection) Act, 1972

Revision Through MCQs (RTM) Compilation (Feb - 2022)

Telegram: https://t.me/insightsIAStips Youtube: https://www.youtube.com/channel/UCpoccbCX9GEIwaiIe4HLjwA

122

(b) Maritime Zones of India Act 1976

(c) Environment Protection Act in 1986

(d) Biological Diversity Act, 2002

Ans: (c)

Explanation: What are CRZ norms?

• Under the section 3 of Environment Protection Act, 1986 of India, Coastal

Regulation Zone notification was issued in February 1991 for the first time.

• In 2018-19, fresh Rules were issued, which aimed to remove certain restrictions

on building, streamlined the clearance process, and aimed to encourage tourism

in coastal areas.

Refer: https://www.insightsonindia.com/2022/02/23/coastal-regulation-zone-crz-norms-

3/

188. Which one of the following is a measure of sustainable income level that can be

secured without decreasing the stock of natural assets?

(a) Natural capital stock

(b) Environment value

(c) Green accounting

(d) Social discount rate

Ans: (c)

Explanation:

• Better macroeconomic and societal indicators are needed to reflect the

contribution of biodiversity and ecosystem services to human well-being.

• One approach that is gaining momentum across the globe is “green

accounting” whereby national accounts are adjusted to include the value of

nature´s goods and services

• Green accounting allows the computation of income for a nation by taking into

account the economic damage and depletion in natural resource base of a

country.

• It is a measure of sustainable income level that can be secured without

decreasing the stock of natural assets.

Refer: https://www.cbd.int/financial/values/india-greenaccountingmethodology.pdf

189. Consider the following statements

1. It was established in the year 1936 as Hailey National Park.

2. It is the place where Project Tiger was first launched in 1973.

Revision Through MCQs (RTM) Compilation (Feb - 2022)

Telegram: https://t.me/insightsIAStips Youtube: https://www.youtube.com/channel/UCpoccbCX9GEIwaiIe4HLjwA

123

3. Its whole area comprises of hills, marshy depressions, riverine belts, grasslands and

large lake.

4. It has sub-Himalayan belt geographical and ecological characteristics.

The above statements refer to which of the following national parks?

(a) Balphakram National Park

(b) Dudhwa National Park

(c) Manas National Park

(d) Jim Corbett National Park

Ans: (d)

Explanation:

• Jim Corbett National Park is the oldest national park in India and was

established in 1936 as Hailey National Park to protect the endangered Bengal

tiger.

• It is located in Nainital district of Uttarakhand and was named after Jim Corbett

who played a key role in its establishment. The park was the first to come under

the Project Tiger initiative.

• The park has sub-Himalayan belt geographical and ecological characteristics. An

ecotourism destination.

Refer: https://www.corbettnationalpark.in/

190. Which of the following are in Nilgiri biosphere reserve?

(a) Kaundinya gundla bhrameshwaram and papikonda wildlife sanctuaries and

mukurthi national park

(b) Kawal and Shree Venkateshwara wildlife sanctuaries; and Nagarjunasagar-srisailam

tiger reserve

(c) Neyyar, peppara and shendurney wildlife sanctuaries and kalakad mundanthurai

tiger reserve

(d) Aralam, Mudumalai, Mukurthi, Sathyamangalam wildlife sanctuary and Silent

Valley national Park

Ans: (d)

Explanation: Nilgiri biosphere reserve:

• It includes the Aralam, Mudumalai, Mukurthi, Nagarhole, Bandipur and

Silent Valley national parks, as well as the Wayanad and Sathyamangalam

wildlife sanctuaries.

Revision Through MCQs (RTM) Compilation (Feb - 2022)

Telegram: https://t.me/insightsIAStips Youtube: https://www.youtube.com/channel/UCpoccbCX9GEIwaiIe4HLjwA

124

Refer: https://en.wikipedia.org/wiki/Nilgiri_Biosphere_Reserve

RTM- REVISION THROUGH MCQS –24th-Feb-2022 191. “To uphold and protect the Sovereignty Unity and Integrity of India” is a provision

made in the

(a) Preamble of the Constitution

(b) Directive Principles of State Policy

(c) Fundamental Rights

(d) Fundamental Duties

Ans: (d)

Explanation:

• It has been mentioned in Article 51-A of the Constitution (Fundamental Duties).

Refer: https://www.insightsonindia.com/2022/02/24/fundamental-duties-must-be-

enforced-says-plea-in-supreme-court/

192. Which of the following statements is/are true of the Fundamental Duties of an Indian

citizen?

1. A legislative process has been provided to enforce these duties.

2. They are correlative to legal duties.

Select the correct answer using the code given below:

(a) 1 only

(b) 2 only

(c) Both 1 and 2

(d) Neither 1 nor 2

Ans: (d)

Revision Through MCQs (RTM) Compilation (Feb - 2022)

Telegram: https://t.me/insightsIAStips Youtube: https://www.youtube.com/channel/UCpoccbCX9GEIwaiIe4HLjwA

125

Explanation:

• There is no legislative process to enforce citizens to do their duties. The legal

duty is one which is required to be carried out as per law. So, there is no

correlation between both duties.

Refer: https://www.insightsonindia.com/2022/02/24/fundamental-duties-must-be-

enforced-says-plea-in-supreme-court/

193. For a citizen of India, the duty to pay taxes is a:

(a) Constitutional obligation

(b) Moral obligation

(c) Fundamental Duty

(d) Legal obligation

Ans: (d)

Explanation:

• As per the Swaran Singh Committee (that suggested inclusion of fundamental

rights); including duty to pay taxes should have been a fundamental duty, but it

was not agreed upon.

• So as per the Direct taxation laws; income and other kinds of taxes are a legal

obligation for an Indian citizen as he uses public services, receives social and

political security from the state etc

Refer: https://www.insightsonindia.com/2022/02/24/fundamental-duties-must-be-

enforced-says-plea-in-supreme-court/

194. With reference to TRIPS Agreement, consider the following statements:

1. It guarantees minimum standards of IP protection.

2. It came into force in 2020.

3. It is applicable to all UN members.

Which of the statements given above is/are not correct?

(a) 1 only

(b) 2 and 3 only

(c) 2 only

(d) 1, 2 and 3

Ans: (b)

Explanation: here the directive word is not correct!!

• The TRIPS agreement was negotiated in 1995 at the WTO, it requires all its

signatory countries to enact domestic law.

o It is applicable to all WTO members.

Revision Through MCQs (RTM) Compilation (Feb - 2022)

Telegram: https://t.me/insightsIAStips Youtube: https://www.youtube.com/channel/UCpoccbCX9GEIwaiIe4HLjwA

126

• It guarantees minimum standards of IP protection. Such legal consistency

enables innovators to monetise their intellectual property in multiple countries.

• In 2001, the WTO signed the Doha Declaration, which clarified that in a public

health emergency, governments could compel companies to license their patents

to manufacturers, even if they did not think the offered price was acceptable.

• This provision, commonly referred to as “compulsory licensing”, was already

built into the TRIPS Agreement and the Doha declaration only clarified its usage.

Refer: https://www.insightsonindia.com/2022/02/24/india-risks-being-left-out-of-trips-

waiver/

195. In northeast India, Kuki people are present in all states except

(a) Arunachal Pradesh.

(b) Mizoram

(c) Nagaland

(d) Manipur

Ans: (a)

Explanation:

• Basically, the Kuki people are an ethnic group native to the Mizo Hills (formerly

Lushai) in Mizoram.

• In northeast India, they are present in all states except Arunachal Pradesh.

• The Kuki Rising, 1917-1919’ — also seen as the anti-colonial freedom struggle of

the Kukis — was fought against the British to preserve their land. During WWII,

the Kukis joined the Indian Army to fight the British again.

Refer: Facts for Prelims: https://www.insightsonindia.com/2022/02/24/mission-2022-

insights-daily-current-affairs-pib-summary-24-february-2022/

196. With reference to National Means-cum-Merit Scholarship Scheme, consider the

following statements:

1. It is a central Sector Scheme.

2. The Scheme provides scholarships to meritorious students of economically weaker

sections to reduce drop-outs in class in VIII.

Which of the statements given above is/are correct?

(a) 1 only

(b) 2 only

(c) Both 1 and 2

(d) Neither 1 nor 2

Ans: (c)

Revision Through MCQs (RTM) Compilation (Feb - 2022)

Telegram: https://t.me/insightsIAStips Youtube: https://www.youtube.com/channel/UCpoccbCX9GEIwaiIe4HLjwA

127

Explanation:

• The Ministry of Education has approved the continuation of National Means-

cum-Merit Scholarship (NMMSS) over the 15th Finance Commission cycle for a

period of five years i.e. from 2021-22 to 2025-26.

• It is a central Sector Scheme, launched in 2008-09 to award scholarships to

meritorious students of economically weaker sections to reduce drop-outs

in class VIII.

• The scheme envisages award of one lakh fresh scholarships every year to

selected students of class IX and their continuation/renewal in classes X to XII

for study in a State Government, Government-aided and Local body schools

under the scheme.

Refer: Facts for Prelims: https://www.insightsonindia.com/2022/02/24/mission-2022-

insights-daily-current-affairs-pib-summary-24-february-2022/

197. Which of the following statements is/are correct about Sea cucumber:

1. Sea cucumber in India is treated as an endangered species.

2. They are marine invertebrates that live on the seafloor found generally in tropical

regions.

3. They act like garbage collectors of the ocean world.

Select the correct answer using the code below:

(a) 1 and 2 only

(b) 2 and 3 only

(c) 1 and 3 only

(d) 1, 2 and 3

Ans: (d)

Explanation:

• Sea cucumber in India is treated as an endangered species listed under

schedule I of Wildlife Protection Act of 1972.

• They are marine invertebrates that live on the seafloor found generally in

tropical regions. They’re named for their unusual oblong shape that resembles

a fat cucumber.

• They are an integral part of the coral ecosystem as one of the main by-

products of the sea cucumbers digestion of sand is calcium carbonate and this is

essential for the survival of the coral reefs.

• They act like garbage collectors of the ocean world, and they recycle nutrients,

thus playing an important role in keeping coral reefs in good condition.

Revision Through MCQs (RTM) Compilation (Feb - 2022)

Telegram: https://t.me/insightsIAStips Youtube: https://www.youtube.com/channel/UCpoccbCX9GEIwaiIe4HLjwA

128

• IUCN Red List: Brown Sea Cucumber (Endangered), Blackspotted Sea Cucumber

(Least Concern), Blue Sea Cucumber (Data Deficient), etc.

Refer: Facts for Prelims: https://www.insightsonindia.com/2022/02/24/mission-2022-

insights-daily-current-affairs-pib-summary-24-february-2022/

198. Consider the following statements regarding Nord Stream 2 pipeline, recently seen in

news:

1. It runs from Russia to Germany.

2. It runs across the Mediterranean Sea.

Which of the statements given above is/are correct?

(a) 1 only

(b) 2 only

(c) Both 1 and 2

(d) Neither 1 nor 2

Ans: (a)

Explanation: Nord Stream 2 Pipeline:

• This is a 1,200-km pipeline that runs from Ust-Luga in Russia to Greifswald

in Germany through the Baltic Sea. It will carry 55 billion cubic metres of gas

per year.

• It was decided to build this pipeline in 2015.

Refer: facts for prelims: https://www.insightsonindia.com/2022/02/24/mission-2022-

insights-daily-current-affairs-pib-summary-24-february-2022/

199. With reference to Geography of Taiwan, Consider the following statements:

1. The East China Sea lies to its north.

2. The Philippine Sea lies to its east.

3. The Luzon Strait lies directly to its south.

4. The South China Sea lies to its southwest.

Which of the given above statements is/are correct?

(a) 1 and 2 only

(b) 3 and 4 only

(c) 1, 2 and 4 only

(d) 1, 2, 3 and 4

Ans: (d)

Explanation:”

• Taiwan is an island country in East Asia

Revision Through MCQs (RTM) Compilation (Feb - 2022)

Telegram: https://t.me/insightsIAStips Youtube: https://www.youtube.com/channel/UCpoccbCX9GEIwaiIe4HLjwA

129

• The East China Sea lies to its north, the Philippine Sea to its east, the

Luzon Strait directly to its south and the South China Sea to its southwest.

• Smaller islands include a number in the Taiwan Strait including the Penghu

archipelago, the Kinmen and Matsu Islands near the Chinese coast, and some of

the South China Sea Islands.

Refer: https://en.wikipedia.org/wiki/Taiwan

200. With reference to criteria for name selection of tropical cyclones, consider the following

statements:

1. The proposed names are rotated every few years

2. The maximum length of the name will be eight letters

3. The name should be short and readily understood when broadcasted

Which of the given above statements is/are correct?

(a) 1 and 2 only

(b) 2 and 3 only

(c) 1 and 3 only

(d) 1, 2 and 3

Ans: (b)

Explanation:

• Worldwide there are six Regional Specialised Meteorological Centres (RSMCs)

and five regional Tropical Cyclone Warning Centres (TCWCs) mandated for

issuing advisories and naming of tropical cyclones. The tropical cyclones forming

over different Ocean basins are named by the concerned RSMCs & TCWCs.

Revision Through MCQs (RTM) Compilation (Feb - 2022)

Telegram: https://t.me/insightsIAStips Youtube: https://www.youtube.com/channel/UCpoccbCX9GEIwaiIe4HLjwA

130

• India Meteorological Department is one of the six RSMCs to provide tropical

cyclone and storm surge advisories to 13 member countries under WMO/ESCAP

Panel

• Link: Criteria for name selection

o The proposed name should be neutral to (a) politics and political figures

(b) religious believes, (c) cultures and (d) gender Name should be chosen

in such a way that it does not hurt the sentiments of any group of

population over the globe

o It should not be very rude and cruel in nature

o S3: It should be short, easy to pronounce and should not be

offensive to any member

o S2: The maximum length of the name will be eight letters

o The proposed name should be provided along with its pronunciation and

voice over

o S1: the names are not rotated every few years. It is believed that since

the cyclones cause so much death, destruction and devastation, names

are considered for retirement and hence is not used repeatedly.

Refer: https://vikaspedia.in/energy/environment/know-your-environment/naming-of-

tropical-cyclones

Revision Through MCQs (RTM) Compilation (Feb - 2022)

Telegram: https://t.me/insightsIAStips Youtube: https://www.youtube.com/channel/UCpoccbCX9GEIwaiIe4HLjwA

131

Revision Through MCQs (RTM) Compilation (Feb - 2022)

Telegram: https://t.me/insightsIAStips Youtube: https://www.youtube.com/channel/UCpoccbCX9GEIwaiIe4HLjwA

132

RTM- REVISION THROUGH MCQS –25th-Feb-2022

201. Consider the following statements with reference to Tribunal Reforms Act, 2021:

1. It amended Finance Act, 2017 that merged the tribunals based on domain.

2. It provides that the Chairperson and Members of the Tribunals will be appointed by

the central government on the recommendation of Chief Justice of India.

3. It seeks to dissolve certain existing appellate bodies and transfer their functions to

other existing judicial bodies.

Which of the statements given above is/are correct?

(a) 1 and 2 only

(b) 2 and 3 only

(c) 1 and 3 only

(d) 1, 2 and 3

Ans: (c)

Explanation: Key changes:

• It seeks to dissolve certain existing appellate bodies and transfer their

functions to other existing judicial bodies.

• It seeks to empower the Central Government to make rules for qualifications,

appointment, term of office, salaries and allowances, resignation, removal and

other terms and conditions of service of Members of Tribunals.

• It provides that the Chairperson and Members of the Tribunals will be

appointed by the Central Government on the recommendation of a Search-

cum-Selection Committee.

• It also provides the composition of the Committee, to be headed by the Chief

Justice of India or a Judge of the Supreme Court nominated by him.

• For state tribunals, there will be a separate search committee.

• The Union government has to ‘preferably’ decide on the recommendations of

the search-cum selection committee within 3 months of the date of the

recommendation.

• Tenure: Chairperson of a Tribunal shall hold office for a term of 4 years or till he

attains the age of 70 years, whichever is earlier. Other Members of a Tribunal

shall hold office for a term of 4 years or till he attains the age of 67 years,

whichever is earlier.

Refer: https://www.insightsonindia.com/2022/02/25/tribunal-reforms-act-of-2021/

Revision Through MCQs (RTM) Compilation (Feb - 2022)

Telegram: https://t.me/insightsIAStips Youtube: https://www.youtube.com/channel/UCpoccbCX9GEIwaiIe4HLjwA

133

202. Consider the following statements:

1. The Central Government announces Fair and Remunerative Prices (FRP) which are

determined on the recommendation of the Cabinet Committee on Economic Affairs

(CCEA).

2. The FRP is based on the recovery of sugar from the cane.

3. CCEA is chaired by the Finance Minister of India.

Which of the statements given above is/are correct?

(a) 1 and 2 only

(b) 2 only

(c) 1 and 3 only

(d) 1, 2 and 3

Ans: (b)

Explanation:

• The FRP is based on the recovery of sugar from the cane. For the sugar

season of 2021-22, FRP has been fixed at Rs 2,900/tonne at a base recovery of

10 per cent.

o Sugar recovery is the ratio between sugar produced versus cane crushed,

expressed as a percentage.

o The higher the recovery, the higher is the FRP, and higher is the sugar

produced.

• The Central Government announces Fair and Remunerative Prices which are

determined on the recommendation of the Commission for Agricultural Costs

and Prices (CACP) and announced by the Cabinet Committee on Economic

Affairs (CCEA).

o CCEA is chaired by the Prime Minister of India.

o The FRP is based on the Rangarajan Committee report on reorganizing

the sugarcane industry.

Refer: https://www.insightsonindia.com/2022/02/25/what-is-fair-and-remunerative-

price-frp-how-is-it-paid/

203. The term “Quantum key distribution” is used in the context of:

(a) Digital security infrastructure

(b) Secure communication method

(c) Internet of things

(d) None of the above

Ans: (b)

Explanation:

Revision Through MCQs (RTM) Compilation (Feb - 2022)

Telegram: https://t.me/insightsIAStips Youtube: https://www.youtube.com/channel/UCpoccbCX9GEIwaiIe4HLjwA

134

• Quantum key distribution is a secure communication method which

implements a cryptographic protocol involving components of quantum

mechanics. It enables two parties to produce a shared random secret key known

only to them, which can then be used to encrypt and decrypt messages.

Refer: https://www.insightsonindia.com/2022/02/25/what-is-quantum-tech-demo-by-

drdo-and-iit-delhi-all-about/

204. Which of the following statements is/are about Non-fungible token (NFT)?

1. NFT is a unique token on the block chain which cannot be replaced with something

else.

2. NFTs can have only one owner at a time.

3. NFT cannot be exchanged for another NFT.

Select the correct answer using the code below:

(a) 1 and 2 only

(b) 2 and 3 only

(c) 1 and 3 only

(d) 1, 2 and 3

Ans: (d)

Explanation:

• S1: An NFT is a non-fungible token. And what this means is, an NFT is a

unique token on the block chain which cannot be replaced with something else.

NFTs can really be anything digital, including drawings, music, photographs,

videos and any type of digital file.

• S2: NFTs can have only one owner at a time.

• S3: Cryptocurrencies such as Bitcoin and Ethereum are also fungible, which

means that one Bitcoin can be exchanged for another.

o But an NFT cannot be exchanged for another NFT because the two are

different and therefore unique.

Refer: https://www.insightsonindia.com/2022/02/25/what-are-nfts/

205. Consider the following statements regarding Cord blood banking.

1. Cord blood is the blood from the baby that is left in the umbilical cord and placenta

after birth, which is a rich source of stem cells.

2. Indian Council of Medical Research (ICMR) allows commercial stem cell banking.

3. Stem cells in cord blood can be used to strengthen the immune system during

cancer treatments.

Which of the above statements is/are correct?

Revision Through MCQs (RTM) Compilation (Feb - 2022)

Telegram: https://t.me/insightsIAStips Youtube: https://www.youtube.com/channel/UCpoccbCX9GEIwaiIe4HLjwA

135

(a) 1 and 2 only

(b) 1 and 3 only

(c) 2 and 3 only

(d) 1, 2 and 3

Ans: (b)

Explanation:

• Cord blood banking involves taking the umbilical cord blood, which is a rich

source of stem cells, and preserving it for future use.

• S2: Indian Council of Medical Research (ICMR) does not recommend

commercial stem cell banking. It says so far there is no scientific basis for

preservation of cord blood for future self use and this practice therefore raises

ethical and social concerns. “Private storage of the cord blood is advisable when

there is an elder child in the family with a condition treatable with these cells

and the mother is expecting the next baby.

• “Globally, cord blood banking is recommended as a source of hematopoietic stem

cell (derived from bone marrow, peripheral blood, or umbilical cord blood)

transplantation for haematological cancers and disorders where its use is

recommended. For all other conditions, the use of cord blood as a source of stem

cells is not yet established,”

• Using the stem cells in cord blood to treat a disease has the following benefits

compared with using those in bone marrow:

• Stem cells from cord blood can be given to more people than those from bone

marrow. More matches are possible when a cord blood transplant is used than

when a bone marrow transplant is used. In addition, the stem cells in cord blood

are less likely to cause rejectionthan those in bone marrow.

• It is harder to collect bone marrow than it is to collect cord blood. Collecting

bone marrow poses some risks and can be painful for the donor.

• Cord blood can be frozen and stored. It is ready for anyone who needs it. Bone

marrow must be used soon after it is collected.

• Stem cells in cord blood can be used to strengthen the immune system during

cancer treatments. Bone marrow stem cells do not have this capability.

Refer: https://www.insightsonindia.com/2022/02/25/umbilical-cord-blood-and-its-

significance/

206. Consider the following statements about Narsinh Mehta:

1. He was a 15th-century poet-saint of Gujarat, India,

2. He was the disciple of Vyasatirtha Swamiji.

3. His bhajan Vaishnav Jan To was Mahatma Gandhi's favorite.

Revision Through MCQs (RTM) Compilation (Feb - 2022)

Telegram: https://t.me/insightsIAStips Youtube: https://www.youtube.com/channel/UCpoccbCX9GEIwaiIe4HLjwA

136

Which of the statements given above is/are correct?

(a) 1 and 2 only

(b) 2 and 3 only

(c) 1 and 3 only

(d) 1, 2 and 3

Ans: (c)

Explanation:

• S1: Narsinh Mehta is regarded as adikavi (the first poet) and bhakta kavi (devout

poet) in Gujarati literature.

• S3: Mahatma Gandhi referenced Narsinh's work considerably in his speeches,

writings, and public prayers. Gandhi's repeated reference to the bhajan

Vaishnav Jana To shaped it as a global song of compassion, moral integrity, and

duty to humankind. Gandhi elevated Narsinh's life and work beyond the

religious context and into greater ethical and moral themes.

• S2: Kanakadasa was the disciple of Vyasatirtha Swamiji.

Refer: Facts for Prelims: https://www.insightsonindia.com/2022/02/25/mission-2022-

insights-daily-current-affairs-pib-summary-24-february-2022-2/

207. The Angadia system was in news recently, is related to:

(a) Agriculture

(b) Parallel Banking

(c) Stock Exchange

(d) Sports Management

Ans: (b)

Explanation:

• The Angadia system is a century-old parallel banking system in the country

where traders send cash generally from one state to another through a person

called Angadia that stands for courier.

• It is by and large used in the jewellery business with Mumbai – Surat being the

most popular route as they are two ends of the diamond trade.

• The cash involved is huge and it is the responsibility of the Angadia to transfer

cash from one state to another for which they charge a nominal fee.

• Generally, it is the Gujarati, Marwari and Malbari community that are involved

in the business.

Refer: Facts for Prelims: https://www.insightsonindia.com/2022/02/25/mission-2022-

insights-daily-current-affairs-pib-summary-24-february-2022-2/

Revision Through MCQs (RTM) Compilation (Feb - 2022)

Telegram: https://t.me/insightsIAStips Youtube: https://www.youtube.com/channel/UCpoccbCX9GEIwaiIe4HLjwA

137

208. Consider the following statements:

1. Panruti cashew is mainly found in Tamil Nadu.

2. It is commonly cultivated in black soil as well as coastal sands.

Which of the statements given above is/are correct?

(a) 1 only

(b) 2 only

(c) Both 1 and 2

(d) Neither 1 nor 2

Ans: (a)

Explanation:

• Panruti cashew is mainly found in Tamil Nadu. This is unique because of its

taste and quality. Although these cashew nuts are slightly smaller in size, they

have a distinct market value. It is commonly known as the ‘gold mine’ of

Cuddalore in Tamil Nadu.

• It is commonly cultivated in laterite & red soils as well as coastal sands.

• Tamil Nadu Cashew Processors and Exporters Association (TNCPEA) has applied

for Geographical Indication (GI) tag for ‘Panruti Cashew’.

Refer: Facts for Prelims: https://www.insightsonindia.com/2022/02/25/mission-2022-

insights-daily-current-affairs-pib-summary-24-february-2022-2/

209. Karakattam is an ancient folk dance of

(a) Kerala

(b) Karnataka

(c) Tamil Nadu

(d) Andhra Pradesh

Ans: (c)

Explanation:

• Karakattam is an ancient folk dance of Tamil Nadu performed in praise of the

rain goddess Mariamman (the rain goddess).

• The performers balance a pot on their head.

Refer: Facts for Prelims: https://www.insightsonindia.com/2022/02/25/mission-2022-

insights-daily-current-affairs-pib-summary-24-february-2022-2/

210. Consider the following statements regarding FASTag technology:

1. FASTag is an electronic toll collection system in India, operated by the National

Payments Corporation of India (NPCI).

Revision Through MCQs (RTM) Compilation (Feb - 2022)

Telegram: https://t.me/insightsIAStips Youtube: https://www.youtube.com/channel/UCpoccbCX9GEIwaiIe4HLjwA

138

2. It employs Radio Frequency Identification (RFID) technology for making toll

payments directly from the prepaid or savings account linked to it.

3. It is affixed on the windscreen of the vehicle and enables to drive through toll plazas

without stopping for transactions.

Which of the given above statements is/are correct?

(a) 1 and 2 only

(b) 3 only

(c) 2 and 3 only

(d) 1, 2 and 3

Ans: (c)

Explanation:

• S1: FASTag is an electronic toll collection system in India, operated by the

National Highway Authority of India (NHAI).

• S2: It employs Radio Frequency Identification (RFID) technology for making toll

payments directly from the prepaid or savings account linked to it or directly toll

owner.

• S3: It is affixed on the windscreen of the vehicle and enables to drive through toll

plazas without stopping for transactions. The tag can be purchased from official

Tag issuers or participating Banks and if it is linked to a prepaid account, then

recharging or top-up can be as per requirement.

• The payment method is a part of the National Electronic Toll Collection

(NETC) programme. The National Payments Corporation of India (NPCI) collects

the payments.

Refer: https://www.business-standard.com/about/what-is-fastag

RTM- REVISION THROUGH MCQS –26th-Feb-2022

211. Consider the following statements about Lachit Borphukan:

1. He was a commander in the Ahom kingdom.

2. He was known for his leadership in the 1671 Battle of Saraighat.

Which of the statements given above is/are correct?

(a) 1 and 2 only

(b) 2 and 3 only

(c) 1 and 3 only

(d) 1, 2 and 3

Ans: (c)

Explanation:

• He was a commander in the Ahom kingdom.

Revision Through MCQs (RTM) Compilation (Feb - 2022)

Telegram: https://t.me/insightsIAStips Youtube: https://www.youtube.com/channel/UCpoccbCX9GEIwaiIe4HLjwA

139

• Known for his leadership in the 1671 Battle of Saraighat that thwarted a drawn-

out attempt by Mughal forces under the command of Ramsingh I to take over

Ahom kingdom.

• The battle of Saraighat was fought on the banks of the Brahmaputra in

Guwahati.

• The National Defence Academy (NDA), ever since 1999 has been conferring the

best passing out cadet with the Lachit Borphukan gold medal.

Refer: https://www.insightsonindia.com/2022/02/26/lachit-borphukan-3/

212. Consider the following statements about the Council of Europe:

1. It was founded in the wake of World War II to uphold human rights in Europe.

2. It has 46 member states.

3. The headquarters of the Council of Europe are in Strasbourg, France.

Which of the statements given above is/are correct?

(a) 1 and 2 only

(b) 2 and 3 only

(c) 1 and 3 only

(d) 1, 2 and 3

Ans: (d)

Explanation:

• The Council of Europe is an international organization founded in the wake of

World War II to uphold human rights, democracy and the rule of law in Europe.

• Founded in 1949 (Treaty of London (1949)), it has 46 member states (including

all 27 EU members), with a population of approximately 820 million, and

operates with an annual budget of approximately 500 million euros.

• Headquarters: Palace of Europe, Strasbourg, France.

• Roles and responsibilities:

o The Council of Europe cannot make binding laws, but it does have the

power to enforce select international agreements reached by European

states on various topics.

Refer: https://www.insightsonindia.com/2022/02/26/council-of-europe/

213. With reference to the Blockchain Technology, consider the following statements:

1. It is a public ledger that everyone can inspect, which no single user controls

2. The structure and design of blockchain is such that all the data in it are about

cryptocurrency only

Revision Through MCQs (RTM) Compilation (Feb - 2022)

Telegram: https://t.me/insightsIAStips Youtube: https://www.youtube.com/channel/UCpoccbCX9GEIwaiIe4HLjwA

140

3. Applications that depend on the basic features of blockchain can be developed

without anybody’s permission

Which of the statements given above is/are correct?

(a) 1 only

(b) 1 and 2 only

(c) 2 only

(d) 1 and 3 only

Ans: (d)

Explanation:

• A blockchain is a decentralized, distributed, and oftentimes public, digital ledger

consisting of records called blocks that is used to record transactions across

many computers so that any involved block cannot be altered retroactively,

without the alteration of all subsequent blocks.

• This allows the participants to verify and audit transactions independently and

relatively inexpensively. A blockchain database is managed autonomously using

a peer-to-peer network and a distributed timestamping server. They are

authenticated by mass collaboration powered by collective self-interests. Such a

design facilitates robust workflow where participants’ uncertainty regarding data

security is marginal.

• The great advantage to an open, permissionless, or public, blockchain network is

that guarding against bad actors is not required and no access control is needed.

This means that applications can be added to the network without the approval

or trust of others, using the blockchain as a transport layer.

• S2 is clearly wrong, Blockchain can be used for a variety of purposes of which

cryptocurrency is only one of them

Refer: https://www.insightsonindia.com/2022/02/26/sc-asks-centre-to-clarify-on-legality-

of-cryptocurrency/

214. Chernobyl disaster was in news recently, is related to which of the following?

(a) nuclear accident

(b) the great hack

(c) melting of glaciers in Antarctica

(d) mass death of fishes

Ans: (a)

Explanation:

Revision Through MCQs (RTM) Compilation (Feb - 2022)

Telegram: https://t.me/insightsIAStips Youtube: https://www.youtube.com/channel/UCpoccbCX9GEIwaiIe4HLjwA

141

• The Chernobyl tragedy was the result of a nuclear accident on 26 April 1986 at

reactor No. 4 at the Chernobyl Nuclear Power Plant, near the town of Pripyat

in the Ukrainian SSR (Ukrainian Soviet Socialist Republic).

o There were nearly 8.4 million people exposed to radiation in the three

nations.

o It occurred when a group of technicians in what was then Soviet-

controlled Ukraine carried out a botched safety test that led to a series of

explosions.

o It is said to have released 400 times more radiation than the atomic

bomb that was dropped on Hiroshima in Japan.

Refer: https://www.insightsonindia.com/2022/02/26/chernobyl-disaster/

215. In which one of the following States is Kuno-Palpur National Park is located?

(a) Odisha

(b) Rajasthan

(c) Madhya Pradesh

(d) Maharashtra

Ans: (c)

Explanation:

• Kuno-Palpur National Park in Madhya Pradesh could become the country’s

first-ever cheetah sanctuary.

• Although the plan of bringing cheetahs to India was in the works for more than a

decade, it came close to a realisation after the Supreme Court’s nod in

January 2020.

Refer: Facts for Prelims: https://www.insightsonindia.com/2022/02/26/mission-2022-

insights-daily-current-affairs-pib-summary-26-february-2022/

216. Consider the following statements:

1. The cheetah is the world’s fastest land mammal.

2. It is listed as endangered in IUCN red listed species.

Which of the statements given above is/are correct?

(a) 1 only

(b) 2 only

(c) Both 1 and 2

(d) Neither 1 nor 2

Ans: (a)

Explanation:

Revision Through MCQs (RTM) Compilation (Feb - 2022)

Telegram: https://t.me/insightsIAStips Youtube: https://www.youtube.com/channel/UCpoccbCX9GEIwaiIe4HLjwA

142

• The cheetah is also the world’s fastest land mammal.

• It is listed as vulnerable in IUCN red listed species.

• The country’s last spotted feline died in Chhattisgarh in 1947. Later, the cheetah

— which is the fastest land animal — was declared extinct in India in 1952.

• The Asiatic cheetah is classified as a “critically endangered” species by the

IUCN Red List, and is believed to survive only in Iran.

Refer: Facts for Prelims: https://www.insightsonindia.com/2022/02/26/mission-2022-

insights-daily-current-affairs-pib-summary-26-february-2022/

217. The ‘Sustainable Cities India' is an initiative of

(a) The Energy and Resources Institute

(b) NITI Aayog

(c) National Biodiversity Authority

(d) None of the above

Ans: (d)

Explanation:

• The World Economic Forum and the National Institute of Urban Affairs

(NIUA) have signed a Memorandum of Understanding (MoU) to collaborate on a

jointly designed ‘Sustainable Cities India program’.

• The ‘Sustainable Cities India program’ intends to enable cities to decarbonize in

a systematic and sustainable way that will reduce emissions and deliver resilient

and equitable urban ecosystems.

• Implementation:

o The Forum and NIUA will adapt the Forum’s City Sprint process and

Toolbox of Solutions for decarbonization in the context of five to seven

Indian cities across two years.

Refer: Facts for Prelims: https://www.insightsonindia.com/2022/02/26/mission-2022-

insights-daily-current-affairs-pib-summary-26-february-2022/

218. Consider the following statements:

1. Corbevax is India’s indigenous Covid-19 vaccine.

2. It is a “recombinant protein sub-unit” vaccine.

Which of the statements given above is/are correct?

(a) 1 only

(b) 2 only

(c) Both 1 and 2

(d) Neither 1 nor 2

Revision Through MCQs (RTM) Compilation (Feb - 2022)

Telegram: https://t.me/insightsIAStips Youtube: https://www.youtube.com/channel/UCpoccbCX9GEIwaiIe4HLjwA

143

Ans: (c)

Explanation:

• It is India’s indigenous Covid-19 vaccine.

• It is a “recombinant protein sub-unit” vaccine.

• It means it is made up of a specific part of SARS-CoV-2 – the spike protein on

the virus’s surface

• The spike protein allows the virus to enter the cells in the body so that it can

replicate and cause disease.

• However, when this protein alone is given to the body, it is not expected to be

harmful as the rest of the virus is absent.

• The body is expected to develop an immune response against the injected spike

protein.

Refer: Facts for Prelims: https://www.insightsonindia.com/2022/02/26/mission-2022-

insights-daily-current-affairs-pib-summary-26-february-2022/

219. SPARSH is an initiative of the

(a) Ministry of Personnel, Public Grievances and Pensions

(b) Ministry of Social Justice and Empowerment

(c) Ministry of Women and Child Development

(d) Ministry of Defense

Ans: (d)

Explanation:

• The Ministry of Defense has signed an agreement with CSC e-Governance

Services India Limited (under Ministry of Electronics and Information

Technology) to expand the coverage of pension services under SPARSH or

System for Pension Administration (Raksha) across four lakh Common Service

Centers.

• This will provide last mile connectivity to pensioners, especially those who live in

remote areas of the country and those who do not have access or technical

knowledge and skill to use SPARSH.

• SPARSH is an initiative of the Ministry of Defense.

• It aims to provide comprehensive services to defense pensioners.

• It promotes the Government’s vision of ‘Digital India’, ‘Direct Benefit Transfer

(DBT)’ and ‘Minimum Government, Maximum Governance’.

Refer: Facts for Prelims: https://www.insightsonindia.com/2022/02/26/mission-2022-

insights-daily-current-affairs-pib-summary-26-february-2022/

Revision Through MCQs (RTM) Compilation (Feb - 2022)

Telegram: https://t.me/insightsIAStips Youtube: https://www.youtube.com/channel/UCpoccbCX9GEIwaiIe4HLjwA

144

220. Due to some reasons, if photosynthesis in plants suddenly stopped happening, what

could be its likely consequence/consequences?

1. most plants would die within short order

2. green algal blooms would cover the entire face of the planet

3. global emissions of CO2 would be greatly reduced

Select the correct answer using the code given below:

(a) 1 only

(b) 2 and 3 only

(c) 1 and 2 only

(d) 1, 2 and 3

Ans: (a)

Explanation:

• If photosynthesis came to an abrupt end, most plants would die within short

order. Although they could hold out for a few days -- or in some cases, a few

weeks -- how long they lived would largely be a factor of how much sugar they

had stored within their cells.

• The only organisms able to exist under such conditions would be the

chemosynthetic bacteria, which can utilize the chemical energy of certain

inorganic compounds and thus are not dependent on the conversion of light

energy.

• S2 and S3 are absurd statements.

Refer: https://www.insightsonindia.com/2021/03/19/artificial-photosynthesis-to-provide-

solutions-for-carbon-capture-and-conversion/

RTM- REVISION THROUGH MCQS –28th-Feb-2022

221. Consider the following statements:

1. The Bhakra dam is built on Beas River.

2. It is Asia’s second tallest dam.

Which of the statements given above is/are correct?

(a) 1 only

(b) 2 only

(c) Both 1 and 2

(d) Neither 1 nor 2

Ans: (b)

Explanation:

• The Bhakra dam is built on Sutlej River.

Revision Through MCQs (RTM) Compilation (Feb - 2022)

Telegram: https://t.me/insightsIAStips Youtube: https://www.youtube.com/channel/UCpoccbCX9GEIwaiIe4HLjwA

145

• It is Asia’s second tallest dam with a height of around 207.26 meters after the

Tehri dam, which has a height of around 261 meters. The Tehri dam is also

situated in India in the state of Uttrakhand.

• The Gobind Sagar reservoir of the dam has a capacity to store water up to 9.34

billion cubic meters.

Refer: https://www.insightsonindia.com/2022/02/28/bhakra-beas-management-board-

bbmb/

222. Consider the following statements:

1. The North Atlantic Treaty Organization (NATO) is an intergovernmental military

alliance.

2. NATO's purpose is to guarantee the freedom and security of its members through

political and military means.

3. The NATO headquarters are located in Zurich, Switzerland.

Which of the statements given above is/are correct?

(a) 1 and 2 only

(b) 2 and 3 only

(c) 1 and 3 only

(d) 1, 2 and 3

Ans: (a)

Explanation:

• Formed in 1949 with the signing of the Washington Treaty, NATO is a security

alliance of 30 countries from North America and Europe. NATO's fundamental

goal is to safeguard the Allies' freedom and security by political and military

means.

• The NATO headquarters are located in Brussels, Belgium, while the

headquarters of Allied Command Operations is near Mons, Belgium.

Refer: https://www.insightsonindia.com/2022/02/28/finland-and-swedens-nato-

membership/

223. Consider the following statements regarding Forest Rights Act.

1. The Ministry of Environment, Forest and Climate Change (MoEFCC) is the

implementing agency.

2. The Act establishes Forest Rights Committee (FRC) comprising members from within

the village to facilitate the implementation of the Act.

3. The FRA provides for equal rights in titles issued under the Act for women.

Which of the above statements is/are correct?

Revision Through MCQs (RTM) Compilation (Feb - 2022)

Telegram: https://t.me/insightsIAStips Youtube: https://www.youtube.com/channel/UCpoccbCX9GEIwaiIe4HLjwA

146

(a) 1 and 3 only

(b) 1 and 2 only

(c) 2 and 3 only

(d) 1, 2 and 3

Ans: (c)

Explanation:

• The Forest Rights Act (FRA) has been in existence for 15 years. As on April 30,

2020, the Ministry of Tribal Affairs had received 42,50,602 claims (individual

and community), of which titles were distributed to 46% of the applicants.

Despite the Ministry being the implementing agency, the role of the Forest

Department in granting titles is crucial because the lands claimed are under its

jurisdiction.

• When a forest dweller files a claim for the land title, or patta, it passes through

three levels of checks — the gram sabha, the sub divisional level committee

(SDLC) and the district level committee (DLC). A highly inclusive body, called the

Forest Rights Committee (FRC), is formed by the gram sabha to facilitate the

process. This committee comprises 10 to 15 people, two-thirds of them should

belong to scheduled tribes and one-third members should be women.

• The FRA provides for equal rights in titles issued under the Act for women.

They have the equitable role at every stage of decision-making.

Refer: https://www.insightsonindia.com/2022/02/28/the-scheduled-tribes-and-other-

traditional-forest-dwellers-recognition-of-rights-act-3/

224. Consider the following statements:

1. The only space station currently in orbit is the International Space Station (ISS).

2. The ISS is backed by the United States, Russia, Australia, France and Canada.

Which of the statements given above is/are correct?

(a) 1 only

(b) 2 only

(c) Both 1 and 2

(d) Neither 1 nor 2

Ans: (a)

Explanation:

• The only space station currently in orbit is the International Space Station (ISS).

The ISS is backed by the United States, Russia, Europe, Japan and Canada.

• So far, China has sent two previous space stations into orbit- the Tiangong-1

and Tiangong-2 were trial stations.

Revision Through MCQs (RTM) Compilation (Feb - 2022)

Telegram: https://t.me/insightsIAStips Youtube: https://www.youtube.com/channel/UCpoccbCX9GEIwaiIe4HLjwA

147

Refer: https://www.insightsonindia.com/2022/02/28/217376/

225. Consider the following statements:

1. Madagascar is an island country in the Indian Ocean.

2. Madagascar is a biodiversity hotspot.

Which of the statements given above is/are correct?

(a) 1 only

(b) 2 only

(c) Both 1 and 2

(d) Neither 1 nor 2

Ans: (c)

Explanation:

• Madagascar is an island country in the Indian Ocean,

• The Madagascar and Indian Ocean Islands Hotspot is one of 36 biodiversity

hotspots on Earth. It is, therefore, one of the planet's richest areas, not only in

terms of biodiversity, but also in regard to endangered species.

Revision Through MCQs (RTM) Compilation (Feb - 2022)

Telegram: https://t.me/insightsIAStips Youtube: https://www.youtube.com/channel/UCpoccbCX9GEIwaiIe4HLjwA

148

Refer: Facts for Prelims: https://www.insightsonindia.com/2022/02/28/mission-2022-

insights-daily-current-affairs-pib-summary-28-february-2022/

226. Consider the following statements about Export Credit Guarantee Corporation of

India:

1. It provides export credit insurance support to Indian exporters.

2. It is a wholly-owned subsidiary of the Reserve Bank of India (RBI).

3. It is based in Chennai, Tamil Nadu.

Which of the above statements is/are correct?

(a) 1 only

(b) 1 and 2 only

(c) 2 and 3 only

(d) 1, 2 and 3

Ans: (a)

Explanation:

• The ECGC Ltd. (LExport Credit Guarantee Corporation of India Ltd) is a

government owned export credit provider.

• It provides export credit insurance support to Indian exporters.

• It is under the ownership of the Ministry of Commerce and Industry,

Government of India.

• It is based in Mumbai.

• It was established in 1957.

Refer: Facts for Prelims: https://www.insightsonindia.com/2022/02/28/mission-2022-

insights-daily-current-affairs-pib-summary-28-february-2022/

227. The Bhasha Certificate Selfie campaign has been launched by the

(a) Ministry of Culture

(b) Ministry of Home Affairs

(c) Ministry of Information and Broadcasting

(d) Ministry of Education

Ans: (d)

Explanation:

• Ministry of Education has launched a campaign ‘Bhasha Certificate Selfie to

encourage cultural diversity and promote mulitlinguism and to foster the

spirit of Ek Bharat Shreshtha Bharat.

• The initiative aims to promote the Bhasha Sangam mobile app, developed by

Ministry of Education and MyGov India.

Revision Through MCQs (RTM) Compilation (Feb - 2022)

Telegram: https://t.me/insightsIAStips Youtube: https://www.youtube.com/channel/UCpoccbCX9GEIwaiIe4HLjwA

149

Refer: Facts for Prelims: https://www.insightsonindia.com/2022/02/28/mission-2022-

insights-daily-current-affairs-pib-summary-28-february-2022/

228. Consider the following statements about Ram Prasad Bismil:

1. He was associated with the Arya Samaj from an early age.

2. He participated in the Muzaffarpur Conspiracy of 1908.

3. He played a crucial role in Ghadar Movement.

Which of the given above statements is/are correct?

(a) 1 and 2 only

(b) 1 only

(c) 2 and 3 only

(d) 1, 2 and 3

Ans: (b)

Explanation:

• S2: Events he was associated with:

o He participated in the Mainpuri conspiracy of 1918.

o On August 9, 1925, Ram Prasad Bismil along with companions

Ashfaqulla Khan and others executed the plan of looting the train at

Kakori near Lucknow.

• S1: He was associated with the Arya Samaj from an early age.

Refer: https://www.insightsonindia.com/2021/06/16/ram-prasad-bismil/

229. Consider the following statements regarding the Arya Samaj:

1. It is a Hindu revivalist movements.

2. It was founded by Dayanand Saraswati in Lahore, Punjab.

3. It believes in the infallible authority of the Vedas.

Which of the given above statements is/are correct?

(a) 1 and 2 only

(b) 3 only

(c) 2 and 3 only

(d) 1, 2 and 3

Ans: (b)

Explanation:

• S1 and S2: It is a Hindu reform movement that was founded by Dayanand

Saraswati in 1875 in Bombay.

• S3: The movement believes in the infallible authority of the Vedas.

o The central objectives of Arya Samaj is to, “eradicate Ignorance (Agyan),

Indigence or Poverty (Abhav) and Injustice (Anayay) from this earth.

Revision Through MCQs (RTM) Compilation (Feb - 2022)

Telegram: https://t.me/insightsIAStips Youtube: https://www.youtube.com/channel/UCpoccbCX9GEIwaiIe4HLjwA

150

o Members of the Arya Samaj believe in one God and reject the worship

of idols.

Refer: https://www.insightsonindia.com/2021/06/16/ram-prasad-bismil/

230. Consider the following pairs:

Tradition State

1. Shigmo Festival Manipur

2. Chillai Kalan Jharkhand

3. Raja Parba festival Odisha

Which of the above pairs is/are correctly matched?

(a) 1 and 2 only

(b) 3 only

(c) 2 and 3 only

(d) 1 and 3 only

Ans: (b)

Explanation

• S3: Raja Parba festival of Odisha:

o This is a three-day-long festival dedicated to Mother Earth (Bhuma

Devi) and womanhood at large.

o The first day of the festival is called Pahili Raja, the second is Mithuna

Sankranti and the third Bhu daha or Basi Raja.

o The fourth and final day is called Vasumati snana (bathing of Mother

Earth).

o This festival is also associated with the end of the summer season and

the arrival of the monsoon.

• S2: Chillai Kalan:: Kashmir valley read here>>

• S1: Shigmo festival:: Goa read here>>

Refer: facts for prelims: https://www.insightsonindia.com/2021/06/16/high-level-

dialogue-on-desertification-land-degradation-and-drought-dldd/

Revision Through MCQs (RTM) Compilation (Feb - 2022)

Telegram: https://t.me/insightsIAStips Youtube: https://www.youtube.com/channel/UCpoccbCX9GEIwaiIe4HLjwA

151